Sie sind auf Seite 1von 168

LSB_F7_Rev Kit:297mm x 210mm

28/10/09

13:36

Page 1

F7
Financial
Reporting
(INT)

ACCA

LSB_F7_Rev Kit:297mm x 210mm

28/10/09

13:36

Page 2

FINANCIAL REPORTING (INTERNATIONAL)

British Library Cataloguing-in-Publication Data


A catalogue record for this book is available from the British Library
Published by InterActive World Wide Limited
Westgate House, 8-9 Holborn
London EC1N 2LL
www.iaww.com
www.studyinteractive.org
ISBN 978-1-907217-21-0
First Edition 2009
Printed in Romania

2009 InterActive World Wide Limited.


London School of Business & Finance and the LSBF logo are trademarks or registered trademarks of London
School of Business & Finance (UK) Limited in the UK and in other countries and are used under license.
All used brand names or typeface names are trademarks or registered trademarks of their respective holders.

We are grateful to the Association of Chartered Certified Accountants (ACCA), the Chartered Institute of
Management Accountants (CIMA) and the Institute of Chartered Accountants of England and Wales (ICAEW)
for their permission to reproduce past examination questions. All the solutions to these questions have been
prepared by InterActive World Wide Limited.

All our rights reserved. No part of this publication may be reproduced, stored in a retrieval system, or
transmitted, in any form or by any means, electronic, mechanical, photocopying, recording or otherwise,
without the prior written permission of InterActive World Wide.

LSB_F7_Rev Kit:297mm x 210mm

28/10/09

13:36

Page 3

FOREWORD

Foreword
Thank you for choosing to study with the London School of Business and Finance (LSBF).
A dynamic, quality-oriented and innovative educational institution, the London School of Business and Finance
offers specialised programmes, designed with students and employers in mind. We are always at the frontline
driving the latest professional developments and trends.
LSBF attracts the highest quality candidates from over 140 countries worldwide. We work in partnership with
leading accountancy firms, banks and best-practice organisations enabling thousands of students to realise
their full potential in accountancy, finance and the business world.
With an international perspective, LSBF has developed a rich portfolio of professional qualifications and executive
education programmes. To complement our face-to-face and cutting-edge online learning products, LSBF is
now pleased to offer tailored study materials to support students in their preparation for exams.
The exam focused content in this manual will provide you with a comprehensive and up-to-date understanding
of the ACCA syllabus. We have an award-winning team of tutors, who are highly experienced in helping students
through their professional exams and have received consistently excellent feedback.
I hope that you will find this manual helpful and wish you the best of luck in your studies.
Aaron Etingen
ACCA, BA, Founder and CEO

LSB_F7_Rev Kit:297mm x 210mm

28/10/09

13:36

Page 4

FINANCIAL REPORTING (INTERNATIONAL)

LSB_F7_Rev Kit:297mm x 210mm

28/10/09

13:36

Page 5

CONTENTS

Contents
Foreword

Contents

How to use this LSBF Revision Kit

About ACCA Paper F6 - Taxation (UK)

13

Questions

17

Answers

71

Feedback and Review Form

161

LSB_F7_Rev Kit:297mm x 210mm

28/10/09

13:36

Page 6

FINANCIAL REPORTING (INTERNATIONAL)

LSB_F7_Rev Kit:297mm x 210mm

28/10/09

13:36

Page 7

CONTENTS
Number

Name of Question

Highveldt

1
3
4
5
6
7
8
9

10
11
12
13
14
15
16
17
18
19
20
21
22
23
24
25
26
27
28
29
30
31
32
33
34
35
36
37
38
39
40

Hepburn

Pilot Paper 200x

Hydan

June 2006

Hedra

December 2005

Holdrite
Hosterling
Horsefield

June 2005

December 2004
December 2006
June 2002

Winger

Pilot Paper 200x

Allgone

June 2003

Petra

Tadeon
Kala

Telenorth

Tourmalet
Wellmay

December 2005
December 2006

Pilot Paper 200x


December 2001
December 2003
June 2007

Peterlee

June 2006

Broadoak

December 2001

Derringdo

June 2003

Wilderness

December 2005

Linnet

June 2004

Elite Leisure

December 2005

Torrent

June 2006

Triangle

June 2005

Bowtock
Atkins
CB

QRS

Harper
FW

LMN
EFG

Breadline

Comparator
Bigwood
Tabba

Nedberg

December 2003
December 2002
May 2005

Pilot Paper 200x


December 2002
May 2005
xxxx

Pilot Paper 200x


June 2002

December 2003
December 2004
December 2005

December 2002

Minster

December 2006

Update

June 2003

Rytetrend
Appraisal

June 2003

F7 Pilot Paper

LSB_F7_Rev Kit:297mm x 210mm

28/10/09

13:36

Page 8

FINANCIAL REPORTING (INTERNATIONAL)

LSB_F7_Rev Kit:297mm x 210mm

28/10/09

13:36

Page 9

F7
How to use
this LSBF
Revision Kit

LSB_F7_Rev Kit:297mm x 210mm

28/10/09

13:36

Page 10

FINANCIAL REPORTING (INTERNATIONAL)

10

LSB_F7_Rev Kit:297mm x 210mm

28/10/09

13:36

Page 11

HOW TO USE THIS LSBF REVISION KIT

How to use this LSBF Revision Kit


So, you have a nice big revision kit packed with questions, but do not know where to start? And when to start?
And how to start?!
Where and When to Start
This rather depends on where you have reached in your studies:

If you have not yet started your class, online program, or self-study plan, then it is a bit early to be attempting
exam-standard questions! However, it would be useful to pick out some questions at random and read
them through. Get a feel for the requirements, as these tend to be examined over and over again and this
will help you when you start your studying. Your brain will link what you read and hear with the exam
question requirements you have already seen, and this should help to strengthen your overall understanding
of how the knowledge gets tested.
If you have started your class, online program, or self-study plan, you will probably have a tutor, or study
planner, which directs you to practise specific questions, based on what you have covered so far.

Try to keep up with the suggested questions as the real exam approaches there are likely to be plenty
more exam questions that you are told to attempt, so do not fall behind! The more questions you have
practised, the more likely you are to be successful.

As the real exam approaches, do not be afraid to repeat questions you have practised before. Any good
question is worth doing at least twice!

Dont fall into the trap of starting with Q1, then Q2, Q3 etc. as this tends to result in lots of practice of the
first syllabus area and virtually no practice of anything else! Pick questions at random from throughout the
book, and keep a record of what you have attempted so far, by ticking the Question List after every attempt.

How to Start Attempting Questions


There are in fact several ways to attempt a question. Some are better methods than others, but it all rather
depends on how close you are to the exams and how your brain works.
THE FULL QUESTION ATTEMPT

To start, read ONLY the requirements:

o Break each requirement down as much as you can for example, the following requirement has three
separate things to do to earn marks:
Discuss the ethical issues in the scenario and how the directors and auditors should manage them.

Your answer would need to discuss issues, explain what the directors should do, and then explain what
the auditors should do three separate tasks, each of them carrying part of the total mark.

o Note how many marks are available. It varies by exam, but the most common allocation is one mark
for each point you adequately make in your answer. So, if you are not sure, a 9-mark requirement
probably wants you to cover 9 separate points.

o Re-read the requirements to make sure you fully understand them it is these that drive the format
of the answer.

Before you read the story/scenario (if there is one), PLAN what might go into your answer. It might be the
case that once you read the story and have a better idea of what is happening, some of your early thoughts
can be discarded. However, if you were asked to produce a tax computation, or an audit plan, or a set of
financial statements, you should have a good idea of the structure and content of your answer without
reading the detailed story.
11

LSB_F7_Rev Kit:297mm x 210mm

28/10/09

13:36

Page 12

FINANCIAL REPORTING (INTERNATIONAL)

If there is a story/scenario supporting the requirements, then clearly you should read it. But:

o Simply copying parts of the story into your answer is unlikely to earn marks unless you ADD
something yourself. Typically this means using the story to illustrate the technical point that the
requirement has forced you to discuss.

o Try to put any relevant information straight into your answer plan it is easy to highlight dozens of
interesting things, then have to read the whole story again to remember why you highlighted them!

When writing your full answer, use the requirement to structure your answer and provide any headings you
may need.

Keep your presentation neat. Most exam questions requiring written answers do NOT want a long
discursive essay. Most answers need a series of relatively short paragraphs explaining a series of points
briefly, but with enough detail to make it obvious what you are trying to explain. For more guidance on
this, look at the Answers in the back of this book.

When you have finished your attempt, take a couple of minutes to rest then review the suggested solution
in the Revision Kit. Make a note of those elements of the answer that you got wrong or missed out. The
brain tends to be a logical thing if you miss a point or make a mistake this time, there is every chance you
will make the same mistake again!

THE QUICK QUESTION ATTEMPT


Of course, in the real exam you have to provide full answers so you MUST make plenty of full attempts, as
described above.

However, especially with written questions, it is often possible to spend only 10-15 minutes on a question and
still get some real benefit.
o Simply read the requirements, and plan an answer as described above in The Full Question Attempt.
Now go and look at the answer, especially the headings and layout. If your plan is similarly structured,
then you were clearly on the right path and understood the tasks set by the examiner.

o Now go back and read the scenario. You know your plan is ok, so try to put things from the scenario
into your planned structure, wherever they seem to be most relevant. Just expand your plan but do
not write out a full answer.

This is a useful exercise as you get closer to the real exam, as it allows you to gain confidence in the examiners
requirements and also see a large number of different questions relatively quickly.
However, for questions with numbers and calculations, only a full attempt is likely to work.

The Last 4 Exam Papers


The most recent exam papers, together with the examiners own suggested solutions, can be found, for free, on
the ACCAs website at www.accaglobal.com/students/acca/exams

In our Revision Kits, additional questions have been included to ensure that you can practise things similar to
recent exam questions. Examiners tend to be very repetitive in style, but clearly they are not going to set
exactly the same story and numbers again!

12

LSB_F7_Rev Kit:297mm x 210mm

28/10/09

13:36

Page 13

F7
About ACCA
Paper F7 Financial
Reporting
(INT)

LSB_F7_Rev Kit:297mm x 210mm

28/10/09

13:36

Page 14

FINANCIAL REPORTING (INTERNATIONAL)

14

LSB_F7_Rev Kit:297mm x 210mm

28/10/09

13:36

Page 15

ABOUT ACCA PAPER F6 - TAXATION

Aim of the Paper


The aim of Paper F7, Financial Reporting is to develop knowledge and skills in understanding and applying
accounting standards and the theoretical framework in the preparation of financial statements of entities,
including groups, and how to analyse and interpret those financial statements. The paper also forms the basis
of the assumed knowledge required in Paper P2, Corporate Reporting.

Outline of the Syllabus

Conceptual framework.
Regulatory framework.
Financial Statements (11 areas).
Business Combinations.
Analysing & interpreting financial statements.

Format of the Exam Paper


Exam Paper (All Compulsory)
Q1 Consolidation including small discussion element; computations designed to test understanding of
principles (25 marks)
Q2 Preparing / Restating Financial Statements (Published A/cs) (25 marks)
Q3 Performance appraisal/interpretation and / or cash flows (25 marks)
Q4 Conceptual / regulatory framework Standards (15 marks)
Q5 Conceptual / regulatory framework Standards (10 marks)
Remember Paper F7 is ACCAs SECOND Level Financial Accounting: it is very different to Paper F3/1.1 (and
will be examined in greater depth!)

Getting the most from your studies


Candidates need to understand the theory and concepts underlying the preparation and regulation of an entity's
financial reports, to apply their knowledge of accounting standards to prepare financial statements of both single
entities and groups, and finally, to demonstrate their analytical skills to assess the performance of entities based
on the information provided by those financial statements.

15

LSB_F7_Rev Kit:297mm x 210mm

28/10/09

13:36

Page 16

FINANCIAL REPORTING (INTERNATIONAL)

16

LSB_F7_Rev Kit:297mm x 210mm

28/10/09

13:36

Page 17

F7
Questions

LSB_F7_Rev Kit:297mm x 210mm

28/10/09

13:36

Page 18

FINANCIAL REPORTING (INTERNATIONAL)

18

LSB_F7_Rev Kit:297mm x 210mm

28/10/09

13:36

Page 19

QUESTIONS

1.

Hepburn

(a)

On 1 October 2008 Hepburn acquired 80% of the equity share capital of Salter by way of a share exchange.
Hepburn issued five of its own shares for every two shares in Salter. The market value of Hepburn's shares on
1 October 2008 was $3 each. The share issue has not yet been recorded in Hepburn's books. The summarised
financial statements of both companies are:
INCOME STATEMENTS
YEAR TO 31 MARCH 2009

Hepburn
$'000
1,200
(650)
550
(120)
Nil
430
(100)
330

Sales revenues
Cost of sales
Gross profit
Operating expenses
Debenture interest
Profit before tax
Income tax expense
Profit for the year

STATEMENTS OF FINANCIAL POSITION


AS AT 31 MARCH 2009
Assets
Non-current assets
Property, plant and equipment
Investments
Current assets
Inventory
Accounts receivable
Bank

$'000

Total equity and liabilities

The following information is relevant:

(i)

$'000

$'000

620
20
640
240
170
20

Total assets
Equity and liabilities
Equity
Equity shares of $1 each
Retained earnings
Non-current liabilities
8% debentures
Current liabilities
Trade accounts payable
Current tax payable

Hepburn

Salter
$'000
1,000
(660)
340
(88)
(12)
240
(40)
200

170
50

430
1,070

Salter

$'000
660
10
670

280
210
40

530
1,200

400
450
850

150
700
850

nil

150

220
1,070

155
45

200
1,200

The fair values of Salter's assets were equal to their book values with the exception of its land, which
had a fair value of $125,000 in excess of its book value at the date of acquisition.
19

LSB_F7_Rev Kit:297mm x 210mm

28/10/09

13:36

Page 20

FINANCIAL REPORTING (INTERNATIONAL)


(ii)
(iii)
(iv)
(v)

In the post acquisition period Hepburn sold goods to Salter at a price of $100,000, this was calculated
to give a mark-up on cost of 25% to Hepburn. Salter had half of these goods in inventory at the year end.
Consolidated goodwill is reviewed annually for impairment. At 31 March 2009 its impaired value was
$180,000.

The current accounts of the two companies disagreed due to a cash remittance of $20,000 to Hepburn
on 26 March 2009 not being received until after the year end. Before adjusting for this, Salter's debit
balance in Hepburn's books was $56,000.
The non-controlling interest is measured using the proportion of net assets method.

Required

(b)

Prepare a consolidated income statement and statement of financial position for Hepburn for the year to
31 March 2009. (20 marks)

At the same date as Hepburn made the share exchange for Salters shares, it also acquired 6,000 'A' shares in
Woodbridge for a cash payment of $20,000. The share capital of Woodbridge is made up of:
Equity voting A shares
Equity non-voting B shares

10,000
14,000

All of Woodbridges equity shares are entitled to the same dividend rights; however during the year to
31 March 2009 Woodbridge made substantial losses and did not pay any dividends.

Hepburn has treated its investment in Woodbridge as an ordinary long-term investment on the basis that:
(i)
(ii)
(iii)

It is only entitled to 25% of any dividends that Woodbridge may pay;


It does not have any directors on the board of Woodbridge;
It does not exert any influence over the operating policies or management of Woodbridge.

Required

Comment on the accounting treatment of Woodbridge by Hepburns directors and state how you believe the
investment should be accounted for. (5 marks)
Note. You are not required to amend your answer to part (a) in respect of the information in part (b).

(25 marks)

2.

Highveldt
Highveldt, a public listed company, acquired 75% of Samsons ordinary shares on 1 April 2008. Highveldt paid an
immediate $3.50 per share in cash and agreed to pay a further amount of $108 million on 1 April 2009.
Highveldts cost of capital is 8% per annum. Highveldt has only recorded the cash consideration of $3.50 per
share.
The summarised statements of financial positions of the two companies at 31 March 2009 are shown below:

Tangible non-current assets (note (i))


Development costs (note (iv))
Investments (note (ii))
Current assets
Total assets

20

Highveldt
$m
420
nil
300
720
133
853

Samson
$m
320
40
20
380
91
471

LSB_F7_Rev Kit:297mm x 210mm

28/10/09

13:36

Page 21

QUESTIONS

Equity and liabilities


Ordinary shares of $1 each
Reserves:
Share premium
Revaluation reserve
Retained earnings 1 April 2008
year to 31 March 2009
Non-current liabilities
10% inter company loan (note (ii))
Current liabilities
Total equity and liabilities

The following information is relevant:


(i)

(ii)

(iii)
(iv)

(v)
(vi)
(vii)

$m

160
190

Highveldt

$m

$m

Samson

$m

270

80

80
45

40
nil

350
745

134
76

nil
108
853

210
330
60
81
471

Highveldt has a policy of revaluing land and buildings to fair value. At the date of acquisition Samsons
land and buildings had a fair value $20 million higher than their book value and at 31 March 2009 this
had increased by a further $4 million (ignore any additional depreciation).

Included in Highveldts investments is a loan of $60 million made to Samson at the date of acquisition.
Interest is payable annually in arrears. Samson paid the interest due for the year on 31 March 2009, but
Highveldt did not receive this until after the year end. Highveldt has not accounted for the accrued
interest from Samson.

Samson had established a line of products under the brand name of Titanware. Acting on behalf of
Highveldt, a firm of specialists, had valued the brand name at a value of $40 million with an estimated life
of 10 years as at 1 April 2008. The brand is not included in Samsons statement of financial position.

Samsons development project was completed on 30 September 2008 at a cost of $50 million.
$10 million of this had been amortised by 31 March 2009. Development costs capitalised by Samson at
the date of acquisition were $18 million. Highveldts directors are of the opinion that Samsons
development costs do not meet the criteria in IAS 38 Intangible Assets for recognition as an asset.

Samson sold goods to Highveldt during the year at a profit of $6 million, one-third of these goods were
still in the inventory of Highveldt at 31 March 2009.

An impairment test at 31 March 2009 on the consolidated goodwill concluded that it should be written
down by $22 million. No other assets were impaired.
It is group policy to measure the non-controlling interest using the proportion of net assets method.

Required
(a)

Calculate the following figures as they would appear in the consolidated statement of financial position
of Highveldt at 31 March 2009:
(i)
(ii)
(iii)

(b)

Goodwill; (8 marks)
Non-controlling interest; (4 marks)
The following consolidated reserves:
share premium, revaluation surplus and retained earnings. (8 marks)

Note: Show your workings.

Explain why consolidated financial statements are useful to the users of financial statements (as opposed
to just the parent companys separate (entity) financial statements). (5 marks)

(25 marks)
21

LSB_F7_Rev Kit:297mm x 210mm

28/10/09

13:36

Page 22

FINANCIAL REPORTING (INTERNATIONAL)

3.

Hydan
On 1 October 2008 Hydan, a publicly listed company, acquired a 60% controlling interest in Systan paying $9
per share in cash. Prior to the acquisition Hydan had been experiencing difficulties with the supply of
components that it used in its manufacturing process. Systan is one of Hydans main suppliers and the
acquisition was motivated by the need to secure supplies. In order to finance an increase in the production
capacity of Systan, Hydan made a non-dated loan at the date of acquisition of $4 million to Systan that carried
an actual and effective interest rate of 10% per annum. The interest to 31 March 2009 on this loan has been
paid by Systan and accounted for by both companies. The summarised draft financial statements of the
companies are:
INCOME STATEMENTS FOR THE YEAR ENDED 31 MARCH 2009
Hydan

Revenue
Cost of sales
Gross profit
Operating expenses
Interest income
Finance costs
Profit/(loss) before tax
Income tax (expense)/relief
Profit/(loss) for the year

$000
98,000
(76,000)
22,000
(11,800)
350
(420)
10,130
(4,200)
5,930

STATEMENTS OF FINANCIAL POSITIONS AS AT 31 MARCH 2009


Non-current assets
Property, plant and equipment
Investments (including loan to Systan)
Current assets
Total assets
Equity and liabilities
Ordinary shares of $1 each
Share premium
Retained earnings
Non-current liabilities
7% Bank loan
10% loan from Hydan
Current liabilities
Total equity and liabilities

The following information is relevant:


(i)

22

preacquisition
$000
24,000
(18,000)
6,000
(1,200)
nil
nil
4,800
(1,200)
3,600

Systan

postacquisition
$000
35,200
(31,000)
4,200
(8,000)
nil
(200)
(4,000)
1,000
(3,000)

Hydan
$000

Systan
$000

18,400
16,000
34,400
18,000
52,400

9,500
nil
9,500
7,200
16,700

10,000
5,000
20,000
35,000

2,000
500
6,300
8,800

6,000
nil
11,400
52,400

nil
4,000
3,900
16,700

At the date of acquisition, the fair values of Systans property, plant and equipment were $1.2 million
in excess of their carrying amounts. This will have the effect of creating an additional depreciation
charge (to cost of sales) of $300,000 in the consolidated financial statements for the year ended
31 March 2009. Systan has not adjusted its assets to fair value.

LSB_F7_Rev Kit:297mm x 210mm

28/10/09

13:36

Page 23

QUESTIONS
(ii)

(iii)
(iv)
(v)

(vi)

In the post acquisition period Systans sales to Hydan were $30 million on which Systan had made a
consistent profit of 5% of the selling price. Of these goods, $4 million (at selling price to Hydan) were
still in the inventory of Hydan at 31 March 2009. Prior to its acquisition Systan made all its sales at a
uniform gross profit margin.

Included in Hydans current liabilities is $1 million owing to Systan. This agreed with Systans receivables
ledger balance for Hydan at the year end.
An impairment review of the consolidated goodwill at 31 March 2009 revealed that its current value
was $375,000 less than its carrying amount.
Neither company paid a dividend in the year to 31 March 2009.

It is Hydan Group policy to measure the non-controlling interest at fair value. At the date of acquisition
of Systan, the fair value of a 40% holding was $5.5million

Required
(a)

(b)

Prepare the consolidated income statement for the year ended 31 March 2009 and the consolidated
statement of financial position at that date. (20 marks)

Discuss the effect that the acquisition of Systan appears to have had on Systans operating performance.
(5 marks)

(25 marks)

4.

Holdrite
Holdrite purchased 75% of the issued share capital of Staybrite and 40% of the issued share capital of Allbrite
on 1 April 2009.
Details of the purchase consideration given at the date of purchase are:

Staybrite: a share exchange of two shares in Holdrite for every three shares in Staybrite plus an issue to the
shareholders of Staybrite 8% loan notes redeemable at par on 30 June 2011 on the basis of $100 loan
note for every 250 shares held in Staybrite.
Allbrite: a share exchange of three shares in Holdrite for every four shares in Allbrite plus $1 per share
acquired in cash. The market price of Holdrites shares at 1April 2009 was $6 per share.
The summarised income statements for the three companies for the year to 30 September 2009 are:
Revenue
Cost of Sales
Gross Profit
Operating expenses
Operating Profit
Interest expense
Profit before tax
Income tax expense
Profit for year

The following information is relevant:

Holdrite
$000
75,000
(47,400)
27,600
(10,480)
17,120
(170)
16,950
(4,800)
12,150

Staybrite
$000
40,700
(19,700)
21,000
(9,000)
12,000

Allbrite
$000
31,000
(15,300)
15,700
(9,700)
6,000

12,000
(3,000)
9,000

6,000
(2,000)
4,000

(i) A fair value exercise was carried out for Staybrite at the date of its acquisition with the following results:
Land
Plant

Book value
$000
20,000
25,000

Fair value
$000
23,000
30,000

23

LSB_F7_Rev Kit:297mm x 210mm

28/10/09

13:36

Page 24

FINANCIAL REPORTING (INTERNATIONAL)


The fair values have not been reflected in Staybrites financial statements. The increase in the fair value of the
plant would create additional depreciation of $500,000 in the post acquisition period in the consolidated
financial statements to 30 September 2009.
Depreciation of plant is charged to cost of sales.
(ii)

The details of each companys share capital and reserves at 1 October 2008 are:

Equity shares of $1 each


Share premium
Retained earnings

Holdrite
$000
20,000
5,000
18,000

Staybrite
$000
10,000
4,000
7,500

Allbrite
$000
5,000
2,000
6,000

In the post acquisition period Holdrite sold goods to Staybrite for $10 million. Holdrite made a profit
of $4 million on these sales. One-quarter of these goods were still in the inventory of Staybrite at 30
September 2009.

(iii)
(iv)

Impairment tests on the goodwill of Staybrite and the investment in Allbrite at 30 September 2009
resulted in the need to write down Staybrites goodwill by $750,000.

(v)

Holdrite paid a dividend of $5 million on 20 September 2009. Staybrite and Allbrite did not make any
dividend payments.

(vi)

Holdrite Group measure the non-controlling interest using the proportion of net assets method.

Required
(a)

Calculate the goodwill arising on the purchase of the shares in Staybrite and the carrying value of
Allbrite at 1 April 2009 and 30 September 2009. (8 marks)

(b)
(c )

5.

Prepare a consolidated income statement for the Holdrite Group for the year to 30 September 2009.
(15 marks)
Show the movement on the consolidated retained earnings attributable to Holdrite for the year to
30 September 2009. (2 marks)
(25 marks)

Hedra
Hedra, a public listed company, acquired the following investments:

(i)

(ii)

On 1 October 2008, 72 million shares in Salvador for an immediate cash payment of $195 million.
Hedra agreed to pay further consideration on 30 September 2009 if the post acquisition profits of
Salvador exceeded an agreed figure at that date. Hedra has not accounted for this deferred payment as
it did not believe it would be payable. The fair value of the payment at 1 October 2008 was $49million.
Salvador also accepted a $50 million 8% loan from Hedra at the date of its acquisition.

On 1 April 2009, 40 million shares in Aragon by way of a share exchange of two shares in Hedra for
each acquired share in Aragon. The share market value of Hedra s shares at the date of this share
exchange was $2.50. Hedra has not yet recorded the acquisition of the investment in Aragon.

The summarised statements of financial position of the three companies as at 30 September 2009 are:
Noncurrent assets
Property, plant and equipment
Investments in Salvador
other

24

$m

Hedra

$m
358
245
45
648

$m

Salvador
$m

240
nil
nil
240

$m

Aragon

$m

270
nil
nil
270
Continued on next page

LSB_F7_Rev Kit:297mm x 210mm

28/10/09

13:36

Page 25

QUESTIONS

Current assets
Inventories
Trade receivables
Cash and bank
Equity and liabilities
Ordinary share capital ($1 each)
Reserves:
Share premium
Revaluation
Retained earnings
Noncurrent liabilities
8% loan note
Deferred tax
Current liabilities
Trade payables
Bank overdraft
Current tax payable
Total equity and liabilities

The following information is relevant.


(a)

$m
130
142
nil

Hedra

$m

272
920

$m
80
97
4

400
40
15
240

nil
45
118
12
50

295
695
45

180
920

Salvador
$m

181
421

$m
110
70
20

120
50
nil
60

50
nil
141
nil
nil

110
230
50

141
421

Aragon

$m

200
470
100

nil
nil
300

nil
nil
40
nil
30

300
400
nil

70
470

Fair value adjustments and revaluations:

(i) Hedras accounting policy for land and buildings is that they should be carried at their fair values. The
fair value of Salvadors land at the date of acquisition was $20 million in excess of its carrying value.
By 30 September 2009 this excess had increased by a further $5 million. Salvadors buildings did not
require any fair value adjustments. The fair value of Hedras own land and buildings at 30 September
2009 was $12 million in excess of its carrying value in the above statement of financial position.
(ii) The fair value of some of Salvadors plant at the date of acquisition was $20 million in excess of its
carrying value and had a remaining life of four years (straightline depreciation is used).

(b)
(c)
(d)

(iii)At the date of acquisition Salvador had unrelieved tax losses of $40 million from previous years.
Salvador had not accounted for these as a deferred tax asset as its directors did not believe the
company would be sufficiently profitable in the near future. However, the directors of Hedra were
confident that these losses would be utilised and accordingly they should be recognised as a deferred
tax asset. By 30 September 2009 the group had not yet utilised any of these losses. The income tax
rate is 25%.

The retained earnings of Salvador and Aragon at 1 October 2008, as reported in their separate financial
statements, were $20 million and $200 million respectively. All profits are deemed to accrue evenly
throughout the year.

Hedras policy is to value non-controlling interests at their fair values. The Directors of Hedra assessed
the fair value of the non-controlling interest in Salvador at the date of acquisition to be $110 million.

An impairment test on 30 September 2009 showed that consolidated goodwill should be written down
by $20million. Hedra has applied IFRS 3 Business combinations since the acquisition of Salvador.

25

LSB_F7_Rev Kit:297mm x 210mm

28/10/09

13:36

Page 26

FINANCIAL REPORTING (INTERNATIONAL)


(e)

The investment in Aragon has not suffered any impairment.

Required

Prepare the consolidated statement of financial position of Hedra as at 30 September 2009.

6.

(25 marks)

Hosterling
Hosterling purchased the following equity investments:

On 1 October 2008: 80% of the issued share capital of Sunlee. The acquisition was through a share exchange
of three shares in Hosterling for every five shares in Sunlee. The market price of Hosterling's shares at
1 October 2008 was $5 per share.

On 1 July 2009: 6 million shares in Amber paying $3 per share in cash and issuing to Amber's shareholders 6%
(actual and effective rate) loan notes on the basis of $100 loan note for every 100 shares acquired.
The summarised income statements for the three companies for the year ended 30 September 2009 are:
Revenue
Cost of sales
Gross profit/(loss)
Other income (note (i))
Distribution costs
Administrative expenses
Finance costs
Profit/(loss) before tax
Income tax (expense)/credit
Profit/(loss) for the year

The following information is relevant:


(i)

(ii)

(iii)

Hosterling
$'000
105,000
(68,000)
37,000
400
(4,000)
(7,500)
(1,200)
24,700
(8,700)
16,000

Sunlee
$'000
62,000
(36,500)
25,500
nil
(2,000)
(7,000)
(900)
15,600
(2,600)
13,000

The other income is a dividend received from Sunlee on 31 March 2009.

The details of Sunlee's and Amber's share capital and reserves at 1 October 2008 were:
Equity shares of $1 each
Retained earnings

Sunlee
$'000
20,000
18,000

Amber
$'000
15,000
35,000

Intellectual property
Land
Plant

Carrying
amount
$'000
18,000
17,000
30,000

Fair value
$'000
22,000
20,000
35,000

A fair value exercise was carried out at the date of acquisition of Sunlee with the following results:

The intellectual property is still in development.

The fair values have not been reflected in Sunlee's financial statements.
Plant depreciation is included in cost of sales.

No fair value adjustments were required on the acquisition of Amber.


26

Amber
$'000
50,000
(61,000)
(11,000)
nil
(4,500)
(8,500)
nil
(24,000)
4,000
(20,000)

Remaining life
(straight line)

see below
not applicable
five years

LSB_F7_Rev Kit:297mm x 210mm

28/10/09

13:36

Page 27

QUESTIONS
(iv)
(v)
(vi)

(vii)

In the year ended 30 September 2009 Hosterling sold goods to Sunlee at a selling price of $18 million.
Hosterling made a profit of cost plus 25% on these sales. $7.5 million (at cost to Sunlee) of these goods
were still in the inventories of Sunlee at 30 September 2009.

Impairment tests for both Sunlee and Amber were conducted on 30 September 2009. They concluded
that the goodwill of Sunlee should be written down by $1.6 million and, due to its losses since
acquisition, the investment in Amber was worth $21.5 million.
All trading profits and losses are deemed to accrue evenly throughout the year.

The non-controlling interest is measured using the proportion of net assets method.

Required
(a)

(b )
(c)

Calculate the goodwill arising on the acquisition of Sunlee at 1 October 2008. (5 marks)

Calculate the carrying amount of the investment in Amber at 30 September 2009 under the equity
method prior to the impairment test. (4 marks)

Prepare the consolidated income statement for the Hosterling Group for the year ended 30 September
2009. (16 marks)

(25 marks)

7.

Horsefield
Horsefield, a public company, acquired 90% of Sandfly's $1 ordinary shares on 1 April 2007 paying $3.00 per
share. The balance on Sandfly's retained earnings at this date was $800,000, and the fair value of a 10%
shareholding was $250,000. On 1 October 2008, Horsefield acquired 30% of Anthill's $1 ordinary shares for
$3.50 per share. The statements of financial position of the three companies at 31 March 2009 are shown
below.
Horsefield
Sandfly
Anthill
$'000
$'000
$'000
$'000
$'000
$'000
Non-current assets
Property, plant and equipment
8,050
3,600
1,650
Investments
910
nil
4,000
12,050
4,510
1,650
Current assets
Inventory
830
340
250
Accounts receivable
520
290
350
Bank
240
nil
100
630
700
1,590
Total assets
13,640
5,140
2,350
Equity and liabilities
Equity
Ordinary shares of $1 each
5,000
1,200
600
Reserves:
Retained earnings b/f
6,000
1,400
800
Profit year to 31 March 2009
1,500
900
600
7,500
2,300
1,400
12,500
3,500
2,000

27

LSB_F7_Rev Kit:297mm x 210mm

28/10/09

13:36

Page 28

FINANCIAL REPORTING (INTERNATIONAL)


Non-current liabilities
10% loan notes
Current liabilities
Accounts payable
Taxation
Overdraft

500
420
220
nil

Total equity and liabilities

240
960
250
190

640
13,640

1,400
5,140

nil
200
150
nil

350
2,350

The following information is relevant.


(i)

Fair value adjustments

On 1 April 2007 Sandfly owned a property that had a fair value of $120,000 in excess of its book value.
The value of this property has not changed since acquisition.

(ii )
(iii )

(iv)
(v)

Just prior to its acquisition, Sandfly was successful in applying for a six-year licence to dispose of
hazardous waste. The licence was granted by the government at no cost. However, Horsefield
estimated that the licence was worth $180,000 at the date of acquisition.

In January 2009 Horsefield sold goods to Anthill for $65,000. These were transferred at a mark up of
30% on cost. Two thirds of these goods were still in the inventory of Anthill at 31 March 2009.

To facilitate the consolidation procedures the group insists that all intragroup current account balances
are settled prior to the year-end. However, a cheque for $40,000 from Sandfly to Horsefield was not
received until early April 2009. Intragroup balances are included in accounts receivable and payable as
appropriate.
There are no indications that goodwill has been impaired.
Anthill is to be treated as an associate of Horsefield.

(vi)

The full goodwill method is used to measure the non-controlling interest.

(a)

Prepare the consolidated statement of financial position of Horsefield as at 31 March 2009 in


accordance with IFRS 3 Business combinations. (20 marks)

Required

(b)

Discuss the matters to consider in determining whether an investment in another company constitutes
associate status. (5 marks)

(25 marks)

8.

Winger
The following list of account balances relates to Winger at 31 March 2009.
Sales revenue (note a)
Cost of sales
Distribution costs
Administration expenses
Lease rentals (note b)
Loan interest paid
Dividend paid
Property at cost (note c)
Plant and equipment cost
Depreciation 1 April 2008 - plant and equipment

28

$'000

185,050
28,700
15,000
20,000
2,000
12,000
200,000
154,800

$'000
358,450

34,800

LSB_F7_Rev Kit:297mm x 210mm

28/10/09

13:36

Page 29

QUESTIONS
Development expenditure (note d)
Profit on disposal of non-current assets
Trade accounts receivable
Inventories: 31 March 2009
Cash and bank
Trade accounts payable
Taxation: over provision in year to 31 March 2008
Equity shares of 25c each
8% loan notes (issued in 2006)
Retained earnings 1 April 2008
The following notes are relevant.
(a)

(b)

(c)

$'000
30,000
55,000
28,240
10,660

741,450

$'000
45,000

29,400
2,200
150,000
50,000
71,600
741,450

Included in sales revenue is $27 million, which relates to sales made to customers under sale or return
agreements. The expiry date for the return of these goods is 30 April 2009. Winger has charged a
mark-up of 20% on cost for these sales.

A lease rental of $20 million was paid on 1 April 2008. It is the first of five annual payments in advance
for the rental of an item of equipment that has a cash purchase price of $80 million. The auditors have
advised that this is a finance lease and have calculated the implicit interest rate in the lease as 12% per
annum. Leased assets should be depreciated on a straight-line basis over the life of the lease.

On 1 April 2008 Winger acquired a new property at a cost of $200 million. For the purpose of
calculating depreciation only, the asset has been separated into the following elements.
Separate asset

Land
Heating system
Lifts
Building

Cost
$'000
50,000
20,000
30,000
100,000

Life

freehold
10 years
15 years
50 years

The depreciation of the elements of the property should be calculated on a straight-line basis. The new
property replaced an existing one that was sold on the same date for $95 million. It had cost $50
million and had a carrying value of $80 million at the date of sale. The profit on this property has been
calculated on the original cost. It had not been depreciated on the basis that the depreciation charge
would not be material.
(d)

(e)

Plant and machinery is depreciated at 20% on the reducing balance basis.

The figure for development expenditure in the list of account balances represents the amounts deferred
in previous years in respect of the development of a new product. Unfortunately, during the current
year, the government has introduced legislation which effectively bans this type of product. As a
consequence of this the project has been abandoned. The directors of Winger are of the opinion that
writing off the development expenditure, as opposed to its previous deferment, represents a change of
accounting policy and therefore wish to treat the write off as a prior period adjustment.
A provision for income tax for the year to 31 March 2009 of $15 million is required.

Required
(a)

(b)
(c )

Prepare Wingers income statement for the year to 31 March 2009, along with the changes in retained
earnings from the statement of changes in equity. (9 marks)

Prepare a statement of financial position as at 31 March 2009 in accordance with International Financial
Reporting Standards as far as the information permits. (11 marks)
Discuss the acceptability of the company's previous policy in respect of non-depreciation of property.
(5 marks)

(25 marks)

29

LSB_F7_Rev Kit:297mm x 210mm

28/10/09

13:36

Page 30

FINANCIAL REPORTING (INTERNATIONAL)

9.

Petra
The following trial balance relates to Petra, a public listed company, at 30 September 2009:
Revenue (note (i))
Cost of sales (note (i))
Distribution costs
Administration expenses
Loan interest paid
Ordinary shares of 25 cents each fully paid
Share premium
Retained earnings 1 October 2008
6% Redeemable loan note (issued in 2007)
Land and buildings at cost (land element $40 million) note (ii))
Plant and equipment at cost (note (iii))
Deferred development expenditure (note (iv))
Accumulated depreciation at 1 October 2008 buildings
plant and equipment
Accumulated amortisation of development expenditure at 1 October 2008
Income tax (note (v))
Deferred tax (note (v))
Trade receivables
Inventories 30 September 2009
Cash and bank
Trade payables
The following notes are relevant.

(i)

(ii)
(iii)

(iv)

(v)

30

$'000

114,000
17,000
18,000
1,500

100,000
66,000
40,000

1,000
24,000
21,300
11,000
413,800

$'000
197,800

40,000
12,000
34,000
50,000

16,000
26,000
8,000
15,000

15,000
413,800

Included in revenue is $12 million for receipts that the companys auditors have advised are commission
sales. The costs of these sales, paid for by Petra, were $8 million. $3million of the profit of $4 million
was attributable to and remitted to Sharma (the auditors have advised that Sharma is the principal for
the transactions). Both the $8 million cost of sales and the $3 million paid to Sharma have been
included in cost of sales.
The buildings had an estimated life of 30 years when they were acquired and are being depreciated on
the straightline basis.

Included in the trial balance figures for plant and equipment is plant that had cost $16 million and had
accumulated depreciation of $6 million at 1 October 2008. Following a review of the companys
operations this plant was made available for sale at the beginning of the year. Negotiations with a
broker have concluded that a realistic selling price of this plant will be $7.5 million and the broker will
charge a commission of 8% of the selling price. The plant had not been sold by the year end. Plant is
depreciated at 20% per annum using the reducing balance method. Depreciation of buildings and plant
is charged to cost of sales.

The development expenditure relates to the capitalised cost of developing a product called the Topaz.
It had an original estimated life of five years. Production and sales of the Topaz started in October 2007.
A review of the sales of the Topaz in late September 2009, showed them to be below forecast and an
impairment test concluded that the fair value of the development costs at 30 September 2009 was only
$18 million and the expected period of further sales (from this date) was only a further two years.

The balance on the income tax account in the trial balance is the underprovision in respect of the
income tax liability for the year ended 30 September 2008. The directors have estimated the provision
for income tax for the year ended 30 September 2009 to be $4 million and the required provision for
deferred tax at 30 September 2009 is $17.6 million.

LSB_F7_Rev Kit:297mm x 210mm

28/10/09

13:36

Page 31

QUESTIONS
Required

Prepare for Petra:

(a)

(b)
(c)

An income statement for the year ended 30 September 2009; and (10 marks)
A statement of financial position as at 30 September 2009 (10 marks)

Note. A statement of changes in equity is NOT required. Disclosure notes are NOT required.

The directors hold options to purchase 24 million shares for a total of $7.2 million. The options were
granted two years ago and have been correctly accounted for. The options do not affect your answer to
(a) and (b) above. The average stock market value of Petras shares for the year ended 30 September
2009 can be taken as 90 cents per share.

Required

A calculation of the basic and diluted earnings per share for the year ended 30 September 2009
(comparatives are not required) (5 marks)

(25 marks)

10. Allgone
The following trial balance relates to Allgone at 31 March 2009:
Sales revenue (note (i))
Purchases
Operating expenses
Loan interest paid
Preference dividend
Land and buildings at valuation (note (ii))
Plant and equipment cost
Software cost 1 April 2006
Available-for-sale investment value at 1 April 2008 (note (iii))
Depreciation 1 April 2008 plant and equipment
Depreciation 1 April 2008 software
Costs of fraud (note (iv))
Trade receivables
Inventory 1 April 2008
Bank
Trade payables
Ordinary shares of 25c each
10% Preference shares (redeemable)
12% Loan note (issued 1 July 2008)
Deferred tax
Revaluation reserve (relating to land and buildings and the investments)
Retained earnings 2008
The following notes are relevant:
(i)

$'000
127,850
12,400
2,400
1,000
130,000
84,300
10,000
12,000
32,000
23,000
19,450

454,400

$'000
236,200

24,300
6,000

350
15,200
60,000
20,000
40,000
3,000
45,000
4,350
454,400

Sales include $8 million for goods sold in March 2009 for cash to Funders, a merchant bank. The cost of
these goods was $6 million. Funders has the option to require Allgone to repurchase these goods
within one month of the year-end at their original selling price plus a facilitating fee of $250,000.
31

LSB_F7_Rev Kit:297mm x 210mm

28/10/09

13:36

Page 32

FINANCIAL REPORTING (INTERNATIONAL)

(ii)

The inventory at 31 March 2009 was counted at a cost value of $85 million. This includes $500,000 of
slow moving inventory that is expected to be sold for a net $300,000.
Non-current assets

On 1 April 2008 Allgone revalued its land and buildings. The details are:
Land
Building

Cost 1 April 2003


$'000
20,000
80,000

Valuation 1 April 2008


$'000
25,000
105,000

The building had an estimated life of 40 years when it was acquired and this has not changed as a result
of the revaluation. Depreciation is on a straight-line basis. The surplus on the revaluation has been added
to the revaluation reserve, but no other movements on the revaluation reserve have been recorded.
Plant and equipment is depreciated at 20% per annum on the reducing balance basis.

(iii)

(iv)

(v)

(vi)

Software is depreciated by the sum of the digits method over a 5-year life.

The investment represents 7.5% of the ordinary share capital of Wondaworld. Changes in the fair value
of the investment are recognised in accordance with IAS 39. Prior to 1 April 2008, an increase of
$5million in the fair value had been recognised. The stock market price of Wondaworlds ordinary
shares was $2.50 each on 1 April 2008 and by 31 March 2009 this had fallen to $2.25.
A $32m fraud was discovered during this financial year. A senior employee of the company, who left in
January 2008, had diverted investment funds into his private bank account. The fraud was discovered by
the employees replacement in April 2008. It is unlikely that any of the funds will be recovered. Allgone
has now implemented tighter procedures to prevent such a fraud recurring. The company has been
advised that this loss will not qualify for any tax relief. The directors have asked if this loss can be
treated as an extraordinary item.
The directors have estimated the provision for income tax for the year to 31 March 2009 at $11.3
million. The deferred tax liability at 31 March 2009 is to be adjusted to reflect the tax base of the
companys net assets being $16 million less than their carrying values. The rate of income tax is 30%.
The movement on deferred tax should be charged to the income statement.
The finance charge relating to the preference shares is $2,000,000 per annum.

Required

In accordance with International Accounting Standards and International Financial Reporting Standards as far as
the information permits, prepare:
(a)

(b)
(c)

The income statement and statement of comprehensive income of Allgone for the year to 31 March
2009 (7 marks)
The statement of Changes in Equity for the year to 31 March 2009 (5 marks)
A statement of financial position as at 31 March 2009 (13 marks)

(25 marks)

Notes to the financial statements are not required.

11. Tadeon
The following trial balance relates to Tadeon, a publicly listed company, at 30 September 2009:

32

Revenue
Cost of sales
Operating expenses
Loan interest paid (note (i))
Rental of vehicles (note (ii))

$'000

118,000
40,000
1,000
6,200

$'000
277,800

LSB_F7_Rev Kit:297mm x 210mm

28/10/09

13:36

Page 33

QUESTIONS
Investment income
25 year leasehold property at cost (note (iii))
Plant and equipment at cost
Investments at amortised cost
Accumulated depreciation at 1 October 2008

$'000

leasehold property
plant and equipment

Equity shares of 20 cents each fully paid


Retained earnings at 1 October 2008
2% Loan note (note (i))
Deferred tax balance 1 October 2008 (note (iv))
Trade receivables
Inventories at 30 September 2009
Bank
Trade payables
Suspense account (note (v))
The following notes are relevant:
(i)

(ii)

225,000
181,000
42,000

53,500
33,300

700,000

$'000
2,000

36,000
85,000
150,000
18,600
50,000
12,000
1,900
18,700
48,000
700,000

The loan note was issued on 1 October 2008. It is redeemable on 30 September 2013 at a large
premium (in order to compensate for the low nominal interest rate). The finance department has
calculated that the effective interest rate on the loan is 5.5% per annum.

The rental of the vehicles relates to two separate contracts. These have been scrutinised by the finance
department and they have come to the conclusion that $5 million of the rentals relate to a finance lease.
The finance lease was entered into on 1 October 2008 (the date the $5 million was paid) for a four year
period. The vehicles had a fair value of $20 million (straight-line depreciation should be used) at
1 October 2008 and the lease agreement requires three further annual payments of $6 million each on
the anniversary of the lease. The interest rate implicit in the lease is to be taken as 10% per annum.
(Note: you are not required to calculate the present value of the minimum lease payments.) The other
contract is an operating lease and should be charged to operating expenses.
Other plant and equipment is depreciated at 121/2% per annum on the reducing balance basis.

(iii)
(iv)
(v)

All depreciation of property, plant and equipment is charged to cost of sales.

On 30 September 2009 the leasehold property was revalued to $200 million. The directors wish to
incorporate this valuation into the financial statements.

The directors have estimated the provision for income tax for the year ended 30 September 2009 at
$38 million. At 30 September 2009 there were $74 million of taxable temporary differences, of which
$20 million related to the revaluation of the leasehold property (see (iii) above). The income tax rate is 20%.
The suspense account balance can be reconciled from the following transactions:

The payment of a dividend in October 2008. This was calculated to give a 5% yield on the company's
share price of 80 cents as at 30 September 2008.

The net receipt in March 2009 of a fully subscribed rights issue of one new share for every three held at
a price of 32 cents each. The expenses of the share issue were $2 million and should be charged to
share premium.
Note. The cash entries for these transactions have been correctly accounted for.

33

LSB_F7_Rev Kit:297mm x 210mm

28/10/09

13:36

Page 34

FINANCIAL REPORTING (INTERNATIONAL)


Required

Prepare for Tadeon:


(a)

(b)

A statement of comprehensive income for the year ended 30 September 2009; and (10 marks)
A statement of financial position as at 30 September 2009. (15 marks)

Note. A statement of changes in equity is not required. Disclosure notes are not required.

(25 marks)

12. Kala
The following trial balance relates to Kala, a publicly listed company, at 31 March 2009:
Land and buildings at cost (note (i))
Plant at cost (note (i))
Investment properties valuation at 1 April 2008 (note (i))
Purchases
Operating expenses
Loan interest paid
Rental of leased plant (note (ii))
Dividends paid
Inventory at 1 April 2008
Trade receivables
Revenue
Income from investment property
Equity shares of $1 each fully paid
Retained earnings at 1 April 2008
8% (actual and effective) loan note (note (iii))
Accumulated depreciation at 1 April 2008 buildings
plant
Trade payables
Deferred tax
Bank
The following notes are relevant:

(i)

$'000
270,000
156,000
90,000
78,200
15,500
2,000
22,000
15,000
37,800
53,200
278,400

739,700

$'000

4,500
150,000
119,500
50,000
60,000
26,000
33,400
12,500
5,400
739,700

The land and buildings were purchased on 1 April 1993. The cost of the land was $70 million. No land
and buildings have been purchased by Kala since that date. 1 April 2008 Kala had its land and buildings
professionally valued at $80 million and $175 million respectively. The directors wish to incorporate
these values into the financial statements. The estimated life of the buildings was originally 50 years and
the remaining life has not changed as a result of the valuation.
Later, the valuers informed Kala that investment properties of the type Kala owned had increased in
value by 7% in the year to 31 March 2009.

(ii)

34

Plant, other than leased plant (see below), is depreciated at 15% per annum using the reducing balance
method. Depreciation of buildings and plant is charged to cost of sales.

1 April 2008 Kala entered into a lease for an item of plant which had an estimated life of five years. The
lease period is also five years with annual rentals of $22 million payable in advance from 1 April 2008.
The plant is expected to have a nil residual value at the end of its life. If purchased this plant would have
a cost of $92 million and be depreciated on a straight-line basis. The lessor includes a finance cost of
10% per annum when calculating annual rentals. (Note: you are not required to calculate the present
value of the minimum lease payments.)

LSB_F7_Rev Kit:297mm x 210mm

28/10/09

13:36

Page 35

QUESTIONS
(iii)

The loan note was issued on 1 July 2008 with interest payable six monthly in arrears.

(iv)

(v )

The provision for income tax for the year to 31 March 2009 has been estimated at $28.3 million. The
deferred tax provision at 31 March 2009 is to be adjusted to a credit balance of $14.1 million.
The inventory at 31 March 2009 was valued at $43.2 million.

Required:

Prepare for Kala:


(a)

(b)
(c)

A statement of comprehensive income for Kala for the year ended 31 March 2009. (10 marks)

A statement of changes in equity for the year ended 31 March 2009. (4 marks)
A statement of financial position as at 31 March 2009. (11 marks)

(25 marks)

13. Telenorth
The following trial balance relates to Telenorth at 30 September 2009.
Sales revenue
Inventory 1 October 2008
Purchases
Distribution expenses
Administration expenses
Loan note interest paid
Interim dividends:
ordinary
preference
Investment income
25 year leasehold building cost
Plant and equipment cost
Computer system cost
Investments at valuation
Depreciation 1 October 2008 (note (b))
Leasehold building
Plant and equipment
Computer system
Trade accounts receivable (note (c))
Bank overdraft
Trade accounts payable
Deferred tax (note (d))
Ordinary shares of $1 each
Suspense account (note (e))
6% loan notes (issued 1 October 2008)
8% preference shares (redeemable)
Revaluation surplus (note (d))
Retained earnings 1 October 2008

$'000
12,400
147,200
22,300
34,440
300
2,000
480
56,250
55,000
35,000
34,500

35,700

435,570

$'000
283,460

1,500

18,000
12,800
9,600
1,680
17,770
5,200
20,000
26,000
10,000
12,000
3,400
14,160
435,570

35

LSB_F7_Rev Kit:297mm x 210mm

28/10/09

13:36

Page 36

FINANCIAL REPORTING (INTERNATIONAL)


The following notes are relevant.
(a)

(b)

An inventory count was not conducted by Telenorth until 4 October 2009 due to operational reasons.
The value of the inventory on the premises at this date was $16 million at cost. Between the year-end
and the inventory count the following transactions have been identified.
$
Normal sales at a mark up on cost of 40%
1,400,000
Sales on a sale or return basis at a mark up on cost of 30%
650,000
Goods received at cost
820,000
All sales and purchases had been correctly recorded in the period in which they occurred.

Telenorth has the following depreciation policy.


Leasehold building straight-line

Plant and equipment five years straight line with residual values estimated at $5,000,000

Computer system 40% per annum reducing balance

(c)

Depreciation of the leasehold building and plant is treated as cost of sales; depreciation of the computer
system is an administration cost.

The outstanding account receivable of a major customer amounting to $12 million was factored to
Kwikfinance on 1 September 2009. The terms of the factoring were as follows.
(i)

(ii)
(iii)

(d)
(e)

The balance will be paid (less the charges below) when the account is collected in full. Any
amount of the account outstanding after four months will be transferred back to Telenorth
at its full book value.
Kwikfinance will charge 1.0% per month of the net amount owing from Telenorth at the
beginning of each month. Kwikfinance had not collected any of the amounts receivable by
the year end.

Telenorth debited the cash from Kwikfinance to its bank account and removed the account
receivable from its sales ledger. It has prudently charged the difference as an administration
cost.

A provision for income tax of $23.4 million for the year to 30 September 2009 is required. The
deferred tax liability is to be increased by $2.2 million, of which $1 million is to be charged direct to the
revaluation surplus.
The suspense account contains the proceeds of two share issues.
(i)

(ii)
(f)

Kwikfinance paid 80% of the outstanding account to Telenorth immediately

The exercise of all the outstanding directors' share options of four million shares on
1 October 2008 at $2 each

A fully subscribed rights issue on 1 July 2009 of 1 for 4 held at a price of $3 each. The stock
market price of Telenorth's shares immediately before the rights issue was $4.

The finance charge relating to the preference shares is equal to the dividend payable.

Required

(a) (i) The income statement of Telenorth for the year to 30 September 2009. (8 marks)

(ii) A statement of financial position as at 30 September 2009 in accordance with International Financial
Reporting Standards as far as the information permits. (12 marks)

(b)

36

Notes to the financial statements are not required.

Calculate the earnings per share in accordance with IAS 33 for the year to 30 September 2009 (ignore
comparatives). (5 marks)

(25 marks)

LSB_F7_Rev Kit:297mm x 210mm

28/10/09

13:36

Page 37

QUESTIONS

14. Tourmalet
The following extracted balances relate to Tourmalet at 30 September 2009:
Ordinary shares of 20 cents each
Retained earnings at 1 October 2008
Revaluation reserve at 1 October 2008
6% Redeemable preference shares 2011
Trade accounts payable
Tax
Land and buildings at valuation (note (iii))
Plant and equipment cost (note (v))
Investment property valuation at 1 October 2008 (note (iv))
1 October 2008 land and buildings
Depreciation 1 October 2008 plant and equipment
Trade accounts receivable
Inventory 1 October 2008
Bank
Sales revenue (note (i))
Investment income (from properties)
Purchases
Distribution expenses
Administration expenses
Interim preference dividend
Ordinary dividend paid
The following notes are relevant:
(i)

(ii)
(iii)

(iv)

(v)

$000

150,000
98,600
10,000
31,200
26,550
3,700
158,450
26,400
23,200
900
2,500
531,500

$000
50,000
47,800
18,500
30,000
35,300
2,100

9,000
24,600

313,000
1,200

531,500

Sales revenue includes $50 million for an item of plant sold on 1 June 2009. The plant had a book value
of $40 million at the date of its sale, which was charged to cost of sales. On the same date,Tourmalet
entered into an agreement to lease back the plant for the next five years (being the estimated remaining
life of the plant) at a cost of $14 million per annum payable annually in arrears. An arrangement of this
type is deemed to have a financing cost of 12% per annum. No depreciation has been charged on the
item of plant in the current year.

The inventory at 30 September 2009 was valued at cost of $285 million. This includes $45 million of
slow moving goods. Tourmalet is trying to sell these to another retailer but has not been successful in
obtaining a reasonable offer. The best price it has been offered is $2 million.

On 1 October 2005 Tourmalet had its land and buildings revalued by a firm of surveyors at $150 million,
with $30 million of this attributed to the land. At that date the remaining life of the building was
estimated to be 40 years. These figures were incorporated into the companys books. There has been
no significant change in property values since the revaluation. $500,000 of the revaluation reserve will
be realised in the current year as a result of the depreciation of the buildings and should be transferred
to retained earnings.
Details of the investment property are:

Value 1 October 2008


Value 30 September 2009

$10 million
$98 million

The company adopts the fair value method in IAS 40 Investment Property of valuing its investment
property.

Plant and equipment (other than that referred to in note (i) above) is depreciated at 20% per annum on
the reducing balance basis. All depreciation is to be charged to cost of sales.
37

LSB_F7_Rev Kit:297mm x 210mm

28/10/09

13:36

Page 38

FINANCIAL REPORTING (INTERNATIONAL)


(vi)

The above balances contain the results of Tourmalets car retailing operations which ceased on
31 December 2008 due to mounting losses. The results of the car retailing operation, which is to be
treated as a discontinued operation, for the year to 30 September 2009 are:
Sales
Cost of sales
Operating expenses (4,000 less 800 tax repayment due)

(vii)
(viii)

$000
15,200
16,000
3,200

The operating expenses are included in administration expenses in the trial balance. Tourmalet is still
paying rentals for the lease of its car showrooms. The rentals are included in operating expenses.
Tourmalet is hoping to use the premises as an expansion of its administration offices. This is dependent
on obtaining planning permission from the local authority for the change of use, however this is very
difficult to obtain. Failing this, the best option would be early termination of the lease which will cost
$1.5 million in penalties. This amount has not been provided for.
The balance on the taxation account in the trial balance is the result of the settlement of the previous
years tax charge. The directors have estimated the provision for income tax for the year to
30 September 2009 at $9.2 million.
The preference shares will be redeemed at par. The finance cost is equivalent to the annual dividend.

Required
(a)

Comment on the substance of the sale of the plant and the directors treatment of it. (5 marks)

(c)

A statement of changes in equity for Tourmalet for the year to 30 September 2009 in accordance with
current International Accounting Standards. (3 marks)

(b)

Prepare the income statement. (17 marks)

Note: A statement of financial position is NOT required. Disclosure notes are NOT required.

(25 marks)

15. Wellmay
The summarised draft financial statements of Wellmay are shown below.

INCOME STATEMENT YEAR ENDED 31 MARCH 2009

$000
4,200
(2,700)
1,500
(470
20
(55)
995
(360)
635

Revenue (note (i))


Cost of sales (note (ii))
Gross profit
Operating expenses
Investment property rental income
Finance costs
Profit before tax
Income tax
Profit for the year

STATEMENT OF FINANCIAL POSITION AS AT 31 MARCH 2009

Assets
Non-current assets
Property, plant and equipment (note (iii))
Investment property (note (iii))

38

$000

$000
4,200
400
4,600

LSB_F7_Rev Kit:297mm x 210mm

28/10/09

13:36

Page 39

QUESTIONS
$000

Current assets
Total assets
Equity and liabilities
Equity
Equity shares of 50 cents each (note (vii))
Reserves:
Retained earnings (note (iv))
Revaluation reserve

$000
1,400
6,000
1,200

2,850
350

Non-current liabilities
8% convertible loan note (2012) (note (v))
Deferred tax (note (vi))
Current liabilities
Total equity and liabilities

600
180

3,200
4,400
780
820
6,000

The following information is relevant to the draft financial statements:

(i)

(ii)
(iii)

Revenue includes $500,000 for the sale on 1 April 2008 of maturing goods to Westwood. The goods
had a cost of $200,000 at the date of sale. Wellmay can repurchase the goods on 31 March 2010 for
$605,000 (based on achieving a lenders return of 10% per annum) at which time the goods are
estimated to have a value of $750,000.

Past experience shows that after the end of the reporting period the company often receives
unrecorded invoices for materials relating to the previous year. As a result of this an accrued charge of
$75,000 for contingent costs has been included in cost of sales and as a current liability.
Non-current assets:

Wellmay owns two properties. One is a factory (with office accommodation) used by Wellmay as a
production facility and the other is an investment property that is leased to a third party under an
operating lease. Wellmay revalues all its properties to current value at the end of each year and uses the
fair value model in IAS 40 Investment property. Relevant details of the fair values of the properties are:
Valuation 31 March 2008
Valuation 31 March 2009

(iv)

Investment property
$000
400
375

The valuations at 31 March 2009 have not yet been incorporated into the financial statements. Factory
depreciation for the year ended 31 March 2009 of $40,000 was charged to cost of sales. As the factory
includes some office accommodation, 20% of this depreciation should have been charged to operating
expenses.
The balance of retained earnings is made up of:

Balance b/f 1 April 2008


Profit for the year
Dividends paid during year ended 31 March 2009
(v)

Factory
$000
1,200
1,350

8% convertible loan note (2012)

$000
2,615
635
(400)
2,850

On 1 April 2008 an 8% convertible loan note with a nominal value of $600,000 was issued at par. It is
redeemable on 31 March 2012 at par or it may be converted into equity shares of Wellmay on the basis
of 100 new shares for each $200 of loan note. An equivalent loan note without the conversion option
would have carried an interest rate of 10%. Interest of $48,000 has been paid on the loan and charged
as a finance cost.

39

LSB_F7_Rev Kit:297mm x 210mm

28/10/09

13:36

Page 40

FINANCIAL REPORTING (INTERNATIONAL)


The present value of $1 receivable at the end of each year, based on discount rates of 8% and 10% are:

(vi)
(vii)

End of year 1
2
3
4

8%
093
086
079
073

10%
091
083
075
068

The carrying amounts of Wellmays net assets at 31 March 2009 are $600,000 higher than their tax base.
The rate of taxation is 35%. The income tax charge of $360,000 does not include the adjustment
required to the deferred tax provision which should be charged in full to the income statement.
Bonus/scrip issue:

On 15 March 2009,Wellmay made a bonus issue from retained earnings of one share for every four
held. The issue has not been recorded in the draft financial statements.

Required

Redraft the financial statements of Wellmay, including a statement of other comprehensive income and a
statement of changes in equity, for the year ended 31 March 2009 reflecting the adjustments required by
notes (i) to (vii) above.
Note: Calculations should be made to the nearest $000.

(25 marks)

16. Peterlee
(a)

(b)

The IASBs Framework for the preparation and presentation of financial statements (Framework) sets
out the concepts that underlie the preparation and presentation of financial statements that external
users are likely to rely on when making economic decisions about an entity.
Explain the purpose and authoritative status of the Framework. (5 marks)

Of particular importance within the Framework are the definitions and recognition criteria for assets
and liabilities.

Define assets and liabilities and explain the important aspects of their definitions. Explain why these
definitions are of particular importance to the preparation of an entitys statement of financial position
and income statement. (8 marks)

(13 marks)

40

LSB_F7_Rev Kit:297mm x 210mm

28/10/09

13:36

Page 41

QUESTIONS

17. Derringdo
(a)

Revenue recognition is the process by which companies decide when and how much income should be
included in the income statement. It is a topical area of great debate in the accounting profession. The
IASB looks at revenue recognition from conceptual and substance points of view. There are occasions
where a more traditional approach to revenue recognition does not entirely conform to the IASB
guidance; indeed neither do some International Accounting Standards.

Required

(b)

Explain the implications that the IASBs Framework for the Preparation and Presentation of Financial
Statements (Framework) and the application of substance over form have on the recognition of income.
Give examples of how this may conflict with traditional practice and some accounting standards.
(6 marks)

Derringdo sells goods supplied by Gungho. The goods are classed as A grade (perfect quality) or B
grade, having slight faults. Derringdo sells the A grade goods acting as an agent for Gungho at a fixed
price calculated to yield a gross profit margin of 50%. Derringdo receives a commission of 125% of the
sales it achieves for these goods. The arrangement for B grade goods is that they are sold by Gungho to
Derringdo and Derringdo sells them at a gross profit margin of 25%. The following information has
been obtained from Derringdos financial records:
Inventory held on premises 1 April 2008
Goods from Gungho year to 31 March 2009
Inventory held on premises 31 March 2009

Required

(c)

A grade
B grade
A grade
B grade
A grade
B grade

$'000
2,400
1,000
18,000
8,800
2,000
1,250

Prepare the income statement extracts for Derringdo for the year to 31 March 2009 reflecting the
above information. (5 marks)

Derringdo acquired an item of plant at a gross cost of $800,000 on 1 October 2008. The plant has an
estimated life of 10 years with a residual value equal to 15% of its gross cost. Derringdo uses
straight-line depreciation on a time apportioned basis. The company received a government grant of
30% of its cost price at the time of its purchase. The terms of the grant are that if the company retains
the asset for four years or more, then no repayment liability will be incurred. If the plant is sold within
four years a repayment on a sliding scale would be applicable. The repayment is 75% if sold within the
first year of purchase and this amount decreases by 25% per annum. Derringdo has no intention to sell
the plant within the first four years. Derringdos accounting policy for capital based government grants
is to treat them as deferred credits and release them to income over the life of the asset to which they
relate.

Required

(i)

(ii)

Discuss whether the companys policy for the treatment of government grants meets the definition of a
liability in the IASBs Framework; and (3 marks)

Prepare extracts of Derringdos financial statements for the year to 31 March 2009 in respect of the
plant and the related grant:
applying the companys policy;

(d)

in compliance with the definition of a liability in the Framework. Your answer should consider
whether the sliding scale repayment should be used in determining the deferred credit for the grant.
(6 marks)

Derringdo sells carpets from several retail outlets. In previous years the company has undertaken
responsibility for fitting the carpets in customers premises. Customers pay for the carpets at the time
they are ordered. The average length of time from a customer ordering a carpet to its fitting is 14 days.

41

LSB_F7_Rev Kit:297mm x 210mm

28/10/09

13:36

Page 42

FINANCIAL REPORTING (INTERNATIONAL)


In previous years, Derringdo had not recognised a sale in income until the carpet had been successfully
fitted as the rectification costs of any fitting error would be expensive. From 1 April 2008 Derringdo
changed its method of trading by sub-contracting the fitting to approved contractors. Under this policy
the sub-contractors are paid by Derringdo and they (the subcontractors) are liable for any errors made
in the fitting. Because of this Derringdo is proposing to recognise sales when customers order and pay
for the goods, rather than when they have been fitted. Details of the relevant sales figures are:
Sales made in retail outlets for the year to 31 March 2009
Sales value of carpets fitted in the 14 days to 14 April 2008
Sales value of carpets fitted in the 14 days to 14 April 2009

$'000
23,000
1,200
1,600

Note: the sales value of carpets fitted in the 14 days to 14 April 2008 are not included in the annual
sales figure of $23 million, but those for the 14 days to 14 April 2009 are included.

Required

Discuss whether the above represents a change of accounting policy, and, based on your discussion,
calculate the amount that you would include in sales revenue for carpets in the year to 31 March 2009.
(5 marks)

(25 marks)

18. Broadoak
The broad principles of accounting for property, plant and equipment involve distinguishing between capital and
revenue expenditure, measuring the cost of assets, determining how they should be depreciated and dealing
with the problems of subsequent measurement and subsequent expenditure. IAS 16 Property, plant and
equipment has the intention of improving consistency in these areas.
Required
(a)

(b)
(c)

Explain:

(i) How the initial cost of property, plant and equipment should be measured (4 marks)

(ii) The circumstances in which subsequent expenditure on those assets should be capitalised. (3 marks)

Explain IAS 16's requirements regarding the revaluation of non-current assets and the accounting
treatment of surpluses and deficits on revaluation and gains and losses on disposal. (8 marks)
(i) Broadoak has recently purchased an item of plant from Plantco, the details of this are:

Basic list price of plant


Trade discount applicable to Broadoak
Ancillary costs:
Shipping and handling costs
Estimated pre-production testing
Maintenance contract for three years
Site preparation costs:
electrical cable installation
concrete reinforcement
own labour costs

$
240,000
12.5% on list price
2,750
12,500
24,000

14,000
4,500
7,500

26,000

Broadoak paid for the plant (excluding the ancillary costs) within four weeks of order, thereby obtaining
an early settlement discount of 3%.
42

LSB_F7_Rev Kit:297mm x 210mm

28/10/09

13:37

Page 43

QUESTIONS
Broadoak had incorrectly specified the power loading of the original electrical cable to be installed by
the contractor. The cost of correcting this error of $6,000 is included in the above figure of $14,000.

The plant is expected to last for 10 years. At the end of this period there will be compulsory costs of
$15,000 to dismantle the plant and $3,000 to restore the site to its original use condition.

Required

Calculate the amount at which the plant will be measured at recognition. (Ignore discounting.)
(5 marks)

(ii) Broadoak acquired a 12 year lease on a property on 1 October 2007 at a cost of $240,000. The
company policy is to revalue its properties to their market values at the end of each year.
Accumulated amortisation is eliminated and the property is restated to the revalued amount. Annual
amortisation is calculated on the carrying values at the beginning of the year. The market values of
the property on 30 September 2008 and 2009 were $231,000 and $175,000 respectively. The
existing balance on the revaluation surplus at 1 October 2007 was $50,000. This related to some
non-depreciable land whose value had not changed significantly since 1 October 2007.

Required

Prepare extracts of the financial statements of Broadoak for the years to 30 September 2008 and 2009
in respect of the leasehold property. (5 marks)

(25 marks)

19. Wilderness
(a)

IAS 36 Impairment of assets was issued in June 1998 and subsequently amended in March 2004. Its main
objective is to prescribe the procedures that should ensure that an entitys assets are included in its
balance sheet at no more than their recoverable amounts. Where an asset is carried at an amount in
excess of its recoverable amount, it is said to be impaired and IAS 36 requires an impairment loss to be
recognised.

Required

(i) Define an impairment loss explaining the relevance of fair value less costs to sell and value in use; and
state how frequently assets should be tested for impairment; (6 marks)
Note: your answer should NOT describe the possible indicators of an impairment.

(b)

(ii) Explain how an impairment loss is accounted for after it has been calculated. (5 marks)

The assistant financial controller of the Wilderness group, a public listed company, has identified the
matters below which she believes may indicate an impairment to one or more assets:

(i) Wilderness owns and operates an item of plant that cost $640,000 and had accumulated
depreciation of $400,000 at 1 October 2008. It is being depreciated at 12% on cost. On 1 April
2009 (exactly half way through the year) the plant was damaged when a factory vehicle collided into
it. Due to the unavailability of replacement parts, it is not possible to repair the plant, but it still
operates, albeit at a reduced capacity. Also it is expected that as a result of the damage the remaining
life of the plant from the date of the damage will be only two years. Based on its reduced capacity,
the estimated present value of the plant in use if $150,000. The plant has a current disposal value of
$20,000 (which will be nil in two years time), but Wilderness has been offered a tradein value of
$180,000 against a replacement machine which has a cost of $1 million (there would be no disposal
costs for the replaced plant). Wilderness is reluctant to replace the plant as it is worried about the
longterm demand for the produce produced by the plant. The tradein value is only available if the
plant is replaced.

Required

Prepare extracts from the statement of financial position and income statement of Wilderness in
respect of the plant for the year ended 30 September 2009. Your answer should explain how you
arrived at your figures. (7 marks)

43

LSB_F7_Rev Kit:297mm x 210mm

28/10/09

13:37

Page 44

FINANCIAL REPORTING (INTERNATIONAL)


(ii) On 1 April 2008 Wilderness acquired 100% of the share capital of Mossel, whose only activity is the
extraction and sale of spa water. Mossel had been profitable since its acquisition, but bad publicity
resulting from several consumers becoming ill due to a contamination of the spa water supply in
April 2009 has led to unexpected losses in the last six months. The carrying amounts of Mossels
assets at 30 September 2009 are:
Brand (Quencher see below)
Land containing spa
Purifying and bottling plant
Inventories

$000
7,000
12,000
8,000
5,000
32,000

The source of the contamination was found and it has now ceased.

The company originally sold the bottled water under the brand name of Quencher, but because of
the contamination it has rebranded its bottled water as Phoenix. After a large advertising campaign,
sales are now starting to recover and are approaching previous levels. The value of the brand in the
balance sheet is the depreciated amount of the original brand name of Quencher.
The directors have acknowledged that $1.5 million will have to be spent in the first three months of
the next accounting period to upgrade the purifying and bottling plant.

Inventories contain some old Quencher bottled water at a cost of $2 million; the remaining
inventories are labeled with the new brand Phoenix. Samples of all the bottled water have been
tested by the health authority and have been passed as fit to sell. The old bottled water will have to
be relabelled at a cost of $250,000, but is then expected to be sold at the normal selling price of
(normal) cost plus 50%.
Based on the estimated future cash flows, the directors have estimated that the value in use of
Mossel at 30 September 2009, calculated according to the guidance in IAS 36, is $20 million. is no
reliable estimate of the fair value less costs to sell of Mossel.

Required

Calculate the amounts at which the assets of Mossel should appear in the consolidated statement of
financial position of Wilderness at 30 September 2009. Your answer should explain how you arrive at
your figures. (7 marks)

(25 marks)

20. Elite Leisure


Elite Leisure is a private limited liability company that operates a single cruise ship. The ship was acquired on
1 October 2000. Details of the cost of the ships components and their estimated useful lives are:
Component

Ships fabric (hull, decks etc)


Cabins and entertainment area fittings
Propulsion system

Original cost
($ million)
300
150
100

Deprecation basis

25 years straightline
12 years straightline
Useful life of 40,000 hours

At 30 September 2008 no further capital expenditure had been incurred on the ship.

In the year ended 30 September 2008 the ship had experienced a high level of engine trouble which had cost
the company considerable lost revenue and compensation costs. The measured expired life of the propulsion
system at 30 September 2008 was 30,000 hours. Due to the unreliability of the engines, a decision was taken
in early October 2008 to replace the whole of the propulsion system at a cost of $140 million. The expected
life of the new propulsion system was 50,000 hours and in the year ended 30 September 2009 the ship had

44

LSB_F7_Rev Kit:297mm x 210mm

28/10/09

13:37

Page 45

QUESTIONS
used its engines for 5,000 hours.

At the same time as the propulsion system replacement, the company took the opportunity to do a limited
upgrade to the cabin and entertainment facilities at a cost of $60 million and repaint the ships fabric at a cost
of $20 million. After the upgrade of the cabin and entertainment area fittings it was estimated that their
remaining life was five years (from the date of the upgrade). For the purpose of calculating depreciation, all the
work on the ship can be assumed to have been completed on 1 October 2008. All residual values can be taken
as nil.
Required

Calculate the carrying amount of Elite Leisures cruise ship at 30 September 2009 and its related expenditure in
the income statement for the year ended 30 September 2009. Your answer should explain the treatment of
each item. (12 marks)

21. Linnet
(a)

Linnet is a large public listed company involved in the construction industry. Revenue on construction
contracts is normally recognised by reference to the stage of completion of the contract. However, in
certain circumstances, revenue is only recognised to the extent that it does not exceed recoverable
contract costs.

Required

(b)

Discuss the principles that underlie each of the two methods and describe the circumstances in which
their use is appropriate. (5 marks)

is part way through a contract to build a new football stadium at a contracted price of $300 million.
Details of the progress of this contract at 1 April 2008 are shown below:
Cumulative sales revenue recognised
Cumulative cost of sales to date
Profit to date

$ million
150
112
38

The following information has been extracted from the accounting records at 31 March 2009:

$ million
Total progress payment received for work certified at 29 February 2009
180
Total costs incurred to date (excluding rectification costs below)
195
Rectification costs
17

Linnet has received progress payments of 90% of the work certified at 29 February 2009. Linnets
surveyor has estimated the sales value of the further work completed during March 2009 was
$20 million.
At 31 March 2009 the estimated remaining costs to complete the contract were $45 million.

The rectification costs are the costs incurred in widening access roads to the stadium. This was the
result of an error by Linnets architect when he made his initial drawings.

Linnet calculates the percentage of completion of its contracts as the proportion of sales value earned
to date compared to the contract price.
All estimates can be taken as being reliable.

Required

Prepare extracts of the financial statements for Linnet for the above contract for the year to 31 March
2009. (8 marks)

(13 marks)
45

LSB_F7_Rev Kit:297mm x 210mm

28/10/09

13:37

Page 46

FINANCIAL REPORTING (INTERNATIONAL)

22. Torrent
Torrent is a large publicly listed company whose main activity involves construction contracts. Details of three
of its contracts for the year ended 31 March 2009 are:
Contract
Date commenced
Estimated duration

Alfa
1 April 2007
3 years
$m
20
15

Beta
1 October 2008
18 months
$m
6
7.5

Ceta
1 October 2008
2 years
$m
12
10

5
12.5
3.5
5.4
12.6

Nil
2
5.5
Nil
1.8

Nil
4
6
Nil
nil

Fixed contract price


Estimated costs at start of contract
Cost to date:
At 31 March 2008
At 31 March 2009
Estimated costs at 31 March 2009 to complete
Progress payments received at 31March 2008
Progress payments received at 31 March 2009

Notes
(i)

(ii)
(iii)

The companys normal policy for determining the percentage completion of contracts is based on the
value of work invoiced to date compared to the contract price. Progress payments received represent
90% of the work invoiced. However, no progress payments will be invoiced or received from contract
Ceta until it is completed, so the percentage completion of this contract is to be based on the cost to
date compared to the estimated total contract costs.
The cost to date of $125 million at 31 March 2009 for contract Alfa includes $1 million relating to
unplanned rectification costs incurred during the current year (ended 31 March 2009) due to
subsidence occurring on site.

Since negotiating the price of contract Beta,Torrent has discovered the land that it purchased for the
project is contaminated by toxic pollutants. The estimated cost at the start of the contract and the
estimated costs to complete the contract include the unexpected costs of decontaminating the site
before construction could commence.

Required

Prepare extracts of the income statement and statement of financial position for Torrent in respect of
the above construction contracts for the year ended 31 March 2009.
(12 marks)

23. Bowtock
Bowtock has leased an item of plant under the following terms:
Commencement of the lease was 1 January 2008.

Term of lease 5 years.

Annual payments in advance $12,000.

Cash price and fair value of the asset $52,000 at 1 January 2008 equivalent to the present value of the
minimum lease payments.

Implicit interest rate within the lease (as supplied by the lessor) 8% per annum (to be apportioned on a time
basis where relevant).
The companys depreciation policy for this type of plant is 20% per annum on cost (apportioned on a time
basis where relevant).

46

LSB_F7_Rev Kit:297mm x 210mm

28/10/09

13:37

Page 47

QUESTIONS
Required

Prepare extracts of the income statement and statement of financial position for Bowtock for the year to 30
September 2009 for the above lease.
(5 marks)

24. Triangle
Triangle, a public listed company, is in the process of preparing its draft financial statements for the year to
31 March 2009. The following matters have been brought to your attention:
(i)

(ii)

(iii)
(iv)

On 1 April 2008 the company brought into use a new processing plant that had cost $15 million to
construct and had an estimated life of ten years. The plant uses hazardous chemicals which are put in
containers and shipped abroad for safe disposal after processing. The chemicals have also contaminated
the plant itself which occurred as soon as the plant was used. It is a legal requirement that the plant is
decontaminated at the end of its life. The estimated present value of this decontamination, using a
discount rate of 8% per annum, is $5 million. The financial statements have been charged with $15
million ($15 million/10 years) for plant depreciation and a provision of $500,000 ($5 million/10 years)
has been made towards the cost of the decontamination. (8 marks)

On 15 May 2009 the companys auditors discovered a fraud in the material requisitions department. A
senior member of staff who took up employment with Triangle in August 2008 had been authorising
payments for goods that had never been received. The payments were made to a fictitious company
that cannot be traced. The member of staff was immediately dismissed. Calculations show that the total
amount of the fraud to the date of its discovery was $240,000 of which $210,000 related to the year to
31 March 2009. (Assume the fraud is material). (5 marks)
The company has contacted its insurers in respect of the above fraud. Triangle is insured for theft, but
the insurance company maintains that this is a commercial fraud and is not covered by the theft clause
in the insurance policy. Triangle has not yet had an opinion from its lawyers. (4 marks)

On 1 April 2008 Triangle sold maturing inventory that had a carrying value of $3 million (at cost) to
Factorall, a finance house, for $5 million. Its estimated market value at this date was in excess of $5
million. The inventory will not be ready for sale until 31 March 2012 and will remain on Triangles
premises until this date. The sale contract includes a clause allowing Triangle to repurchase the
inventory at any time up to 31 March 2012 at a price of $5 million plus interest at 10% per annum
compounded from 1 April 2008. The inventory will incur storage costs until maturity. The cost of
storage for the current year of $300,000 has been included in trade receivables (in the name of
Factorall). If Triangle chooses not to repurchase the inventory, Factorall will pay the accumulated s
storage costs on 31 March 2012. The proceeds of the sale have been debited to the bank and the sale
has been included in Triangles sales revenue. (8 marks)

Required

Explain how the items in (i) to (iv) above should be treated in Triangles financial statements for the year
to 31 March 2009 in accordance with current international accounting standards. Your answer should
quantify the amounts where possible.
The mark allocation is shown against each of the four matters above.

(25 marks)

25. Atkins
The principle of recording the substance or economic reality of transactions rather than their legal form lies at
the heart of the Framework for Preparation and Presentation of Financial Statements and several International
Financial Reporting Standards. The development of this principle was partly in reaction to a minority of public
interest companies entering into certain complex transactions. These transactions sometimes led to
accusations that company directors were involved in 'creative accounting'.
(i)

Atkins's operations involve selling cars to the public through a chain of retail car showrooms. It buys
most of its new vehicles directly from the manufacturer on the following terms:

Atkins will pay the manufacturer for the cars on the date they are sold to a customer or six months

47

LSB_F7_Rev Kit:297mm x 210mm

28/10/09

13:37

Page 48

FINANCIAL REPORTING (INTERNATIONAL)


after they are delivered to its showrooms whichever is the sooner.

The price paid will be 80% of the retail list price as set by the manufacturer at the date that the
goods are delivered.

Atkins will pay the manufacturer 1.5% per month (of the cost price to Atkins) as a 'display charge'
until the goods are paid for.

Atkins may return the cars to the manufacturer any time up until the date the cars are due to be
paid for. Atkins will incur the freight cost of any such returns. Atkins has never taken advantage of
this right of return.

The manufacturer can recall the cars or request them to be transferred to another retailer any time
up until the time they are paid for by Atkins.

Required

Discuss which party bears the risks and rewards in the above arrangement and come to a conclusion on
how the transactions should be treated by each party. (6 marks)

(ii)

Atkins bought five identical plots of development land for $2 million in 2006. On 1 October 2008 Atkins
sold three of the plots of land to an investment company, Landbank, for a total of $2.4 million.This price
was based on 75% of the fair market value of $3.2 million as determined by an independent surveyor at
the date of sale. The terms of the sale contained two clauses:

Atkins can re-purchase the plots of land for the full fair value of $3.2 million (the value determined at
the date of sale) any time until 30 September 2011; and
On 1 October 2011, Landbank has the option to require Atkins to re-purchase the properties for
$3.2 million. You may assume that Landbank seeks a return on its investments of 10% per annum.

Required

Discuss the substance of the above transactions; and (3 marks)

Prepare extracts of the income statement and statement of financial position (ignore cash) of Atkins for
the year to 30 September 2009:
If the plots of land are considered as sold to Landbank; and (2 marks)
Reflecting the substance of the above transactions. (4 marks)

(15 marks)

26. CB
On 1 February 2008, CB, a listed entity, had 3 million ordinary shares in issue.

On 1 March 2008, CB made a rights issue of 1 for 4 at $6.50 per share. The issue was completely taken up by
shareholders.
Extracts from CBs financial statements for the year ended 31 January 2009 are presented below:
CB: Extracts from income statement for the year ended 31 January 2009
Operating profit
Finance cost
Profit before tax
Income tax expense
Profit for the period

48

$000
1,380
(400)
980
(255)
725

LSB_F7_Rev Kit:297mm x 210mm

28/10/09

13:37

Page 49

QUESTIONS
CB: Extracts from summarised statement of changes in equity for the year ended
31 January 2009
Balance at 1 February 2008
Issue of share capital
Total comprehensive income
Equity dividends
Balance at 31 January 2009

$000
7,860
4,875
1,625
(300)
14,060

Just before the rights issue, CBs share price was $7.50, rising to $8.25 immediately afterwards. The share price
at close of business on 31 January 2009 was $6.25.
At the beginning of February 2009, the average price earnings ratio in CBs business sector was 28.4, and the
P/E of its principal competitor was 42.5.
Required:

(a) Calculate the earnings per share for CB for the year ended 31 January 2009, and its PE ratio at that date.
(6 marks)

(b) Discuss the significance of PE ratios to investors and CBs PE ratio relative to those of its competitor and
business sector (4 marks)

(10 marks)

27. QRS
The directors of QRS, a listed entity, have met to discuss the business medium to long term financing
requirements. Several possibilities were discussed, including the issue of more shares using a rights issue. In
many respects, this would be the most desirable option because the entity is already quite highly geared.
However, the directors are aware of several recent cases where rights issues have not been successful because
share prices are currently quite low and many investors are averse to any kind of investment in shares.

Therefore, the directors have turned their attention to other options. The finance director is on sick leave, and
so you, her assistant, have been given the task of responding to the following note from the Chief Executive:

Now that weve had a chance to discuss possible financing arrangements, the directors are in agreement that we should
structure our issue of financial instruments in order to be able to classify them as equity rather than debt. Any increase
in the gearing ratio would be unacceptable. Therefore, we have provisionally decided to make two issues of financial
instruments as follows:

1. An issue of non-redeemable preference shares to raise $4million. These shares will carry a fixed interest rate of 6%
and because they are shares they can be classified as equity.

2. An issue of 6% convertible bonds, issued at par value, to raise $6million. These bonds will carry a fixed date for
conversion in four years time. Each $100 of debt will be convertible at the holders option into 130 $1 shares. In
our opinion, these bonds can actually be classified as equity immediately, because they are
convertible within five years on terms that are favourable to the holder.

Please confirm that these instruments will not increase our gearing ratio should they be issued.

Note: You determine that the market rate available for similar non-convertible bonds is currently 8%.

Required

Explain to the directors the accounting treatment in respect of debt/equity classification required by IAS 32
Financial Instruments: Disclosure and Presentation for each of the proposed issues, advising them on the
acceptability of classifying the instruments as equity.
Your explanation should be accompanied by calculations where appropriate.

(10 marks)
49

LSB_F7_Rev Kit:297mm x 210mm

28/10/09

13:37

Page 50

FINANCIAL REPORTING (INTERNATIONAL)

28. Harper
You are a partner in a small audit and accounting practice. You have just completed the audit and finalised the
financial statements of a small family owned company in discussion with its managing director Mrs Harper.
After the meeting Mrs Harper has asked for your help. She has obtained the published financial statements of
several quoted companies in which she is considering buying some shares as a personal investment. She
presents you with the following information:
(a)

In the year to 30 September 2009, two companies, Gamma and Toga, reported identical profits before
tax of $100 million. Information in the Chairmens reports said both companies also expected profits
from their core activities (to be interpreted as from continuing operations) to grow by 10% in the
following year. Mrs Harper has extracted information from the income statements and made the
following summary:
Operating profit:
Continuing activities
Acquisitions
Discontinued activities

Gamma
$ million
70
nil
30
100

Toga
$ million
90
50
(40)
100

A note to the financial statements of Toga said that both the discontinuation and acquisition occurred
on 1 April 2009 and were part of an overall plan to focus on its traditional core activities after incurring
large losses on a new foreign venture.

Required

(i) Briefly explain to Mrs Harper why information on discontinued operations is useful; (3 marks)

(ii) Calculate the expected operating profit for both companies for the year to 30 September 2010
(assuming the Chairmens growth forecasts are correct):

(b)

in the absence of information of the discontinued operations; and


based on the information provided above. (4 marks)

Taylor is another company about which Mrs Harper has obtained the following information from its
published financial statements:
Earnings per share:
Year to 30 September
Basic earnings per share

2009
25 cents

2008
20 cents

Statement of financial position extracts:


8% Convertible loan stock

$ million
200

$ million
200

The earnings per share is based on attributable earnings of $50 million ($30 million in 2008) and 200
million ordinary shares in issue throughout the year (150 million weighted average number of ordinary
shares in 2008).

The loan stock is convertible to ordinary shares in 2011 on the basis of 70 new shares for each $100 of
loan stock.
Note to the financial statements:

There are directors share options (in issue since 2006) that allow Taylors directors to subscribe for a
total of 50 million new ordinary shares at a price of $1.50 each.
(Assume the current rate of income tax for Taylor is 25% and the market price of its ordinary shares
throughout the year has been $2.50).

50

LSB_F7_Rev Kit:297mm x 210mm

28/10/09

13:37

Page 51

QUESTIONS
Mrs Harper has read that the trend of the earnings per share is a reliable measure of a companys profit
trend. She cannot understand why the increase in profits is 67% ($30 million to $50 million), but the
increase in the earnings per share is only 25% (20 cents to 25 cents). She is also confused by the
company also quoting a diluted earnings per share figure, which is lower than the basic earnings per share.

Required
(i)

(ii)

(c)

(iii)

Explain why the trend of earnings per share may be different from the trend of the reported
profit, and which is the more useful measure of performance; (3 marks)
Calculate the diluted earnings per share for Taylor based on the effect of the convertible
loan stock and the directors share options for the year to 30 September 2009 (ignore
comparatives); and (5 marks)
Explain the relevance of the diluted earnings per share measure. (4 marks)

Mrs Harper has noticed that the tax charge for a company called Stepper is $5 million on profits before
tax of $35 million. This is an effective rate of tax of 14.3%. Another company Jenni has an income tax
charge of $10 million on profit before tax of $30 million. This is an effective rate of tax of 33.3% yet
both companies state the rate of income tax applicable to them is 25%. Mrs Harper has also noticed
that in the statements of cash flow each company has paid the same amount of tax of $8 million.

Required

Advise Mrs Harper of the possible reasons why the income tax charge in the financial statements as a
percentage of the profit before tax may not be the same as the applicable income tax rate, and why the
tax paid in the statement of cash flow may not be the same as the tax charge in the income statement.
(6 marks)

(25 marks)

29. FW
FW is a listed entity involved in the business of oil exploration, drilling and refining in three neighbouring
countries Aye, Bee and Cee. The business has been consistently profitable, creating high returns for its
international shareholders. In recent years, however, there has been an increase in environmental lobbying in
FWs three countries of operation. Two years ago, and environmental group based in Cee started lobbying the
government to take action against FW for alleged destruction of valuable wildlife habitats in Cees protected
wetlands and the displacement of the local population. At the time, the directors of FW took legal advice, on
the basis of which they assessed the risk of liability at less than 50%. A contingent liability of $500million was
noted in the financial statements to cover possible legal costs, compensation to displaced persons and
reinstatement of the habitats, as well as fines.

FW is currently preparing its financial statements for the year ended 28 February 2009. Recent advice from
the entitys legal advisers has assessed that the risk of a successful action against FW has increased, and must
now be regarded as more likely than not to occur. The board of directors has met to discuss the issue. The
directors accept that a provision of $500million is required but would like to be informed of the effects of the
adjustment on certain key ratios that the entity headlines in its annual report. All of the directors are
concerned about the potentially adverse effect on the share price, as FW is actively engaged in a takeover bid
that would involve a substantial share exchange. In addition they feel that the publics image of the entity is
likely to be damaged.

The draft financial statements for the year ended 28 February 2009 include the following information relevant
for the calculation of key ratios. All figures are before taking into account the $500million provision. The
provision will be charged to operating expenses.

51

LSB_F7_Rev Kit:297mm x 210mm

28/10/09

13:37

Page 52

FINANCIAL REPORTING (INTERNATIONAL)


Net assets (before long term loans) at 1 March 2008
Net assets (before long term loans) at 28 February 2009-08-18
Long term loans at 28 February 2009-08-18
Share capital and reserves at 1 March 2008
Share capital and reserves at 28 February 2009-08-18
Revenue
Operating profit
Profit before tax
Profit for the period

$m
9,016
10,066
4,410
4,954
5,656
20,392
2,080
1,670
1,002

The number of ordinary shares in issue throughout the years ended 29 February 2008 and 2009 was 6,000
million shares of 25c each.

FWs key financial ratios for the 2008 financial year (calculated using the financial statements for the year ended
28 February 2008) were
-

Return on equity (using average equity)


Return on net assets (using average net assets)
Gearing (debt as percentage of equity)
Operating profit margin
Earnings per share

Required

24.7%
17.7%
82%
10.1%
12.2c

Prepare a briefing paper that analyses and interprets the effects of making the environmental provision on FWs
key financial ratios.You should take into account the possible effects on the public perception of FW

(12 marks)

30. LMN
LMN trades in motor vehicles, which are manufactured and supplied by their manufacturer, IJK. Trading
between the two entities is subject to a contractual agreements, the principal terms of which are as follows:

LMN is entitled to hold on its premises at any one time up to 80 vehicles supplied by IJK. LMN is free to
specify the ranges and models of vehicle supplied to it. IJK retains legal title to the vehicles until such time
as they are sold to a third party by LMN.
While the vehicles remain on its premises, LMN is required to insure them against loss or damage.

The price at which vehicles are supplied is determined at the time of delivery; it is not subject to any
subsequent alteration.

When LMN sells a vehicle to a third party, it is required to inform IJK within three working days. IJK
submits an invoice to LMN at the originally agreed price; the invoice is payable by LMN within 30 days.

LMN is entitled to use any of the vehicles supplied to it for demonstration purposes and road testing.
However, if more than a specified number of kilometres are driven in a vehicle, LMN is required to pay IJK a
rental charge.

LMN has the right to return any vehicle to IJK at any time without incurring a penalty, except for any rental
charge incurred in respect of excess kilometres driven.

Required

Discuss the economic substance of the contractual arrangement between the two entities in respect of the
recognition of inventory and of sales. Refer where appropriate to IAS 18 Revenue.

(10 marks)

52

LSB_F7_Rev Kit:297mm x 210mm

28/10/09

13:37

Page 53

QUESTIONS

31. EFG
You are an accountant at EFG, an entity that has recently embarked upon an aggressive programme of
acquisitions in order to grow its market share as rapidly as possible. EFG has targeted J, a well-established
entity operating in the same sector, but with a significant level of export sales.

In order to be able to respond to opportunities quickly, EFG has extablished a basic set of four key financial
ratios to assess the performance and position of target businesses. If a businesss ratios fall within the set
criteria, more detailed analysis will follow prior to the launch of a formal bid.
The four key ratios and the criteria are:

Gross profit margin


Operating profit margin
Return on total capital employed
Gearing (long-term liabilities/shareholders funds)

should exceed 25%


should exceed 13%
should exceed 25%
should not exceed 25%

Js most recent financial statements are as follows:

J Income statement for the year ended 31 January 2009


$000
1,810
(1,381)
429
(236)
193
(9)
184
(50)
134

Revenue
Cost of sales
Gross profit
Operating expenses
Profit from operations
Finance cost
Profit before tax
Income tax expense
Net profit for the year

J Statement of changes in equity for the year ended 31 January 2009


Share capital
$000

Balance at 1 Feb 2008


Transfer
Total comprehensive income
Dividends
Balance at 31 Jan 2009

350

350

J Statement of financial position at 31 Jan 2009


Non-current assets
Property, plant and equipment
Current assets
Inventories
Trade receivables
Cash

Revaluation
reserve
$000
210
(10)
200
$000

Retained
earnings
$000
96
10
134
(21)
219

Total
$000
656
134
(21)
769

$000
707

201
247
18

466
1,173
Continued on next page

53

LSB_F7_Rev Kit:297mm x 210mm

28/10/09

13:37

Page 54

FINANCIAL REPORTING (INTERNATIONAL)


$000

Equity and liabilities


Share capital
Revaluation reserve
Retained earnings

350
200
219

Non-current liabilities
Long term borrowings
Current liabilities
Trade payables
Income tax

$000

769
248

142
14

156
1,173

Js directors who each hold a significant percentage of the ordinary share capital in the entity are interested in
EFGs potential bid, and they have co-operated fully in providing information. On a recent visit to the entity,
EFGs finance director has ascertained that , in many respects, the financial and operating policies of the two
businesses are very similar.
However there are some differences, summarised as follows:

J has a policy of revaluation of property, but EFGs key ratios are set on the assumption of valuation at
depreciated historic cost. J owns one property, a warehouse building that was revalued five years ago. At
that time, the revaluation surplus was $250,000, and the estimated useful life of the property was 25 years
assuming a residual value of nil. J depreciated property on the straight line basis.

J employs a highly skilled team of sales representatives who are paid a substantial profit-related bonus at the
end of each year. for the year ended 31 January 2009, the total bonus paid was $96,000 included in operating
expenses. EFGs operating policy does not include the payment of bonuses to staff; the directors prefer to
reward staff by a fixed salary. The financial controller estimates that EFGs operating policy would involve
payment of an additional $50,000 of fixed salaries instead of the bonus.
The issued share capital of J includes $50,000 of 4% preferred stock. The directors of EFG believe this
should be classified as a long-term liability.

J values inventories using an average cost basis, whereas EFGs valuation policy is first in first out (FIFO). Js
accountants have estimated that the valuation of their opening and closing inventories on a FIFO basis
would be:
At 1 February 2008
At 31 January 2009-08-16

$208,000
$218,000

Js opening inventories at average cost were $197,000.

Required

(a) Calculate the four key financial ratios for J before making any adjustments in respect of changes required by
EFGs financial and operating policies. (2 marks)

(b) Calculate the four key financial ratios for J after making adjustments in respect of changes required by EFGs
financial and operating policies (For this purpose, assume that the alteration in respect of the remuneration
of sales representatives would take effect from 1 February 2008). Using EFGs criteria, advise the directors
on whether or not they should pursue the potential acquisition of J. (16 marks)

(18 marks)

54

LSB_F7_Rev Kit:297mm x 210mm

28/10/09

13:37

Page 55

QUESTIONS

32. Breadline
You are the assistant financial controller of Judicious. One of your company's credit controllers has asked you
to consider the account balance of one of your customers, Breadline. He is concerned at the pattern of
payments and increasing size and age of the account balance. As part of company policy he has obtained the
most recently filed financial statements of Breadline and these are summarised below. A note to the financial
statements of Breadline states that it is a wholly owned subsidiary of Wheatmaster, and its main activities are
the production and distribution of bakery products to wholesalers. By coincidence your company's Chief
Executive has been made aware that Breadline may be available for sale. She has asked for your opinion on
whether Breadline would make a suitable addition to the group's portfolio.
BREADLINE
INCOME STATEMENT YEAR TO:
Sales revenue
Cost of sales
Gross profit
Operating expenses
Finance costs: loan note
overdraft
Profit before tax
Taxation
Profit for the year

STATEMENTS OF FINANCIAL POSITION AS AT:


Non-current assets
Freehold premises at valuation
Leasehold premises
Plant
Current assets
Inventory
Accounts receivable
Bank
Total assets
Equity and liabilities
Equity
Ordinary shares of $1 each
Reserves
Share premium
Revaluation surplus
(re freehold premises)
Retained earnings

31 December 2009
$'000
$'000
8,500
(5,950)
2,550
(560)
10
10
(20)
1,970
(470)
1,500

31 December 2008
$'000
$'000
6,500
(4,810)
1,690
(660)
nil
5
(5)
1,025
(175)
850

31 December 2009
$'000
$'000

31 December 2008
$'000
$'000

nil
2,500
1,620
4,120
370
960
nil

1,330
5,450

1,250
nil
750
2,000
240
600
250

500

100

200

nil

nil
3,000

700
1,700

3,200
3,700

1,090
3,090

2,400
2,500
Continued on next page

55

LSB_F7_Rev Kit:297mm x 210mm

28/10/09

13:37

Page 56

FINANCIAL REPORTING (INTERNATIONAL)

Non-current liabilities
% loan note
Current liabilities
Accounts payable
Overdraft
Total equity and liabilities

31 December 2009
$'000
$'000

31 December 2008
$'000
$'000

500
1,030
220

1,250
5,450

nil
590
nil

590
3,090

From your company's own records you have ascertained that sales to Breadline for the year to 2009 and 2008
were $1,200,000 and $800,000 respectively and the year-end account balances were $340,000 and $100,000
respectively. Normal credit terms, which should apply to Breadline, are that payment is due 30 days after the
end of the month of sale. You are also aware that the company has not changed its address and is trading from
the same premises. A note to Breadline's financial statements says that the profit on the disposal of its freehold
premises has been included in cost of sales as this is where the depreciation on the freehold was charged.
Dividends of $900,000 were paid in 2009. No dividends were paid in 2008.
Note. A commercial rate of interest on the loan note of Breadline would be 8% per annum.

Required

(a) Describe the matters that may be relevant when entity financial statements are used to assess the
performance of a company that is a wholly owned subsidiary. (5 marks)
Note. Your answer should give attention to related party issues.

(b) From the information above and with the aid of suitable ratios, prepare a report for your Chief Executive
on the overall financial position of Breadline. Your answer should include reference to matters in the
financial statements of Breadline that may give you cause for concern or require further investigation.
(20 marks)

(25 marks)

33. Comparator
Comparator assembles computer equipment from bought in components and distributes them to various
wholesalers and retailers. It has recently subscribed to an interfirm comparison service. Members submit
accounting ratios as specified by the operator of the service, and in return, members receive the average
figures for each of the specified ratios taken from all of the companies in the same sector that subscribe to the
service. The specified ratios and the average figures for Comparators sector are shown below.
Ratios of companies reporting a full years results for periods ending between
1 July 2009 and 30 September 2009
Return on capital employed
Revenue/capital employed
Gross profit margin
Net profit (before tax) margin
Current ratio
Quick ratio
Inventory holding period
Accounts receivable collection period
Accounts payable payment period
Debt to equity
Dividend yield
Dividend cover

56

221%
18 times
30%
125%
16:1
09:1
46 days
45 days
55 days
40%
6%
3 times

LSB_F7_Rev Kit:297mm x 210mm

28/10/09

13:37

Page 57

QUESTIONS
Comparators financial statements for the year to 30 September 2009 are set out below:
Income statement
Sales revenue
Cost of sales
Gross profit
Other operating expenses
Interest payable
Inventory write off due to obsolescence
Profit before taxation
Income tax expense
Profit for the year

Extracts of changes in equity


Retained earnings 1 October 2008
Net profit for the year
Dividends paid (interim $60,000; final $30,000)
Retained earnings 30 September 2009
Statement of financial position
Non-current assets (note (i))
Current assets
Inventory
Accounts receivable
Bank

$'000
2,425
(1,870)
555
(215)
(34)
(120)
186
(90)
96

179
96
(90)
185

$'000
275
320
Nil

Equity and liabilities


Equity
Ordinary shares (25 cents each)
Retained earnings
Non-current liabilities
8% loan notes
Current liabilities
Bank overdraft
Trade accounts payable

595
1,135
150
185
335
300

65
350
85

Notes

(i) The details of the non-current assets are:

At 30 September 2009

$'000
540

Cost
$'000
3,600

500
1,135
Accumulated
depreciation
$'000
3,060

(ii) The market price of Comparators shares throughout the year averaged $600 each.

Net book
value
$'000
540

Required

(a) Explain the problems that are inherent when ratios are used to assess a companys financial performance.
Your answer should consider any additional problems that may be encountered when using interfirm
comparison services such as that used by Comparator. (7 marks)

57

LSB_F7_Rev Kit:297mm x 210mm

28/10/09

13:37

Page 58

FINANCIAL REPORTING (INTERNATIONAL)


(b) Calculate the ratios for Comparator equivalent to those provided by the interfirm comparison service.
(6 marks)

(c) Write a report analysing the financial performance of Comparator based on a comparison with the sector
averages. (12 marks)

(25 marks)

34. Bigwood
Bigwood, a public company, is a high street retailer that sells clothing and food. The managing director is very
disappointed with the current years results. The company expanded its operations and commissioned a
famous designer to restyle its clothing products. This has led to increased sales in both retail lines, yet overall
profits are down.
Details of the financial statements for the two years to 30 September 2009 are shown below.
INCOME STATEMENT:
Revenue
Cost of sales

- clothing
- food
- clothing
- food

Gross profit
Other operating expenses
Operating profit
Interest expense
Profit before tax
Income tax expense
Profit for the year
SUMMARISED CHANGES IN EQUITY:
Retained profit b/f
Profit for the year
Dividends paid
Retained profit c/f

Year to 30 September
2009
$000
$000
16,000
7,000
23,000
14,500
4,750
(19,250)
3,750
(2,750)
1,000
(300)
700
(250)
450

Year to 30
September 2009
$000
1,900
450
(600)
1,750

Year to 30
September 2008
$000
1,100
1,400
(600)
1,900

STATEMENTS OF FINANCIAL POSITION AS AT:


Property, plant and equipment at cost
Accumulated depreciation
Current assets
Inventory

clothing
food
Trade receivables
Bank
Total assets
58

Year to 30 September
2008
$000
$000
15,600
4,000
19,600
12,700
3,000
(15,700)
3,900
(1,900)
2,000
(80)
1,920
(520)
1,400

30 September 2009
$000
$000
17,000
(5,000)
12,000

30 September 2008
$000
$000
9,500
(3,000)
6,500

2,700
200
100
Nil

1,360
140
50
450

3,000
15,000

2,000
8,500

LSB_F7_Rev Kit:297mm x 210mm

28/10/09

13:37

Page 59

QUESTIONS

Equity and liabilities


Issued ordinary capital ($1 shares)
Share premium
Retained earnings
Non-current liabilities
Long-term loans
Current liabilities
Bank overdraft
Trade payables
Current tax payable

30 September 2009
$000
$000

30 September 2008
$000
$000

5,000
1,000
1,750
7,750

3,000
Nil
1,900
4,900

3,000

1,000

930
3,100
220

4,250
15,000

nil
2,150
450

2,600
8,500

Note. The directors have signalled their intention to maintain annual dividends at $600,000 for the
foreseeable future.
The following information is relevant:
(i)

(ii)

The increase in property, plant and equipment was due to the acquisition of five new stores and the
refurbishment of some existing stores during the year. The carrying value of fixtures scrapped at the
refurbished stores was $1.2 million; they had originally cost $3 million. Bigwood received no scrap
proceeds from the fixtures, but did incur costs of $50,000 to remove and dispose of them. The losses
on the refurbishment have been charged to operating expenses. Depreciation is charged to cost of
sales apportioned in relation to floor area (see below).
The floor sales areas (in square metres) were:
Clothing
Food

(iii)
(iv)

30 September
2009
48,000
6,000
54,000

30 September
2008
35,000
5,000
40,000

2009
93%
21 times

2008
339%
33 times

94%
321%
20%
071:1

186%
25%
71%
077 :1

68 days
15 days
59 days
28%
33 times

39 days
17 days
50 days
17%
25 times

The share price of Bigwood averaged $6.00 during the year to 30 September 2008, but was only $3.00
at 30 September 2009.
The following ratios have been calculated:

Return on capital employed


Net assets turnover
Gross profit margin
clothing
food
Net profit (after tax) margin
Current ratio
Inventory holding period
clothing
food
Accounts payment period
Gearing
Interest cover

59

LSB_F7_Rev Kit:297mm x 210mm

28/10/09

13:37

Page 60

FINANCIAL REPORTING (INTERNATIONAL)


Required

(a) Prepare, using the indirect method, a statement of cash flows for Bigwood for the year to 30 September
2009 (12 marks)
(b) Write a report analysing the financial performance and financial position of Bigwood for the two years
ended 30 September 2009. (13 marks)

Your report should utilise the above ratios and the information in your statement of cash flows. It should
refer to the relative performance of the clothing and food sales and be supported by any further ratios you
consider appropriate.

(25 marks)

35. Tabba
The following draft financial statements relate to Tabba, a private company.
Statements of financial position as at:

Property, plant and equipment (note (ii))


Current assets
Inventories
Trade receivables
Insurance claim (note (iii))
Cash and bank

30 September 2008
$'000
$'000
15,800

2,550
3,100
1,500
850

1,850
2,600
1,200
nil

Total assets
Equity and liabilities
Share capital ($1 each)
Reserves:
Revaluation (note (ii))
Retained earnings

nil
2,550

Noncurrent liabilities
Finance lease obligations (note (ii))
6% loan notes
10% loan notes
Deferred tax
Government grants (note (ii))

2,000
800
nil
200
1,400

Current liabilities
Bank overdraft
Trade payables
Government grants (note (ii))
Finance lease obligations (note (ii))
Current tax payable

nil
4,050
600
900
100

Total equity and liabilities

60

30 September 2009
$'000
$'000
10,600

8,000
18,600
6,000

2,550
8,550

4,400

5,650
18,600

5,650
21,450
6,000

1,600
850

1,700
nil
4,000
500
900
550
2,950
400
800
1,200

2,450
8,450

7,100

5,900
21,450

LSB_F7_Rev Kit:297mm x 210mm

28/10/09

13:37

Page 61

QUESTIONS
The following additional information is relevant:

(i)

Income statement extract for the year ended 30 September 2009:


$000
270
(260)
40
50
50
100

Operating profit before interest and tax


Interest expense
Interest receivable
Profit before tax
Net income tax credit
Profit for the year

Note. The interest expense includes finance lease interest.


(ii)

The details of the property, plant and equipment are:

At 30 September 2008
At 30 September 2009

Cost
$000
20,200
16,000

Acc'd
depreciation
$000
4,400
5,400

Carrying
value
$000
15,800
10,600

During the year Tabba sold its factory for its fair value $12 million and agreed to rent it back, under an
operating lease, for a period of five years at $1 million per annum. At the date of sale it had a carrying
value of $7.4 million based on a previous revaluation of $8.6 million less depreciation of $1.2 million
since the revaluation. The profit on the sale of the factory has been included in operating profit. The
surplus on the revaluation reserve related entirely to the factory. No other disposals of noncurrent
assets were made during the year.
Plant acquired under finance leases during the year was $1.5 million. Other purchases of plant during
the year qualified for government grants of $950,000.

(iii)

Amortisation of government grants has been credited to cost of sales.

The insurance claim related to flood damage to the companys inventories which occurred in September
2008. The original estimate has been revised during the year after negotiations with the insurance
company. The claim is expected to be settled in the near future.

Required
(a)

(b)

Prepare a statement of cash flows using the indirect method for Tabba in accordance with IAS 7
Statement of Cash Flows for the year ended 30 September 2009. (17 marks)

Using the information in the question and your statement of cash flows, comment on the change in the
financial position of Tabba during the year ended 30 September 2009. (8 marks)
Note. You are not required to calculate any ratios.

(25 marks)

61

LSB_F7_Rev Kit:297mm x 210mm

28/10/09

13:37

Page 62

FINANCIAL REPORTING (INTERNATIONAL)

36. Nedberg
The financial statements of Nedberg for the year to 30 September 2009, together with the comparative
of financial position for the year to 30 September 2008 are shown below:
INCOME STATEMENT YEAR TO 30 SEPTEMBER 2009

Sales revenue
Cost of sales (note (1))
Gross Profit for the period
Operating expenses (note (1))
Interest Loan note
Profit before tax
Taxation
Profit for the year

$m
3,820
(2,620)
1,200
(300)
(30)
870
(270)
600

STATEMENTS OF FINANCIAL POSITION AS AT 30 SEPTEMBER


Non-current assets
Property, plant and equipment
Intangible assets (note (2))
Current assets
Inventory
Trade receivables
Cash
Total assets
Equity and liabilities
Ordinary Shares of $1 each
Reserves:
Share premium
Revaluation surplus
Retained earnings
Total equity
Non-current liabilities (note(3))
Current liabilities (note(4))
Total equity and liabilities

$m

1,420
990
70

Notes to the financial statements


(1)

(2)

2,480
5,020

2008
$m

940
680
nil

$m
1,830
300
2,130

1,620
3,750

750

500

350
140
1,890
3,130
870
1,020
5,020

100
nil
1,600
2,200
540
1,010
3,750

Cost of sales includes depreciation of property, plant and equipment of $320 million and a loss on the
sale of plant of $50 million. It also includes a credit for the amortisation of government grants.
Operating expenses include a charge of $20 million for the impairment of goodwill.
Intangible non-current assets

Deferred development expenditure


Goodwill

62

2009
$m
1,890
650
2,540

2009
$m
470
180
650

2008
$m
100
200
300

LSB_F7_Rev Kit:297mm x 210mm

28/10/09

13:37

Page 63

QUESTIONS
(3)

Non-current liabilities
10% loan note
Government grants
Deferred tax

(4)

Current liabilities
Trade payables
Bank overdraft
Accrued loan interest
Taxation

The following additional information is relevant:

(i)

(ii)

2009
$m
300
260
310
870

2009
$m
875
nil
15
130
1,020

2008
$m
100
300
140
540

2008
$m
730
115
5
160
1,010

Intangible non-current assets

The company successfully completed the development of a new product during the current year,
capitalising a further $500 million before amortisation charges for the period.
Property, plant and equipment/revaluation reserve

The company revalued its buildings by $200 million on 1 October 2008. The surplus was credited to
revaluation surplus.
New plant was acquired during the year at a cost of $250 million and a government grant of $50
million was received for this plant.

On 1 October 2008 a bonus issue of 1 new share for every 10 held was made from the revaluation
surplus.
$10 million has been transferred from the revaluation surplus to realised profits as a year-end
adjustment in respect of the additional depreciation created by the revaluation.

The remaining movement on property, plant and equipment was due to the disposal of obsolete plant.

(iii)

Share issues

(iv)

Dividends

In addition to the bonus issue referred to above Nedberg made a further issue of ordinary shares for cash.

Dividends paid during the year amounted to $320,000.

Required
(a)

(b)

A statement of cash flows for Nedberg for the year to 30 September 2009 prepared in accordance with
IAS 7 Statement of cash flows. (20 marks)
Comment briefly on the financial position of Nedberg as portrayed by the information in your
statement of cash flows. (5 marks)

(25 marks)

63

LSB_F7_Rev Kit:297mm x 210mm

28/10/09

13:37

Page 64

FINANCIAL REPORTING (INTERNATIONAL)

37. Minster
Minster is a publicly listed company. Details of its financial statements for the year ended 30 September 2009,
together with a comparative statement of financial position, are:
STATEMENT OF FINANCIAL POSITION AT
Non-current assets (note (i))
Property, plant and equipment
Software
Investments at fair value through
profit and loss
Current assets
Inventories
Trade receivables
Amounts due from construction
contracts
Bank

30 September 2008
$'000
$'000

1,280
135

940
nil

150
1,565

125
1,065

480
270

510
380

80
nil

55
35

Total assets
Equity and liabilities
Equity shares of 25 cents each
Reserves
Share premium (note (ii))
Revaluation reserve
Retained earnings

150
60
950

Non-current liabilities
9% loan note
Environmental provision
Deferred tax

120
162
18

Current liabilities
Trade payables
Bank overdraft
Current tax payable
Total equity and liabilities

64

30 September 2009
$'000
$'000

830
2,395
500

350
25
60

1,160
1,660

300

435
2,395

980
2,045
300

85
25
965

nil
nil
25
555
40
50

1,075
1,375

25

645
2,045

LSB_F7_Rev Kit:297mm x 210mm

28/10/09

13:37

Page 65

QUESTIONS
STATEMENT OF COMPREHENSIVE INCOME FOR THE YEAR ENDED 30 SEPTEMBER 2009
Revenue
Cost of sales
Gross profit
Operating expenses
Finance costs (note (i))
Investment income and gain on investments
Profit before tax
Income tax expense
Profit for the year
Other comprehensive income revaluation surplus on land
Total comprehensive income

The following supporting information is available:

(i)

$'000
1,397
(1,110)
287
(125)
162
(40)
20
142
(57)
85
35
120

Included in property, plant and equipment is a coal mine and related plant that Minster purchased on
1 October 2008. Legislation requires that in ten years' time (the estimated life of the mine) Minster will
have to landscape the area affected by the mining. The future cost of this has been estimated and
discounted at a rate of 8% to a present value of $150,000. This cost has been included in the carrying
amount of the mine and, together with the unwinding of the discount, has also been treated as a
provision. The unwinding of the discount is included within finance costs in the income statement.
Depreciation of property, plant and equipment for the year was $255,000.

There were no disposals of property, plant and equipment during the year.

The software was purchased on 1 April 2009 for $180,000.

(ii)
(iii)

The market value of the investments had increased during the year by $15,000. There have been no
sales of these investments during the year.

On 1 April 2009 there was a bonus (scrip) issue of equity shares of one for every four held utilising the
share premium reserve. A further cash share issue was made on 1 June 2009. No shares were
redeemed during the year.
A dividend of 5 cents per share was paid on 1 July 2009.

Required
(a)

(b)

Prepare a statement of cash flows for Minster for the year to 30 September 2009 in accordance with
IAS 7 Statement of cash flows. (15 marks)
Comment on the financial performance and position of Minster as revealed by the above financial
statements and your statement of cash flows. (10 marks)

(25 marks)

65

LSB_F7_Rev Kit:297mm x 210mm

28/10/09

13:37

Page 66

FINANCIAL REPORTING (INTERNATIONAL)

38. Rytetrend
Rytetrend is a retailer of electrical goods. Extracts from the companys financial statements are set out below:
INCOME STATEMENT FOR THE YEAR ENDED 31 MARCH
Sales revenue
Cost of sales
Gross profit
Other operating expense
Interest payable
loan notes
overdraft
Profit before taxation
Income tax expense
Profit for the year

$'000

(260)
(200)

2009

$'000
31,800
(22,500)
9,300
(5,440)

STATEMENTS OF FINANCIAL POSITION AS AT 31 MARCH


Assets
Non-current assets (note (i))
Current assts
Inventory
Receivables
Bank
Total assets
Equity and liabilities
Equity
Ordinary capital ($1 shares)
Share premium
Retained earnings
Non-current liabilities
10% loan notes
6% loan notes
Current liabilities
Bank overdraft
Trade payables
Taxation
Warranty provision (note (ii))
Total equity and liabilities

$'000
2,650
1,100
Nil

1,050
2,850
720
500

Notes

(i) The details of the non-current assets are:

At 31 March 2008
At 31 March 2009
66

Cost
$'000
27,500
37,250

2009

(460)
3,400
(1,000)
2,400

$'000
24,500

3,750
28,250

$'000

2008

$'000
23,500
(16,000)
7,500
(4,600)

(500)
Nil

$'000

(500)
2,400
(800)
1,600
2008

3,270
1,950
400

$'000
17,300

5,620
22,920

11,500
1,500
8,130
21,130

10,000
Nil
6,160
16,160

Nil
2,000

4,000
Nil

5,120
28,250

Nil
1,980
630
150

Accumulated
Depreciation Net book value
$'000
$'000
10,200
17,300
12,750
24,500

2,760
22,920

LSB_F7_Rev Kit:297mm x 210mm

28/10/09

13:37

Page 67

QUESTIONS
During the year there was a major refurbishment of display equipment. Old equipment that had cost $6
million in September 2004 was replaced with new equipment at a gross cost of $8 million. The equipment
manufacturer had allowed Rytetrend a trade in allowance of $500,000 on the old display equipment. In
addition to this Rytetrend used its own staff to install the new equipment. The value of staff time spent on
the installation has been costed at $300,000, but this has not been included in the cost of the asset. All staff
costs have been included in operating expenses. All display equipment held at the end of the financial year
is depreciated at 20% on its cost. No equipment is more than five years old.

(ii) Operating expenses contain a charge of $580,000 for the cost of warranties on the goods sold by
Rytetrend. The company makes a warranty provision when it sells its products and cash payments for
warranty claims are deducted from the provision as they are settled.
(iii) Dividends paid in the year ended 31 March 2009 were $430,000.

Required

(a) Prepare a cash flow statement for Rytetrend for the year ended 31 March 2009. (12 marks)

(b) Write a report briefly analysing the operating performance and financial position of Rytetrend for the years
ended 31 March 2008 and 2009. (13 marks)
Your report should be supported by appropriate ratios.

(25 marks)

39. Update
Most companies prepare their financial statements under the historical cost convention. In times of rising
prices it has been said that without modification such financial statements can be misleading.
Required

(a) Explain the problems that can be encountered when users rely on financial statements prepared under the
historical cost convention for their information needs. (6 marks)

Note: Your answer should consider problems with the income statement and the statement of financial
position.

(b) Update has been considering the effect of alternative methods of preparing their financial statements. As an
example they picked an item of plant that they acquired from Suppliers on 1 April 2006 at a cost of
$250,000.
The following details have been obtained:

the company policy is to depreciate plant at 20% per annum on the reducing balance basis.
the movement in the retail price index has been:

Suppliers price catalogue at 31 March 2009 shows an item of similar plant at a cost of $320,000. On
reading the specification it appears that the new model can produce 480 units per hour whereas the model
owned by Update can only produce 420 units per hour.

1 April 2006
1 April 2007
1 April 2008
31 March 2009

180
202
206
216

Required

Calculate for Update the depreciation charge for the plant for the year to 31 March 2009 (based on year end
values) and its carrying value in the statement of financial position on that date using:

the historical cost basis;


a current purchasing power basis; and
a current cost basis. (6 marks)

(12 marks)
67

LSB_F7_Rev Kit:297mm x 210mm

28/10/09

13:37

Page 68

FINANCIAL REPORTING (INTERNATIONAL)

40. Appraisal
Reactive is a publicly listed company that assembles domestic electrical goods which it then sells to both
wholesale and retail customers. Reactives management were disappointed in the companys results for the
year ended 31 March 2008. In an attempt to improve performance the following measures were taken early in
the year ended 31 March 2009:
A national advertising campaign was undertaken
Rebates to all wholesale customers purchasing goods above set quantity levels were introduced
The assembly of certain lines ceased and was replaced by bought in completed products. This allowed
Reactive to dispose of surplus plant.
Reactives summarised financial statements for the year ended 31 March 2009 are set out below:
Income statement

Revenue (25% cash sales)


Cost of sales
Gross profit
Operating expenses
Profit on disposal of plant (note 1)
Finance charges
Profit before tax
Income tax expense
Profit for the period

Statement of financial position


Non-current assets
Property, plant and equipment (note 1)
Current assets
Inventory
Trade receivables
Bank

$million

250
360
Nil

Equity and liabilities


Equity shares of 25c each
Retained earnings

Total equity and liabilities

68

$million
550

Total assets

Non-current liabilities
8% loan notes
Current liabilities
Bank overdraft
Trade payables
Current tax payable

$million
4,000
(3,450)
550
(370)
180
40
(20)
200
(50)
150

610
1,160
100
380
480
200

10
430
40

480
1,160

LSB_F7_Rev Kit:297mm x 210mm

28/10/09

13:37

Page 69

QUESTIONS
Below are ratios for the year ended 31 March 2008:

Return on capital employed (profit before interest and tax


over total assets less current liabilities)
Net asset (equal to capital employed) turnover
Gross profit margin
Net profit (before tax) margin
Current ratio
Closing inventory holding period
Trade receivables collection period
Trade payables payment period
Dividend yield
Dividend cover

Notes:

28.1%
4 times
17%
6.3%
1.6:1
46 days
45 days
55 days
3.75%
2 times

Reactive received $120million from the sale of plant that had a carrying amount of $80million at the date of
its sale;
The market price of Reactives shares throughout the year averaged $3.75 each;
There were no issues of redemption of shares or loans during the year;

Dividends paid during the year ended 31 March 2009 amounted to $90million, maintaining the same
dividend paid in the year ended 31 March 2008.

Required

(a) Calculate ratios for the year ended 31 March 2009 (showing your workings) for Reactive, equivalent to
those provided above (10 marks)

(b) Analyse the financial performance and position of Reactive for the year ended 31 March 2009 compared to
the previous year (10 marks)

(c) Explain in what ways your approach to performance appraisal would differ if you were asked to assess the
performance of a not-for-profit organisation (5 marks)

(25 marks)

69

LSB_F7_Rev Kit:297mm x 210mm

28/10/09

13:37

Page 70

FINANCIAL REPORTING (INTERNATIONAL)

70

LSB_F7_Rev Kit:297mm x 210mm

28/10/09

13:37

Page 71

F7
Answers

LSB_F7_Rev Kit:297mm x 210mm

28/10/09

13:37

Page 72

FINANCIAL REPORTING (INTERNATIONAL)

72

LSB_F7_Rev Kit:297mm x 210mm

28/10/09

13:37

Page 73

ANSWERS

1.

(a)

Hepburn

Consolidated income statement for Hepburn for the year ended 31 March 2009
Revenue
Cost of sales

Gross profit
Operating expenses
Debenture interest

Profit before tax


Income tax expense

Profit for the year


Attributable to:
Owners of the parent (_)
Non-controlling interest (W5)
Consolidated Statement of Financial Position for Hepburn
as at 31 March 2009
Non-current assets
Tangible: Property, plant and equipment (620 + 660 + 125)
Intangible: Goodwill (W4)
Investments (20 + 10)
Current assets
Inventory (240 + 280 10 (W7))
Accounts receivable (170 + 210 56 Interco)
Bank (20 + 40 + 20 cash in transit)
Total assets

Equity and Liabilities


Equity shares (400 + 300 (W8))
Share premium (W8)
Retained earnings (W6)
Non-controlling interest (W5)
Non-current liabilities
8% debentures
Current liabilities
Trade accounts payable (170 + 155 36 Interco)
Tax payable (50 + 45)
Total equity and liabilities

$000
1,600
(890)
22
710
(184)
(6)
22
520
(120)
22
400
22
388
20

Marks
1

See W2
See W2
1/2
1/2

1/2

$000
1,405
180
30
22
1,615
510
324
80
22
914
22
2,529
22
700
600
500
22
1,800
195
150
289
95
384
22
2,529
22

1/2
1/2

1
1
1

1
1

See W6
1
1/2

1/2

73

LSB_F7_Rev Kit:297mm x 210mm

28/10/09

13:37

Page 74

FINANCIAL REPORTING (INTERNATIONAL)


Workings
W1

Group structure
H
80%
S

W2

Consolidation schedule
Revenues
Cost of sales
URP (W7)
Operating expenses
Impt of GW
Debenture interest
Income taxes

W3

H
$000
1,200
(650)
(10)
(120)
(20)

(100)

Net assets of Salter


Share capital
Retained earnings
Fair value adjt

W4

1 Oct 2008 (owned 6m)

Goodwill

S (6m)
$000
500
(330)

Goodwill
Impairment charge (

Carrying value of goodwill


Tutorial note

(184)

(6)
(20)
22
100

(6)
(120)

@ Acqn date
$000
150
600

125
22
975

125
22
875

(20% x 875 (W3))


Fair value of net assets of Salter at acquisition (W3)

Group
$000
1,600
(890)

(44)

@ Repg date
$000
150
700

Consideration (W8)
Non-controlling interest at acquisition

Adj
$000
(100)
100

1/2
1/2

See W4

Movement
$000
100

(700 6/12
x 200)

$000
900
175
22
1,075
(875)
22
200
(20)
22
180

In this question, the NCI is measured as a proportion of net assets.Therefore there is no NCI
goodwill.The impairment loss of $20m is therefore attributable to the parent company only.As such it
is charged to the parent companys profits in (W2) and (W6)

74

See W8
1/2
1/2

LSB_F7_Rev Kit:297mm x 210mm

28/10/09

13:37

Page 75

ANSWERS
W5

Non-controlling interest
NCI in profit: 20% x 100 (W2)
NCI in net assets: 20% x 975 (W3)

W6

Group reserves
Hepburn
Group share of Salters post acquisition profits (80% x 100 (W3))
Impairment of goodwill
URP in inventory (W7)

W7

URP in inventory

W8

Share for share exchange

$000
20
195
$000
450
80
(20)
(10)
22
500

1/2

1/2
1/2

$100,000 x 1/2 x 25/125 = $10,000

Shares acquired:

Therefore shares issued:


Consideration:

150,000 x 80%
= 120,000
120,000 x 5/2
= 300,000
300,000 x $3
= $900,000

1
1

Nominal value of new shares to be recorded in CSFP: $300,000


(b)

Premium to be recorded in CSFP: $600,000

Hepburns directors have treated Woodbridge as a long-term non-current asset investment rather
than an associate or subsidiary.

An associate is an entity over which another entity has significant influence.This is presumed to exist
when between 20% and 50% of the voting power is held by the investing company. The presumption
can, however be rebutted where is can be clearly shown that the investor does not have significant
influence.
A subsidiary is an entity over which another entity has control, presumed to exist where more than
50% of the voting power is held by the investing company.
In their assessment of the investment in Woodbridge, directors have concentrated on the fact that
Hepburn owns 25% of the total (type A and B) shares in Woodbridge however exerts no influence
over the operating policies or management of the company. On this basis they appear to have
rebutted the presumption that Woodbridge is an associate.

Hepburn does however own 60% of the equity voting A shares in Woodbridge and so controls 60% of
the voting power, meaning that it meets the definition of a subsidiary.
On this basis it appears that the results of Woodbridge should be consolidated into the group
accounts on a line by line basis. It is not an option to exclude Woodbridge from consolidation on the
basis that it is loss-making and would adversely affect group results.

1
1

75

LSB_F7_Rev Kit:297mm x 210mm

28/10/09

13:37

Page 76

FINANCIAL REPORTING (INTERNATIONAL)

2.

(a)

Highveldt

Tutorial note

Although this is not a full consolidation question, it should be approached in the same way, using
standard workings.
(i) Goodwill

Consideration
Cash ($3.50 x 75% x 80m)
Deferred cash ($108m/1.08)

Non-controlling interest at acquisition


(25% x 296 (W2))

Fair value of net assets of Salter at acquisition (W2)


Goodwill
Impairment charge

Carrying value of goodwill

Tutorial note

(a)
(a)

210
100
22
310
74
22
384
(296)
22
88
(22)
22
66

Marks
1
1

1/2

for 25%
+ See W2

In this question, the NCI is measured as a proportion of net assets.Therefore there is no NCI
goodwill.The impairment loss of $22m is therefore attributable to the parent company only.As such it
is charged against group reserves (a) (iii).
(ii) Non-controlling interest

NCI in net assets: 25% x 326 (W2)

(iii) Group share premium

Highveldt
75% post acquisition movement in share premium of Samson

Group revaluation reserve


Highveldt
75% post acquisition movement in revaluation reserve of Samson (75% x 4)
Group retained earnings
Highveldt
75% post acquisition movement in share premium of Samson (75% x 48 (W2)
Write off of goodwill
Interest ($60m x 10%)
Unwinding of discount on deferred consideration

76

$m

$m

81.5

$m

1/2

for 25%
+ see W2

80
Nil
22
80
$m
45
3
22
48
350
36
(22)
6
(8)
22
362

1/2

1/2

for 75%
1 for 4

1/2

1/2

for 75%
+ See W2
1
1
1

LSB_F7_Rev Kit:297mm x 210mm

28/10/09

13:37

Page 77

ANSWERS
Workings

W1

Group structure
H
75%
S

W2

Net assets of Salter

@ Repg date
$m
Share capital
80
Share premium
40
Revaluation reserve

FV adjustment (i)
24
Retained earnings
210
Brand (iii)
36
Development costs (iv)
(40)
URP (v) 6m/3
(2)
22
348

Tutorial notes:

(b)

1 April 2008 (held 1yr)


@ Acqn date
$m
80
40

20
134
40
(18)

Movement
$000

4
48

22
296

1/2

1/2

1+1

1/2

+ 1/2

1+1
1+1
1

The increase in the fair value of the land and buildings post acquisition is treated as part of the
revaluation reserve

The brand is internally generated for Samson and so cannot appear on its own statement of
financial position. For the group, however, it is a purchased assed and is therefore included as an
asset at its fair value at acquisition. Subsequent amortisation is charged to group reserves.

The development costs should not have been capitalised as they do not meet the IAS 38 criteria.
The capitalised amount is therefore reversed.

In the separate entity financial statements of a parent company, a subsidiary is held as a non-current
asset investment at cost.Any dividend income is recorded in the parents income statement.

The cost of the investment in the subsidiary is likely to differ significantly from the value of the net
assets owned. In the first years of ownership, the cost will normally exceed the net assets owned (the
excess being goodwill). After a number of years when the subsidiary has made profits, the cost of the
investment will be less than the book value of the net assets owned.

1 mark per
valid point

Similarly, the dividend paid by a subsidiary to its parent company and so reflected in their income
statement is unlikely to be equal to the parent companys share of that subsidiarys profits.
For these reasons, single entity financial statements of a parent company provide only a limited
amount of useful information about its subsidiaries.

Full consolidated accounts, whereby all of the subsidiarys assets and liabilities and income and
expenses are added on a line by line basis (and then the share owned by the non-controlling interest
allocated to them) are more useful because:

They reflect the fact that the whole of the subsidiary, its assets, liabilities and profits are
controlled by the parent company.

77

LSB_F7_Rev Kit:297mm x 210mm

28/10/09

13:37

Page 78

FINANCIAL REPORTING (INTERNATIONAL)

3.
(a)

They reflect the historic performance of the subsidiary by including accumulated profits.

They quantify goodwill, being the premium paid on acquisition of the subsidiary.

The full position and performance of a subsidiary (such as high gearing or losses) are revealed to
users of the accounts

Hydan

Consolidated income statement for Hydan for the year ended 31 March 2009
Revenue
Cost of sales

Gross profit
Operating expenses
Interest income
Finance costs

Profit before tax


Income tax expense

Profit for the year


Attributable to:
Owners of the parent ( )
Non-controlling interest (W5)

$000
103,200
(77,500)
222
25,700
(20,175)
150
(420)
222
5,255
(3,200)
222
2,055
3,605
(1,550)

Consolidated Statement of Financial Position for Hydan as at 31 March 2009


Non-current assets
Property, plant and equipment (18,400 + 9,500 + 900)
Goodwill (W4)
Investments (16,000-10,800 (W4) 4,000 loan)
Current assets (18,000 + 7,200 200 URP 1,000 Interco)
Total assets

Equity and Liabilities


Equity shares
Share premium
Retained earnings (W6)
Non-controlling interest (W5)
Non-current liabilities
Bank loan
Current liabilities (11,400 + 3,900 1,000 Interco)

Total equity and liabilities

78

Marks
1

See W2
See W2
1
1
1/2

See W5

$000

28,800
2,925
1,200
222
32,925
24,000
222
56,925
222
10,000
5,000
17,675
222
32,675
3,950
6,000
14,300
222
56,925
222

See W4
1

11/2

1/2

1/2

See W6
See W5
1/2

LSB_F7_Rev Kit:297mm x 210mm

28/10/09

13:37

Page 79

ANSWERS
Workings

W1

Group structure
H
60%
S

W2

Consolidation schedule
Revenues
Cost of sales
URP (W7)
Extra depn
Operating expenses
Impt of GW
Interest income
Finance costs
Income taxes

W3

H
$000
98,000
(76,000)
(11,800)
350
(420)
(4,200)

Net assets of Systan


Share capital
Share premium
Retained earnings
Fair value adjt
Depn on FV adjt
URP (W7)
GW impairment

W4

1 Oct 2008 (owned 6m)

@ Repg date
$000
2,000
500
6,300
1,200
(300)
(200)
(375)
222
9,125

Goodwill

S
$000
35,200
(31,000)
(200)
(300)
(8,000)
(375)
(200)
1,000
3,875
@ Acqn date
$000
2,000
500
9,300
1,200

Fair value of net assets of Salter at acquisition (W3)


)

Carrying value of goodwill

Group
$000
103,200
(77,500)

(200)
200

(20,175)
150
(420)
(3,200)

Movement
$000

1/2

1
1

1/2

1/2

(6,300 + 3,000
post acqn loss)
(3,875)

1/2
1/2
1/2
1/2
1/2

222
13,000

Consideration ($9 x 60% x 2,000)


Fair value of non-controlling interest at acquisition
Goodwill
Impairment charge (

Adj
$000
(30,000)
30,000

$000
10,800
5,500
222
16,300
(13,000)
222
3,300
(375)
222
2,925

1/2

See W3

79

LSB_F7_Rev Kit:297mm x 210mm

28/10/09

13:37

Page 80

FINANCIAL REPORTING (INTERNATIONAL)


Tutorial note

In this question, the NCI is measured at fair value.Therefore goodwill attributable to the NCI arises.
The impairment loss of $375,000 is therefore attributable to the NCI and parent company in
proportion to their shareholdings. Including this impairment in the subsidiarys column in W2 and the
subsidiarys net assets at the reporting date in W3 ensures that the NCI is allocated their share of the
loss and the group share is included within profits attributable to the owners of the parent company
and group reserves.
W5

Non-controlling interest

$000
(1,550)

NCI in profit: 40% x (3,875) (W2)


NCI in net assets: 40% x 9,125 (W3)
NCI goodwill (5,500 (40% x 13,000))
W6

3,650
300
222
3,950

Group reserves

$000
20,000
(2,325)
222
17,675

Hydan
Group share of Systans post acquisition loss (60% x (3,875) (W3))
W7
(b)

URP in inventory

$4m x 5% = $200,000

In the six months prior to its acquisition by Hydan, Systan made a profit of $3.6million. In the six
months after the acquisition, the company made a loss of $3million.This appears to be a result of the
following:

In the post acquisition period, 85% of revenue is derived from sales to Hydan. On these sales,
Systan makes a 5% profit on selling price. Prior to the acquisition a profit margin of 25% was made
consistently ($6m/$24m). This indicates that Hydan has imposed a low transfer price on Systan
which has contributed to the loss for the period.

Operating expenses have increased from $1.2 million in the six months prior to the acquisition to
$8m in the six months post acquisition.This may be the result of Hydan passing group expenses
back to Systan or re-charging management costs.

Finance costs of $200,000 are the direct result of the loan from Hydan, made to finance an increase
in production capacity for the benefit of Hydan. In a normal commercial situation, the effect of
such interest costs on profits would be negated by increased gross profits as a result of the increase
in production. In this instance, however, the low transfer price means this is not the case.

Overall, therefore, the acquisition appears to have had a negative effect on Systans operating
performance. Furthermore, Systan appear to have increased the margin charged on goods sold to
third parties to over 50% (W) which is likely to result in a loss of external customers.This will
further exacerbate the problem in future years.

Working
Revenue
Profit
Margin

80

Hydan
$30m
5% x 30m
= $1.5m
5%

Third parties
$5.2m
$2.7m

Total
$35.2m
$4.2m

52%

12%

1/2

for 40%
+ See W3
See W3
1/2

1/2

1/2

for 60%
+ see W3

1 mark per
valid point

LSB_F7_Rev Kit:297mm x 210mm

28/10/09

13:37

Page 81

ANSWERS

4.

(a)

Holdrite

Goodwill

Consideration
Shares: (10,000 x 75%) x 2/3 x $6
Loan note: (10,000 x 75%) /250 x $100
Non-controlling interest at acquisition 25% x 34,000 (W2)
Fair value of net assets of Staybrite at acquisition (W2)
Goodwill at 1 April 2009
Impairment charge

Goodwill at 30 September 2009

Tutorial note

$000
30,000
3,000
222
33,000
8,500
222
41,500
(34,000)
222
7,500
(750)
222
6,750

1
1
1/2

for 25%
+ see W2
See W2
1/2

In this question, the NCI is measured as a proportion of net assets.Therefore there is no NCI
goodwill.The impairment loss of $750,000 is therefore attributable to the parent company only.As
such it is charged against the parents profits in W3 below.

Investment in Associate
Cost
Shares: (5,000 x 40%) x3/4 x $6
Cash: (5,000 x 40%) x $1

Investment at 1 April 2009


Share of post-acquisition profits 40% x 2,000 (W2)

(b)

Investment at 30 September 2009

9,000
2,000
222
11,000
800
222
11,800

1
1
1

Consolidated income statement for Holdrite for the year ended 30 September
2009
Revenue
Cost of sales

Gross profit
Operating expenses
Interest expense
Share of profit of associates (4,000 x 6/12 x 40%)
Profit before tax
Income tax expense

(c)

$000

Profit for the year


Attributable to:
Owners of the parent ( )
Non-controlling interest (25% x 4,000 (W3))

Movement on consolidated retained earnings attributable to Holdrite


Brought forward at 1 October 2008
Profit attributable to Holdrines shareholders
Dividends paid
Carried forward at 30 September 2009

$000
85,350
(48,750)
222
36,600
(15,730)
(170)
800
222
21,500
(6,300)
222
15,200
14,200
1,000
$000
18,000
14,200
(5,000)
222
27,200

See W3
See W3
1 + 1 for
40%
1

1 + 1 for
25%

1/2

1/2

81

LSB_F7_Rev Kit:297mm x 210mm

28/10/09

13:37

Page 82

FINANCIAL REPORTING (INTERNATIONAL)


W1

Group structure
H

75%

40%
A

S
W2

Net assets of Staybrite at acquisition

1 April 08 (owned 6m)


$000
10,000
4,000
12,000
3,000
5,000
222
34,000

Share capital
Share premium
Retained earnings (7,500 + (9,000 x 6/12))
FV Adjustment land
FV adjustment plant
W3

Consolidation schedule
Revenue
Cost of sales
URP (4m x 1/4)
Extra depreciation
Operating expenses
Impt of GW
Interest expense
Income taxes

82

H
$000
75,000
(47,400)
(1,000)
(10,480)
(750)
(170)
(4,800)

S (6m)
$000
20,350
(9,850)
(500)
(4,500)
(1,500)
222
4,000

Adj
$000
(10,000)
10,000

Group
$000
85,350
(48,750)
(15,730)
(170)
(6,300)

1/2

1/2
1/2

2
1
1
1
1

LSB_F7_Rev Kit:297mm x 210mm

28/10/09

13:37

Page 83

ANSWERS

5.

Hedra

Consolidated Statement of Financial Position for Hedra as at 30 September 2009


Non-current assets
Property, plant and equipment
(358 + 240 + 25 FV adjt + 12 revn + 15 FV adjt)
Goodwill (W3)
Investments
Investment in associate (W6)

Current assets
Inventory (130 + 80)
Trade receivables (142 + 97)
Cash and bank
Total assets

Equity and Liabilities


Equity shares (400 + 80 (W7))
Share premium (40 + 120 (W7))
Revaluation reserve (W5)
Retained earnings (W5)
Non-controlling interest (W4)
Non-current liabilities
Deferred tax (45 (40m x 25%))
Current liabilities
Trade payables (118 + 141)
Bank overdraft
Current tax payable
Deferred consideration (acquisition of Salvador)
Total equity and liabilities

$m

650

94
45
220
222
1,009
210
239
4
222
453
222
1,462
222
480
160
30
269
222
939
118
35
259
12
50
49
222
370
222
1,462
222

Marks

1/2 + 1 +
1/2 + 1

See W3
1/2

See W6

1/2

1/2
1/2

1
1

See W5
See W5
See W4
1
1/2

1/2
1/2

83

LSB_F7_Rev Kit:297mm x 210mm

28/10/09

13:37

Page 84

FINANCIAL REPORTING (INTERNATIONAL)


Workings
W1

Group structure
H
60%

40%
A

W2

Net assets of Salvador


Share capital
Share premium
Revaluation reserve
FV adjustment (land)
Retained earnings
Fair value adjt plant
FV adjt deferred tax
asset (40 x 25%)
GW impairment
Net assets of Aragon
Share capital
Retained earnings

W3

S bought 1 Oct 2008 (owned 1 yr)


A bought 1 April 2009 (owned 6m)
@ Repg date
$m
120
50
25
60
15
10
(20)
222
260
@ Repg date
$000
100
300

@ Acqn date
$m
120
50
20
20
20
10
222
240
@ Acqn date
$000
100
250

Goodwill
Consideration
Cash
Deferred cash
Fair value of non-controlling interest at acquisition
Fair value of net assets of Salvador at acquisition (W2)

Goodwill
Impairment charge

Carrying value of goodwill

84

Movement
$m

1/2

1
1

15
222
20

1/2

Movement
$000
50

200 + (100 x
6/12)
$m
195
49
222
244
110
222
354
(240)
222
114
(20)
222
94

1/2

1/2

See W2
1/2

LSB_F7_Rev Kit:297mm x 210mm

28/10/09

13:37

Page 85

ANSWERS
Tutorial note

In this question, the NCI is measured at fair value.Therefore goodwill attributable to the NCI arises.
The impairment loss of $20,000 is therefore attributable to the NCI and parent company in
proportion to their shareholdings. Including this impairment in the subsidiarys net assets at the
reporting date in W2 ensures that the NCI is allocated their share of the loss and the group share is
included within group reserves.
W4

Non-controlling interest

FV at acquisition
NCI share of post-acquisition movement in reserves 40% x 20 (W2)
W5

Group retained earnings


Hedra
Group share of Salvadors post acquisition profits (60% x 15 (W2))
Group share of Aragons post acquisition profits (40% x 50 (W2))
Group revaluation reserve
Hedra
Revaluation of Hedras land and buildings
Group share of Salvadors post acquisition revaluation surplus
(60% x 5 (W2))

Tutorial note

$m
110
8
222
118
$m
240
9

20
222
269
$m
15
12
3
222
30

1
1

1/2
for 60%
+ see W2

1/2

1/2

1/2

Increases in the fair value of non-current assets of the subsidiary since acquisition should be recorded
as a revaluation reserve.
W6

Investment in associate

Cost (W7)
Group share of post acquisition profits 40% x 50 (W2)
W7

Share for share exchange


Shares issued

40m x 2
= 80m
Consideration:
300,000 x $2.50
= $200m
Nominal value of new shares to be recorded in CSFP: $80m
Premium to be recorded in CSFP: $120m

$m
200
20
222
220

See W7
1/2

1
1/2

85

LSB_F7_Rev Kit:297mm x 210mm

28/10/09

13:37

Page 86

FINANCIAL REPORTING (INTERNATIONAL)

6.

(a)

Hosterling

Goodwill

Consideration
3/5 x 20,000 x 80% x $5
Non-controlling interest at acquisition
20% x 50,000 (W2)
Fair value of net assets of Sunlee at acquisition (W2)
Goodwill at 1 October 2008

Investment in Associate
Cost
Cash: $3 x 6m
Loan notes: 6m/100 x $100
Group share of post-acquisition loss
40% x 20m x 3/12
Investment at 30 September 2009
Impairment loss

Carrying value at 30 September 2009

(b)

Marks

48,000
10,000
222
58,000
(50,000)
222
8,000

1
1/2

for 20%
+ W2

See W2

$000
18,000
6,000
222
24,000

(2,000)
222
22,000
(500)
222
21,500

Consolidated income statement for Hosterling for the year ended 30 September
2009
Revenue
Cost of sales

Gross profit
Distribution costs
Administrative expenses
Finance costs
Share of profit of associates (W4)
Profit before tax
Income tax expense
Profit for the year

Attributable to: Owners of the parent ( )


Non-controlling interest (20% x 12,000 (W3))

86

$m

$000
149,000
(89,000)
222
60,000
(6,000)
(16,100)
(2,100)
(2,500)
222
33,300
(11,300)
222
22,000
19,600
2,400

1 1/2

See W3
1/2

See W3
1/2

See W6
1/2

1
1 + 1 for
20%

LSB_F7_Rev Kit:297mm x 210mm

28/10/09

13:37

Page 87

ANSWERS
W1

Group structure
H
80%

40%
A

W2

S acquired 1 October 2008 (1 year)


A acquired 1 July 2009 (3m)

Net assets of Sunlee at acquisition

$000
20,000
18,000
4,000
3,000
5,000
222
50,000

Share capital
Retained earnings
FV adjustment intellectual property
FV Adjustment land
FV adjustment plant
W3

Consolidation schedule
Revenue
Cost of sales
URP (7.5m x 25/125)
Extra depreciation
Distribution costs
Administrative expenses
Impt of GW in S
Finance costs
Income taxes

H
$000
105,000
(68,000)
(1,500)
(4,000)
(7,500)
(1,600)
(1,200)
(8,700)

Tutorial note

S
$000
62,000
(36,500)
(1,000)
(2,000)
(7,000)
(900)
(2,600)
222
12,000

Adj
$000
(18,000)
18,000

Group
$000
149,000
(89,000)
(6,000)
(16,100)
(2,100)
(11,300)

1/2

1
1
1

1 1/2

1 1/2
1
1

1/2

1/2

1/2

1/2

In this question, the NCI is measured as a proportion of net assets.Therefore there is no NCI
goodwill.The impairment loss of $1,600,000 is therefore attributable to the parent company only.As
such it is charged in Hs column in W3
W4

Share of losses of Associate

40% x (20m x 3/12)


Impairment (part (a))

$000
(2,000)
(500)
222
(2,500)

1
1

87

LSB_F7_Rev Kit:297mm x 210mm

28/10/09

13:37

Page 88

FINANCIAL REPORTING (INTERNATIONAL)

7.

Horsefield

Consolidated Statement of Financial Position for Horsefield as at 31 March 2009


Non-current assets
Property, plant and equipment (8,050 + 3,600 + 120 FV adjustment)
Goodwill (W3)
Licence (180 60)
Investments (4,000 3,240 630 + 910)
Investment in associate (W6)
Current assets
Inventory (830 + 340)
Accounts receivable (520 + 290 40 Interco)
Cash and bank (240 + 40 cash in transit)
Total assets

Equity and Liabilities


Equity shares
Retained earnings (W5)

$000

11,770
1,190
120
1,040
717
222
14,837
1,170
770
280
222
2,220
222
17,057
222
5,000
8,883
222
13,883
394

Non-controlling interest (W4)


Non-current liabilities
Loan notes (500 + 240)
Current liabilities
Accounts payable (420 + 960)
Taxation (220 + 250)
Overdraft

740
1,380
470
190
222
2,040
222
17,057
222

Total equity and liabilities


Workings
W1

Group structure
H
90%
S

88

30%
A
S bought 1 April 07 (owned 2 yrs)
A bought 1 Oct 08 (owned 6m)

Marks

1 1/2

See W3
1

1 1/2

See W6

1/2

1
1

1/2

See W5
See W4
1/2
1/2
1/2
1/2

LSB_F7_Rev Kit:297mm x 210mm

28/10/09

13:37

Page 89

ANSWERS
W2

W3

Net assets of Sandfly

@ Repg date
$000
Share capital
1,200
Retained earnings
2,300
Fair value adjt property
120
FV adjt licence
180
Amortisation of
licence (2/6 x 180)
(60)
222
3,740
Goodwill

@ Acqn date
$000
1,200
800
120
180
222
2,300

Consideration
90% x 1,200,000 x $3
Fair value of non-controlling interest at acquisition
Fair value of net assets of Salvador at acquisition (W2)

Goodwill
W4

Non-controlling interest
FV at acquisition
NCI share of post-acquisition profits (10% x 1,440 (W2))

W5

Group retained earnings


Horsefield
Group share of Sandflys post acquisition profits
(90% x 1,440 (W2))
Group share of Anthills post acquisition profits
(30% x (600 x 6/12))
Group share of URP (30% x $1,000 (W7))

W6

Investment in associate
Cost (30% x 600,000 x $3.50)
Group share of post acquisition profits
(30% x (600 x 6/12))
Group share of URP (30% x $1,000 (W7))

W7

URP in inventory
2/3 x $65,000 x 30/130

Movement
$m

1/2

1/2

1+1
1+1
1

1,440

$000
3,240
250
222
3,490
(2,300)
222
1,190
$000
250
144
222
394

1
1
See W2

1/2

$000
7,500
1,296
90
(3)
222
8,883
$m
630
90
(3)
222
717

1
for 10%
+ W2

1
1/2

for 90%
+ W2
1
1

1
1
1

$000
1

89

LSB_F7_Rev Kit:297mm x 210mm

28/10/09

13:37

Page 90

FINANCIAL REPORTING (INTERNATIONAL)


Tutorial note

(b)

Where the associate is the selling company, the group share of the URP is deducted from both group
reserves and the investment in the associate.
An associate is defined as an entity over which an investor has a significant influence. Significant
influence is the power to participate in the financial and operating policy decisions of the investee.

1 mark per
valid point

Significant influence is presumed to exist where an investor has at least 20% of the voting power in
another company (but normally not more than 50% as this indicates control). This presumption can
however be rebutted where it can clearly be demonstrated that the investor does not have significant
influence.

Significant influence can be evidenced in a number of different ways, for example:

Representation on the board of directors

8.

(a)

Participation in policy-making decisions such as the amount of a dividend


Material transactions between the investor and investee

Interchange of management personnel between the two companies

Provision by one company to another of essential technical information.

Winger

Income statement for Winger for the year ended 31 March 2009
Revenue (358,450 27,000)
Cost of sales (W1)

Gross profit
Distribution expenses
Administration expenses
Profit on disposal of land and buildings (W2)
Write off of research project
Profit before interest and tax
Finance costs (7,200 + (50m x 8%))

Profit before tax


Income tax (15,000 2,200)
Profit for the year

Tutorial note

Marks

$000
331,450
(208,550)
222
122,900
(28,700)
(15,000)
15,000
(30,000)
222
64,200
(11,200)
222
53,000
(12,800)
222
40,200

1/2

See W1
1/2
1/2
1/2
1/2

1 1/2
1

The writing off of the development costs should not be treated as a prior period adjustment through
reserves as it does not constitute a change in accounting policy. Wingers accounting policy has in fact
not changed, the costs are being written off as they no longer meet the capitalisation criteria of IAS 38.

Statement of changes in equity

Balance at 1 April 2008


Total comprehensive income
Dividends paid
Transfer from revaluation surplus (W2)
Balance at 31 March 2009

90

Retained
earnings
$000
71,600
40,200
(12,000)
30,000
222
129,800

1/2
1/2

LSB_F7_Rev Kit:297mm x 210mm

28/10/09

13:37

Page 91

ANSWERS
(b)

Statement of financial position of Winger at 31 March 2009


Non-current assets
Property (W4)
Plant and equipment (W4)
Current assets
Inventory (28,240 + 22,500 sale or return)
Receivables (55,000 27,000 sale or return)
Cash
Total assets

Equity and liability


25c Equity shares
Retained earnings (part (a))
Non-current liabilities
Finance lease (W3)
8% loan notes
Current liabilities
Trade accounts payable
Income tax
Finance lease (W3)
Loan interest accrual ((50,000 x 8%) 2,000)
Total equity and liabilities

$000
194,000
160,000
222
354,000
50,740
28,000
10,660
222
89,400
222
443,300
222
150,000
129,800
222
279,800
47,200
50,000
29,400
15,000
20,000
2,000
222
66,400
222
443,300
222

1 1/2

1
1

1/2

1/2
1/2

1/2
1/2

1
1
1

Workings

(W1) Cost of sales


Draft per TB
Sale or return inventory (27,000 x 100/120)
Depreciation of plant (W5)
Depreciation of property (W5)
(W2) Disposal of revalued property
Proceeds
Carrying value

Profit on disposal
Carrying value at disposal
Historic cost

Transfer from revaluation reserve to retained earnings

$000
185,050
(22,500)
40,000
6,000
222
208,550

1/2

1/2

$000
95,000
(80,000)
222
15,000
80,000
(50,000)
222
30,000

91

LSB_F7_Rev Kit:297mm x 210mm

28/10/09

13:37

Page 92

FINANCIAL REPORTING (INTERNATIONAL)


(W3) Finance lease

y/e 31 Mar 09
y/e 31 mar 10

b/f

Repayment

$000
80,000
67,200

c/f

$000
(20,000)
(20,000)

$000
60,000
47,200

Therefore:
Liability due > 1 year is $47,200
Liability due < 1 year is $20,000 (67,200 47,200)
(W4) Non-current assets

Interest
at 12%
$000
7,200
5,664

Property
$000
80,000
200,000
(80,000)
222
200,000

6,000
222
6,000
194,000

Cost b/f
Additions
Disposals

Cost c/f
Depreciation b/f
Depreciation charge (W5)
Depreciation c/f
Carrying value c/f

(W5) Depreciation charge

(c)

Property

Land
Heating system
Lifts
Building

Plant

Leased
Old plant

Not depreciated
20m / 10 years
30m / 15 years
100m / 50 years
80m / 5 years
(154,800-34,800) x 20%

c/f
$000
67,200
52,864

P&E
$000
154,800
80,000

222
234,800
34,800
40,000
222
74,800
160,000
$000
2,000
2,000
2,000
222
6,000
16,000
24,000
222
40,000

IAS 16 Property, Plant and Equipment requires that the depreciable value of all assets other than land
is written off over the useful life of the asset.

Depreciable value is calculated as cost less the expected residual value at the end of the assets useful
life.
This requirement holds even where:

A property is maintained to a high standard

The current market value of the property exceeds its cost

On this basis, the previous policy of Winger was inappropriate and the directors were correct to start
depreciating the new property

One potentially valid argument for non-depreciation is materiality: accounting standards are not
applicable to items which are not material. In the case of depreciation, however, this materiality level
refers to both the charge for the year and accumulated depreciation. It is likely that even where the
depreciation charge for the year is immaterial, accumulated depreciation will not be.

92

1 mark per
valid point

LSB_F7_Rev Kit:297mm x 210mm

28/10/09

13:37

Page 93

ANSWERS

9.

(a)

Petra

Income statement for Petra for the year ended 30 September 2009
Revenue (197,800 12,000 + 1,000 (W1))
Cost of sales (W2)
Gross profit
Distribution costs
Administration expenses

Profit before interest and tax


Finance costs (50m x 6%)
Profit before tax
Income tax ( W3)

(b)

Profit for the year

Statement of financial position of Petra at 30 September 2009

Non-current assets
Land and Buildings (W5)
Plant and equipment (W5)
Development costs (W6)
Current assets
Inventory
Receivables
Cash
Non-current assets held for sale (W4)
Total assets

Equity and liability


Ordinary share capital
Share premium
Retained earnings (34,000 + 13,100)
Non-current liabilities
6% redeemable loan
Deferred tax
Current liabilities
Payables
Income tax
Loan interest accrual ((50,000 x 6%) 1,500)
Total equity and liabilities

$000
331,450
(128,100)
222
58,700
(17,000)
(18,000)
222
23,700
(3,000)
222
20,700
(7,600)
222
13,100

Marks
2

See W2
1/2

1/2

1
See W3

$000
82,000
24,000
18,000
222
124,000
21,300
24,000
11,000
222
56,300
6,900
222
187,200
222
40,000
12,000
47,100
222
99,100
50,000
17,600
15,000
4,000
1,500
222
20,500
222
187,200
222

1/2

1/2

1/2
1/2
1/2

1/2

1/2

1/2

1
1

93

LSB_F7_Rev Kit:297mm x 210mm

28/10/09

13:37

Page 94

FINANCIAL REPORTING (INTERNATIONAL)


Workings

(W1) Commission sales


Revenue
Cost of sales
Profit

$000
12,000
11,000
2222
1,000

Deduct from revenue in IS


Deduct from cost of sales in
IS (being 8,000 + 3,000)
Include in revenue

Tutorial note

IAS 18 Revenues requires that only the commission element of commission sales is recorded as
income.
(W2) Cost of sales

$000
114,000
(8,000)
(3,000)
6,000
2,000
3,100
14,000
222
128,100

Draft per TB
Commission sales cost of sales
Commission sales remitted amount
Depreciation of plant (W5)
Depreciation of land and buildings (W5)
Loss on transfer of asset to held for sale (W4)
Impairment of development costs
(W3) Income tax charge

$000
4,000
1,000

Charge for the year


Underprovision from previous year
Movement in deferred tax provision
(17,600 15,000)

2,600
222
7,600

(W4) Non-current assets held for sale

$000
10,000
6,900
3,100

Carrying value of asset at transfer (16,000 6,000)


Fair value less costs to sell (7,500 x 92%)
Loss on transfer to held for sale

(W5) Non-current assets


Cost b/f
Disposals

Cost c/f
Depreciation b/f
Disposals
Depreciation charge
Depreciation c/f
Carrying value c/f

94

Depreciation charge L&B


Depreciation charge P & E

(100,000 40,000)/30yrs = 2,000


[(66-16) (26-6)] x 20% = 6,000

L&B
$000
100,000

222
100,000
16,000

2,000
222
18,000
82,000

P&E
$000
66,000
(16,000)
222
50,000
26,000
(6,000)
6,000
222
26,000
24,000

1/2
1/2
1/2
1/2
1/2

1/2

1/2
1/2

LSB_F7_Rev Kit:297mm x 210mm

28/10/09

13:37

Page 95

ANSWERS
(W6) Development costs
Cost b/f
Amortisation b/f
Impairment (

(c)

Carrying value

Basic EPS

Profits attributable to ordinary shareholders


22222222222222222
Number of ordinary shares

13,100,000
22222
160,000,000
Fully diluted EPS
Options

Proceeds
Number of shares at MV (7.2m/90c)
Therefore number of free shares ( )
Therefore DEPS:

$13,100,000
2222222222
(160,000,000 + 16,000,000)

$000
40,000
(8,000)
222
32,000
(14,000)
222
18,000

8.2c

$7,200,000
8,000,000
16,000,000
24,000,000
7.4c

10. Allgone
(a)

Income statement for Allgone for the year ended 31 March 2009

Revenue (236,200 8,000 (W1))


Cost of sales (W2)

Gross profit
Operating expenses (12,400 + 3,000 L&B depreciation W3)
Profit before interest and tax
Finance costs (W4)
Profit before tax
Income tax ( W5)

Profit for the year

Statement of comprehensive income for Allgone for the year ended


31 March 2009
Profit for the year
Surplus on revaluation of land and buildings (W6)
Loss on AFS investments
Total comprehensive income for the year

$000
228,200
(147,000)
222
81,200
(15,400)
222
65,800
(5,850)
222
59,950
(13,100)
222
46,850

$000
46,850
40,000
(1,200)
222
85,650

Marks
1/2

See W2
1/2

See W4
See W5

1/2

1/2

95

LSB_F7_Rev Kit:297mm x 210mm

28/10/09

13:37

Page 96

FINANCIAL REPORTING (INTERNATIONAL)


(b)

Statement of changes in equity for Allgone for the year ended 31 March 2009

At 1 April 2008
Prior period adjustment (W7)
Restated
Total comprehensive income
Transfer (W3)
At 31 March 2009

(c)

Share
capital
$000
60,000

222
60,000

222
60,000

Statement of financial position of Allgone at 31 March 2009


Non-current assets
Land and Buildings (W8)
Plant and equipment (W8)
Software (W8)
Available-for-sale investments (W9)
Current assets
Inventory (8,500 200 write down + 6,000 Funders)
Receivables
Total assets
Equity and liability
Ordinary share capital (SOCE)
Revaluation reserve (SOCE)
Retained earnings (SOCE)
Non-current liabilities
10% redeemable preference shares
12% loan notes
Deferred tax
Current liabilities
Bank
Payables
Income tax
Funders loan
Accruals (W4)
Total equity and liabilities

96

Retained
earnings
$000
4,350
(32,000)
222
(27,650)
46,850
1,000
222
20,200

Revaluation
reserve
$000
5,000
222
5,000
38,800
(1,000)
222
42,800

1/2

1
2
1

1/2

$000
127,000
48,000
2,000
10,800
222
187,800

14,300
23,000
222
37,300
222
225,100
222

1 1/2

60,000
42,800
20,200
222
123,000
20,000
40,000
4,800
350
15,200
11,300
8,000
2,450
222
37,300
222
225,100
222

1
1
1

1/2

1/2
1/2
1/2

1/2
1/2

1/2
1/2
1/2

1
1

LSB_F7_Rev Kit:297mm x 210mm

28/10/09

13:37

Page 97

ANSWERS
Workings

(W1) Funders Bank

The substance of the transaction with Funders Bank is that it is a loan of $8m secured on
goods.The $8m recorded as revenue should therefore be reversed and instead recorded as a
current liability.The facilitating fee should be treated as an accrued finance cost.

(W2) Cost of sales

$000
19,450
127,850
(8,300)
(6,000)
12,000
2,000
222
147,000

Opening inventory
Purchases
Closing inventory
Cost of goods re Funders
Depreciation of plant and equipment (W3)
Depreciation of software (W3)
(W3) Depreciation
Building

Plant and equipment


Software

(W4) Finance costs

105,000 / (40 5 yrs)


Historic cost depreciation: 80,000/40 years = 2,000
Therefore reserves transfer of $1,000 per annum
from revaluation reserve to retained earnings.
(84,300 24,300) x 20%
sum of digits
= n(n+1)/2
= 5 x 6/2
= 15
Therefore in year 3
3/15 x 10,000

Redeemable preference shares


(10% x 20m)
12% loan notes (12% x 40m x 9/12)
Funder Banks fee
(W5) Taxation
Charge for year
Movement on deferred tax
Provision reqd ($16m x 30%)
Provision per TB
Tax charge (IS)

Income statement
$000
2,000
3,600
250
222
5,850

Paid per TB
$000
1,000
(2,400)

1/2
1/2
1/2
1/2
1/2

$000
3,000

12,000

2,000
Accrual
$000
1,000
1,200
250
222
2,450
$000
11,300
4,800
(3,000)
222
1,800
222
13,100

1/2
1/2
1/2

1/2

1/2

97

LSB_F7_Rev Kit:297mm x 210mm

28/10/09

13:37

Page 98

FINANCIAL REPORTING (INTERNATIONAL)


(W6) Surplus on revaluation of land and buildings

Cost
Depreciation to revaluation
80,000 x 5/40yrs

CV at date of revaluation
Revalued amount
Revaluation surplus

(W7) Fraud

Land
$000
20,000

Buildings
$000
80,000

222
20,000
25,000
222
5,000

(10,000)
222
70,000
105,000
222
35,000

Total
$000

40,000

Although not highlighted until the current year, the fraud occurred in the previous financial
year. It must therefore be accounted for as a prior period adjustment through the statement
of changes in equity.

(W8) Non-current assets

Cost b/f
Revaluation

Cost c/f
Depreciation b/f
Revaluation
Charge for the year (W 3)
3,000
CV in SFP

(W9) Available for sale investments


Fair value at 1 April 2008
Fair value at 31 March 2009 (2.25/2.50 x 12m)

Land and
Plant and
buildings
equipment
$000
$000
100,000
84,300
30,000

222
222
130,000
84,300
10,000 (W6)
24,300
(10,000)

3,000
12,000
222
222
36,300
8,000
127,000
48,000

Fall in FV charged as other comprehensive income against previous


revaluation surplus of $5m

98

Software
$000
10,000

222
10,000
6,000

2,000
222
2,000
$000
12,000
10,800
222
1,200

LSB_F7_Rev Kit:297mm x 210mm

28/10/09

13:37

Page 99

ANSWERS

11. Tadeon
(a)

Statement of comprehensive income for Tadeon for the year ended


30 September 2009

Profit before tax


Income tax ( W5)

$000
277,800
(144,000)
222
133,800
2,000
(41,200)
(4,250)
222
90,350
(36,800)
222

Total comprehensive income for the year

53,550
20,000
(4,000)
222
69,550

Revenue
Cost of sales (W2)

Gross profit
Investment income
Operating expenses (40,000 + 1,200 operating lease rental)
Finance costs (2,750 (W3) + 1,500 (W4))
Profit for the year
Other comprehensive income
Surplus on revaluation (W6)
Deferred tax on revaluation surplus
(b)

Marks

Statement of financial position of Tadeon at 30 September 2009


Non-current assets
Leasehold property
Plant and equipment (W7)
Vehicles (W7)
Investments
Current assets
Inventory
Receivables
Total assets

1/2

See W2
1/2

1
2

See W5

1
1

$000
200,000
84,000
15,000
42,000
222
341,000

1/2

33,300
53,500
222
86,800
222
427,800
222

1/2

1
1

1/2

1/2

99

LSB_F7_Rev Kit:297mm x 210mm

28/10/09

13:37

Page 100

FINANCIAL REPORTING (INTERNATIONAL)


Equity and liability
Ordinary share capital (150,000 + 50,000 (W8))
Share premium (W8)
Revaluation reserve (20,000 4,000)
Retained earnings (18,600 + 53,550 30,000 dividend)

200,000
28,000
16,000
42,150
222
286,150

Non-current liabilities
2% Loan note (W3)
Deferred tax (20% x 74m)
Finance lease obligation (W4)
Current liabilities
Bank
Payables
Income tax
Finance lease obligation (W4)

51,750
14,800
10,500
1,900
18,700
38,000
6,000
222
64,600
222
427,800
222

Total equity and liabilities

1
1
1

1 1/2

1 1/2
1
1
1/2
1/2

1
1

Workings

(W1) Cost of sales

$000
118,000
9,000
12,000
5,000
222
144,000

Per TB
Depreciation of leasehold (W2)
Depreciation of plant and equipment (W2)
Depreciation of vehicles (W2)
(W2) Depreciation
Leasehold
Plant and equipment
Vehicles

(W3) Loan note

y/e 30 Sept 2009

(W4) Finance lease

y/e 30 Sept 2009


y/e 30 Sept 2010
Due > 1 year
Due < 1 year

100

225,000 / 25 years
(181,000 85,000) x 12 %
20,000 / 4 years
B/f
$000
50,000
b/f
$000
20,000
16,500

Interest at 5.5% Interest paid (TB)


$000
$000
2,750
(1,000)
Repayment
$000
(5,000)
(6,000)

(16,500 10,500)

c/fInterest at 10%
$000
$000
15,000
1,500
10,500
1,050
$10,500
$6,000

$000
9,000
12,000
5,000
c/f
$000
51,750
c/f
$000
16,500
11,550

1/2
1/2
1/2
1/2

LSB_F7_Rev Kit:297mm x 210mm

28/10/09

13:37

Page 101

ANSWERS
(W5) Tax charge
Tax charge for year
Deferred tax provision
@ 1 Oct 08
@ 30 Sept 09
(54m x 20%)

$000
38,000

1/2

12,000

10,800
222
(1,200)
222
36,800

Tutorial note

Of the taxable temporary differences of $74m, $20m relate to the revaluation of the leasehold
property. In accordance with IAS 12, deferred tax arising on an item of other comprehensive income
is dealt with as other comprehensive income.Therefore the tax charge in the income statement
includes only the deferred tax relating to the other taxable temporary differences.

(W6) Revaluation surplus

$000

Valuation
Carrying value
Cost
Dep to 1.10.08
Dep for yr (W2)

225,000
(36,000)
(9,000)
222

Surplus on revaluation
(W7) Plant and equipment and vehicles
Cost b/f
Additions

Cost c/f
Depreciation b/f
Charge (W2)

Depreciation c/f
CV at 30 Sept 09

P&E
$000
181,000

222
181,000
85,000
12,000
222
97,000
84,000

(W8) Suspense account


Per TB
Dividend
Rights issue
Nominal value
Premium

(5% x 80c x 150m/0.2)


(150m/3)
[(150m/0.2)/3 x 12c] issue costs of 2,000

$000
200,000

(180,000)
222
20,000
Vehicles
$000

20,000
222
20,000

5,000
222
5,000
15,000
$000
48,000
30,000
(50,000)
(28,000)
222
nil

101

LSB_F7_Rev Kit:297mm x 210mm

28/10/09

13:37

Page 102

FINANCIAL REPORTING (INTERNATIONAL)

12. Kala
(a)

Statement of comprehensive income for Kala for the year ended 31 March 2009

Profit before tax


Income tax ( 28,300 + (14,100 12,500))

$000
278,400
(115,700)
222
162,700
10,800
(15,500)
(10,000)
222
148,000
(29,900)
222

Total comprehensive income for the year

118,100
45,000
222
163,100

Revenue
Cost of sales (W1)

Gross profit
Investment income (4,500 + 6,300 gain on invt property)
Operating expenses
Finance costs (3,000 (W3) + 7,000 (W4))
Profit for the year
Other comprehensive income
Surplus on revaluation (W6)

(b)

Balance at 31 March 2009

Share
capital
$000
150,000

222
150,000

Statement of financial position of Kala at 31 March 2009


Non-current assets
Land and Buildings (W6)
Plant and equipment (W5)
Investment Property (90,000 x 1.07)
Current assets
Inventory
Receivables
Total assets

102

1/2

See W2
1

1/2

1 1/2

1 1/2

Statement of changes in equity for Kala for the year ended 31 March 2009

Balance at 1 April 2008


Total comprehensive income
Dividends
(c)

Marks

Retained
earnings
$000
119,500
118,100
(15,000)
222
222,600

Revaluation
surplus
$000

45,000
222
45,000

1
1
1
1

$000
250,000
184,100
96,300
222
530,400
43,200
53,200
222
96,400
222
626,800
222

1
1
1

1/2
1/2

LSB_F7_Rev Kit:297mm x 210mm

28/10/09

13:37

Page 103

ANSWERS
Equity and liability
Ordinary share capital
Revaluation reserve (SOCE)
Retained earnings (SOCE)

150,000
45,000
222,600
222
417,600

Non-current liabilities
8% Loan note
Deferred tax
Finance lease obligation (W4)
Current liabilities
Bank
Payables
Income tax
Finance lease obligation (W4)
Accruals

50,000
14,100
55,000
5,400
33,400
28,300
22,000
1,000
222
90,100
222
626,800
222

Total equity and liabilities

1/2

1/2
1/2

1/2

1
1

1/2

1/2
1/2

1/2

Workings

(W1) Cost of sales

$000
37,800
78,200
(43,200)
18,400
19,500
5,000
222
115,700

Opening inventory
Purchases
Closing inventory
Depreciation of leased plant (W2)
Depreciation of owned plant (W2)
Depreciation of land and buildings (W2)
(W2) Depreciation
Leased plant
Owned plant

92,000/5 years
(156,000 26,000) x 15%

Land and buildings

175m/(50 15) years

(W3) Finance costs loan notes


8% x 50,000 x 9/12
(W4) Finance lease
y/e 30 March 09
y/e 30 March 10
Due > 1 year
Due < 1 year

b/f
$000
92,000
77,000

Income statement
$000
3,000
Repayment
$000
(22,000)
(22,000)

(77,000 55,000)

$000
18,400
19,500
222
37,900
5,000
Paid per TB
$000
(2,000)
c/fInterest at 10%
$000
$000
70,000
7,000
55,000
5,500

Accrual
$000
1,000
c/f
$000
77,000
60,500
$55,000
$22,000

103

LSB_F7_Rev Kit:297mm x 210mm

28/10/09

13:37

Page 104

FINANCIAL REPORTING (INTERNATIONAL)


(W5) Plant and equipment

$000
156,000
92,000
222
248,000
(26,000
222
184,100

B/f at 1 April 08
Additions (lease)
Depreciation b/f

Depreciation charge (W2) (37,900)


(W6) Land and buildings
Historic cost
Depreciation to 1 April 2008
Revaluation surplus (

Valuation at 1 April 2008


Depreciation for the year (W2)
CV at 31 March 2009

Land
$000
70,000

70,000
10,000
222
80,000

222
80,000

Buildings
$000
200,000
(60,000)
140,000
35,000
222
175,000
(5,000)
222
170,000

Total

45,000
250,000

13. Telenorth
(a)

i) Income statement for Telenorth for the year ended 30 September 2009
Revenue
Cost of sales (W1)

Gross profit
Investment income
Distribution expenses
Administration expenses (34,440 + 10,160 (W3) 2,400 (W5))
Finance costs (W4)
Profit before tax
Income tax ( 23,400 + 1,200)
Profit for the year

104

$000
283,460
(155,170)
222
128,290
1,500
(22,300)
(42,200)
(1,656)
222
63,634
(24,600)
222
39,034

Marks
1/2

See W1
1/2
1/2

1 1/2
1 1/2
1

LSB_F7_Rev Kit:297mm x 210mm

28/10/09

13:37

Page 105

ANSWERS
ii) Statement of financial position of Telenorth at 30 September 2009
Non-current assets
Land and Buildings (W6)
Plant and equipment (W6)
Computer system (W6)
Investments
Current assets
Inventory (W2)
Trade accounts receivable (35,700 + 12,000 (W5))
Total assets

Equity and liability


Ordinary share capital (20,000 + 10,000 (W7))
Share premium (W7)
Revaluation reserve (3,400 1,000)
Retained earnings (14,160 + 39,034 2,000 ord divi)
Non-current liabilities
6% Loan note
Deferred tax (5,200 + 2,200)
8% redeemable preference shares
Current liabilities
Bank
Trade accounts payable
Income tax
Loan (W5)
Accruals (W4)
Total equity and liabilities

$000
36,000
32,200
15,240
34,500
222
117,940
16,680
47,700
222
64,380
222
182,320
222
30,000
16,000
2,400
51,194
222
99,594
10,000
7,400
12,000
1,680
17,770
23,400
9,600
876
222
53,326
222
182,320
222

1/2

1/2
1/2
1/2

1
1

1/2

1/2

1/2

1/2
1/2
1/2
1/2
1/2

1
1

Workings

(W1) Cost of sales


Opening inventory
Purchases
Closing inventory (W2)
Depreciation of buildings (W3)
Depreciation of plant and equipment (W3)

$000
12,400
147,200
(16,680)
2,250
10,000
222
155,170

1/2

1/2
1/2
1/2
1/2

105

LSB_F7_Rev Kit:297mm x 210mm

28/10/09

13:37

Page 106

FINANCIAL REPORTING (INTERNATIONAL)


(W2) Closing inventory

$000
16,000

At 4 October 2009
Add back:
Normal sales 1,400 x 100/140
Sale or return sales 650 x 100/130
Deduct:
Purchases

1,000
500
(820)
222
16,680

(W3) Depreciation
P&E
Computer system
Land and buildings

(W4) Finance costs

(55,000 5,000) / 5 years


(35,000 9,600) x 40%
56,250/25years

Income statement
$000
Loan notes 6% x 10,000
600
Redeemable preference shares 8% x 12,000
960
Factoring fee (W5)
96
222
1,656

(W5) Kwikfinance

Entries recorded
Entry should have been
Therefore correcting entries

Paid per TB
$000
(300)
(480)

Dr
Dr
Cr
Dr
Cr
Dr
Cr
Cr

Admin
Cash (80% x 12,000)
Receivables
Cash
Loan
Receivables
Admin expense
Loan

Dr
Cr

Finance costs (9,600 x 1%)


Accrual

Interest on the loan arrangement should also be recorded:

Tutorial note

$000
10,000
10,160
2,250
Accrual
$000
300
480
96
222
876
2,400
9,600
12,000
9,600
9,600
12,000
2,400
9,600
96
96

The substance of this arrangement is that Kwikfinance has made a loan to Telenorth, secured on its
accounts receivables book.The risks and rewards of the receivables have not been transferred to
Kwikfinance as any accounts unpaid after 4 months are transferred back to Telenorth. It is therefore
incorrect to derecognise the receivables.

106

LSB_F7_Rev Kit:297mm x 210mm

28/10/09

13:37

Page 107

ANSWERS
(W6) Non-current assets
Building
$000
56,250
(18,000)
(2,250)
222
36,000

Cost b/f
Depreciation b/f
Depreciation for the year
CV

(W7) Suspense and share issues

Share capital
$000

Per TB
Options
NV 4m x $1
Premium 4m x $1
Rights issue
NV 6m x $1
Premium 6m x $2
(b)

Earnings per share

Basic EPS

Plant and
equipment
$000
55,000
(12,800)
(10,000)
222
32,200
Share premium
$000

4,000
6,000
222
10,000

Computer
system
$000
35,000
(9,600)
(10,160)
222
15,240
Suspense
$000
26,000

4,000

(4,000)
(4,000)

12,000
222
16,000

(6,000)
(12,000)
222
nil

Profits attributable to ordinary shareholders


22222222222222222
Number of ordinary shares in issue

Number of ordinary shares in issue


Date
1 Oct X0 30 June 09
1 July X1 30 Sept 09
Therefore Basic EPS
Working

Bonus fraction
TERP

So bonus fraction is

Number of
shares in issue
24,000
30,000

Bonus
fraction
X 4/3.80 (W)

Time
apportionment
X 9/12
X 3/12

$39,034,000
22222
26,447,000
MV pre rights
22222
TERP

old shares
new shares

4 x $4
1 x $3
2
5
$19/5 gives TERP of $3.80
$4.00
22
$3.80

Weighted
average shares
18,947
7,500
222
26,447
= 148c

3
1
1 for profit
from IS

16
3
2
19

107

LSB_F7_Rev Kit:297mm x 210mm

28/10/09

13:37

Page 108

FINANCIAL REPORTING (INTERNATIONAL)

14. Tourmalet
(a)

Sale of plant

Tourmalet has sold an item of plant during the year and then leased it back for the remainder of its
useful life. The period of the lease indicates that this is a finance lease.

1 mark per
valid point

As Tourmalet will continue to use the asset for the remainder of its useful life, it has not relinquished
the risks and rewards of ownership of the asset and should therefore not derecognise it.

The commercial substance of the sale and leaseback transaction is in fact that of a financing
arrangement, whereby Tourmalet have been loaned $50million secured on the item of plant.

This transaction should therefore not be accounted for as a sale and separate leaseback as the
directors propose. Instead:

(b)

The asset should continue to be held in the statement of financial position at book value of
$40million (less subsequent depreciation)

The proceeds received of $50million should be recorded as a loan in the statement of financial
position

Interest arising at 12% per annum should be recorded as a finance cost in the income statement

Income statement for Tourmalet for the year ended 30 September 2009

$000
247,800
(128,800)
222
119,000
1,200
(26,400)
(20,000)
(200)

Revenue (313,000 50,000 15,200 discontd ops)


Cost of sales (W1)

Gross profit
Investment income
Distribution expenses
Administration expenses (23,200 3,200 discontd ops)
Other expenses (10,000 9,800 investment property)
Finance costs
Redeemable preference dividend (6% x 30,000)
Loan (12% x 50,000 x 6/12)
Profit before tax
Income tax (9,200 2,100 overprovision)

Profit for the year from continuing activities


Loss for year from discontinued operations (W4)
Profit for the year

108

1,800
2,000
222

(3,800)
222
69,800
(7,100)
222
62,700
(5,500)
222
57,200

Marks
2

See W1
1/2
1/2

1
1

1
1
1
See W5

LSB_F7_Rev Kit:297mm x 210mm

28/10/09

13:37

Page 109

ANSWERS
Workings

(W1) Cost of sales

$000
26,550
158,450
(26,000)
(40,000)
(16,000)
8,000
14,800
3,000
222
128,800

Opening inventory
Purchases
Closing inventory (W2)
Cost of plant (sale and leaseback transaction)
Costs relating to discontinued operations
Depreciation of leased plant (W3)
Depreciation of owned plant (W3)
Depreciation of buildings (W3)
(W2) Closing inventory

(W3) Depreciation

1
1

1/2

1
1
1

$000
8,000
14,800
3,000

40m / 5 years
(98,600 24,600) x 20%
(150,000 30,000)/40 years

(W4) Discontinued operations

$000
15,200
(16,000)
(3,200)
(1,500)
222
(5,500)

Revenue
Cost of sales
Operating expenses
Termination of lease penalty
(c)

1/2

$000
28,500
(2,500)
222
26,000

Closing inventory
Write down for slow moving goods

Leased plant
Other plant
Buildings

1/2

1/2

1/2
1/2

Statement of changes in equity for Tourmalet for the year ended 30 September
2009

At 1 October 2008
Total comprehensive income
Transfer
Dividend
At 30 Sept 2009

Share
capital
$000
50,000

222
50,000

Retained
earnings
$000
47,800
57,200
500
(2,500)
222
103,000

Revaluation
reserve
$000
18,500

(500)
222
18,000

Total
$000
116,300
57,200
222
171,000

1/2

1/2
1/2

1/2

109

LSB_F7_Rev Kit:297mm x 210mm

28/10/09

13:37

Page 110

FINANCIAL REPORTING (INTERNATIONAL)

15. Wellmay

Marks

Statement of comprehensive income for Wellmay for the year ended


31 March 2009
Revenue (4,200 500 (W1))
Cost of sales (2,700 200 (W1) 75 (W2) 8 (W3))
Gross profit
Investment income
Operating expenses (470 + 8 (W3))
Loss on investment property (W3)
Finance costs (55 + 50 (W1) 48 + 56 (W4))
Profit before tax
Income tax ( 360 + 30 (W5))

Profit for the year


Surplus on revaluation of factory (W3)
Total comprehensive income

$000
3,700
(2,417)
222
1,283
20
(478)
(25)
(113)
222
687
(390)
222
297
190
222
487

1
2
1/2

1
1

1 1/2
1
1

Statement of changes in Equity for Wellmay for the year ended 31 March 2009

b/f
Total comprehensive income
Bonus issue
Loan note issue (W4)
Dividends

Share capital
$000
1,200

c/f

300
222
1,500

Equity
in loan
$000

40

222
40

Statement of financial position of Wellmay at 31 March 2009


Non-current assets
Property , plant and equipment (W3)
Investment property
Current assets (1,400 + 200 (W1))
Total assets

Equity and liability


Ordinary share capital
Equity in loan
Retained earnings
Revaluation reserve
Non-current liabilities
8% convertible loan note (W4)
Deferred tax (W5)
Current liabilities (820 + 550 (W1) 75 (W2)
Total equity and liabilities

110

Retained
earnings
$000
2,615
297
(300)

(400)
222
2,212

Revaluation
reserve
$000
350
190

222
540

Total
$000
4,165
487
40
(400)
222
4,292

1/2

1
1
1

1/2
1/2

$000
4,390
375
222
4,765
1,600
222
6,365
222
1,500
40
2,212
540
222
4,292
568
210
1,295
222
6,365
222

2
1
1

1/2

1/2
1/2

2
1
2

LSB_F7_Rev Kit:297mm x 210mm

28/10/09

13:37

Page 111

ANSWERS
Workings

(W1) Sale and repurchase

This transaction is a financing arrangement in substance.The $500,000 advance to Wellmay is a


loan secured on the maturing goods. No sale should be recorded and instead:

A loan (current) of $500,000 is recorded at 1 April 2008

In the year to 31 March 2009 interest of $50,000 ($500,000 x 10%) is charged to profits
and increases the loan capital outstanding to $550,000

To reverse the sale already recorded:

Deduct $500,000 from revenue

Deduct $200,000 from cost of sales

Add $200,000 to closing inventory

(W2) Contingency provision

This provision does not meet the recognition criteria of IAS 37 and must therefore be
reversed by deducting $75,000 from current liabilities and cost of sales.

(W3) Properties

Owner-occupied property

(i) revaluation
Valuation at 31 March 2008
Depreciation
Revaluation gain (

Valuation at 31 March 2009


(ii) depreciation adjustment
$40,000 x 20%
deduct from cost of sales
add to operating expenses
Investment property
Valuation at 31 March 2008
Fall in value
Valuation at 31 March 2009

$000
1,200
(40)
222
222
1,160
190
222
1,350
$000
8,000

$000
400
(25)
222
222
375

111

LSB_F7_Rev Kit:297mm x 210mm

28/10/09

13:37

Page 112

FINANCIAL REPORTING (INTERNATIONAL)


(W4) Convertible loan note

$000
600
$000
43.6
39.8
36.0
440.6
222

Proceeds
PV of equivalent redeemable loan note:
09: (600,000 x 8%) x 0.91
10: (600,000 x 8%) x 0.83
11: (600,000 x 8%) x 0.75
12: [600,000 + (600,000 x 8%)] x 0.68
Debt element of loan notes

Equity element of loan notes


Carrying value of liability
y/e 31.3.09

b/f
$
560,000

Int at 10%
$
56,000

Paid
$
(48,000)

(W5) Deferred tax


Deferred tax provision required (600,000 x 35%)
Provision b/f

Charge to IS

(W6) Bonus issue


Number of shares in issue 1,200,000/0.50
Bonus shares 2,400,000/4
Dr Retained earnings
Cr Share capital

2,400,000
600,000
600,000 x 50c

(560)
222
40
c/f
$
568,000
$000
210
(180)
222
30
$000
300
300

16. Peterlee
(a)

Purpose and status of the Framework

Purpose

The Framework provides the underlying concepts and principles on which accounting standards are
based.To this end it includes, for example, explanations of the quantitative characteristics of financial
statements (relevance, reliability, understandability and comparability) and definitions of an asset and
liability.

The purpose of the framework is to:

Provide principles-based guidance where an accounting standard does not deal with a transaction

Assist users in the interpretation of financial statements

112

Assist in the development of future accounting standards

Assist auditors in forming their opinion as to whether financial statements conform to accounting
standards
Provide a basis for reducing the number of alternative accounting treatments permitted by standards
Assist national standard-setters in the development of national accounting standards.

1 per point
to max of
3

LSB_F7_Rev Kit:297mm x 210mm

28/10/09

13:37

Page 113

ANSWERS
Status

(b)

The Framework is not an accounting standard, nor do its contents override any accounting standard.
Where there is a conflict between an accounting standard and the Framework, the accounting
standard prevails.

An asset is defined as a resource controlled by an entity as a result of past events and from which
future economic benefits are expected to flow to the entity.

Assets and liabilities: definitions and recognition criteria

A liability is defined as a present obligation of the entity arising from past events, the settlement of
which is expected to result in an outflow from the entity of resources embodying economic benefits.

Assets

The application of the definition of an asset ensures that all quantifiable items which provide a value
or economic benefits to an entity are included in the statement of financial position, but items which
will not provide economic benefits are not.

The definition does not require assets to be owned, but rather controlled by an entity. For this reason,
assets held under a finance lease are reported as assets, since they provide clear future economic
benefits.
The definition requires that economic benefits are expected to flow to the entity.Thus, for example, a
contingent or uncertain asset is not included in the statement of financial position unless an economic
inflow is virtually certain.

The requirement for an asset to arise from a past event precludes the inclusion of future expected
assets such as receivables from future sales in the statement of financial position, and so reduces the
potential for overstatement of assets and manipulation of accounts.
Liabilities

Again, the definition of a liability ensures that all known quantifiable liabilities are included within the
statement of financial position, however liabilities cannot be created in order to manipulate the
accounts.

A liability is a present obligation as the result of a past event. In other words, an event has already
occurred which gives rise to an obligation to transfer payment.Thus future expected liabilities are not
included in the financial statements.

The definition requires an expected outflow of economic resources. In the case of provisions
accounting, expected is defined as more likely than not. Thus provisions without purpose are
prohibited and so can not be used to manipulate profits, as was the case historically with big bath
accounting.

In conclusion, the definitions of both assets and liabilities ensure that the statement of financial
position provides reliable and complete data and so is useful to the users of the financial statements.

17. Derringdo
(a)

Recognition of income

The Framework defines income as revenue and gains and states that it should be recognised in the
income statement when an increase in an asset or decrease in a liability that can be measured reliably
has arisen.

Thus, the framework includes all gains, whether realised or unrealised within its definition of income.
The revision to IAS 1 in 2007 applied this element of the framework by extending the income
statement to include other comprehensive income, being unrealised income e.g. a property revaluation.

1 for
discussion
of control
1 for
discussion
of
economic
benefits
1 for
discussion
of past
events
1
1 for
discussion
of present
obligation
and past
event
1 for
discussion
of
expected
outflow
1

Max 8
marks for
part (b)

Marks
1

113

LSB_F7_Rev Kit:297mm x 210mm

28/10/09

13:37

Page 114

FINANCIAL REPORTING (INTERNATIONAL)


IAS 18 Revenue on the other hand concentrates only on realised amounts, stating that revenue
includes only economic benefits received and receivable. It requires that revenue is recognised in
profit or loss.

This traditional approach to recording only realised amounts as income in profit or loss is in conflict
with certain newer accounting standards. For example:

IAS 39 requires that changes in the fair value of financial instruments categorised as fair value
through profit and loss are recognised in profit or loss.

IAS 49 requires that changes in the fair value of investment properties are recognised in profit or
loss.

IAS 18 Revenue applies the concept of substance over form by requiring that revenue is recorded
when the significant risks and rewards of ownership of goods have been transferred to the purchaser.
An application of this requirement to a sale and repurchase agreement means that no revenue is
recognised. In substance such an agreement is a provision of finance and the sales proceeds are
recognised as a loan.
(b)

Gungho

Extract of income statement for Derringdo for y/e 31 March 2009


Revenue
Grade A (W1)
Grade B (W2)

$000
4,600
11,400
222
16,000
(8,550)
222
7,450

Cost of sales (W2)


Gross profit

Workings

(W1) Grade A

Derringdo acts as an agent for the sale of Grade A goods and therefore should record only its
sales commission as revenue.

18,400 x 100/50
38,800 x 12.5%

$000
2,400
18,000
(2,000)
222
18,400
36,800
4,600

8,550 x 100/75

$000
1,000
8,800
(1,250)
222
8,550
11,400

Opening inventory
From Gungho
Closing inventory

Cost of goods sold


Selling price
Commission
(W2) Grade B
Opening inventory
From Gungho
Closing inventory
Cost of sales
Revenue

114

1
1
1
1
Max 6
marks for
part (a)

LSB_F7_Rev Kit:297mm x 210mm

28/10/09

13:37

Page 115

ANSWERS
(c)

(i) Government grant

In line with the requirements of IAS 20 Accounting for Government Grants and Disclosure of Government
Assistance, Derringdo has treated the $240,000 ($800,000 x 30%) grant received as a deferred credit,
in other words a liability, in the statement of financial position.

According to the Framework, a liability is a present obligation arising from a past event, the settlement
of which is expected to result in an outflow of economic benefits.
The grant does not meet this definition in the following respects:

The grant will only result in an outflow of economic benefits if the asset for which it was awarded
is sold within 4 years of its purchase

By the year end the asset has not been sold and so there is no past event or present obligation

This is an example of an accounting standard conflicting with the Framework. In these instances, the
requirements of the standard override those of the Framework.

(ii)Extracts from Derringdos financial statements for the year ended 31 March
2009
Applying company policy

Income statement
Depreciation charge ((800 x 85%)/10 years x 6/12m)
Release of grant (240,000/10 years x 6/12m)
Statement of financial position
Property, plant and equipment (800 34)
Current liabilities
Deferred income (240,000/10)
Non-current liabilities
Deferred income (240 12 released 12 current)

$000
(34)
12

1
1

$000
766

24

204

$000
(34)
240

In compliance with Framework

Income statement

Depreciation charge ( as before)


Grant income
Statement of financial position
Property, plant and equipment (as before)

$000
766

Tutorial note

As the grant does not meet the definition of a liability, the Framework would require that it is
recognised as income when received.

115

LSB_F7_Rev Kit:297mm x 210mm

28/10/09

13:37

Page 116

FINANCIAL REPORTING (INTERNATIONAL)


(d)

Sale of carpets

A change in accounting policy occurs if there has been a change in:

Recognition (for example an item previously recognised as an asset is now recognised as an


expense)

Presentation, or

Measurement basis.

In this case Derringdo has changed when revenue is recorded. It has not, however, changed the
recognition, presentation or measurement basis of revenue.

Therefore this does not constitute a change in accounting policy, and the change should be accounted
for prospectively i.e. from the date of change.
Income statement for Derringdo for year ended 31 March 2009
Revenue
Sales before 1 April 2008
Sales after 1 April 2008
Tutorial note

1
1

$000
1,200
23,000
222
24,200

1
1

Sales before the change on 1 April 2008 should be recorded when the carpets are fitted.Therefore all
carpets fitted within the first 14 days of the financial year form part of the years revenue.
Sales after the change on 1 April 2008 should be recorded at the date of order and payment,
regardless of when they are fitted. Therefore the carpets fitted in the first 14 days of April 2009 do
form part of the revenue for the year ended 31 March 09, as they are included in sales made in retail
outlets in this period.

18. Broadoak
(a)

Measurement of non-current assets

(i) IAS 16 requires that property, plant and equipment is measured initially at cost.
Cost includes:

Purchase price (after trade discounts)

Direct costs in bringing the asset to the location and condition necessary for normal
operation, such as:

Import duties

Costs of site preparation


Delivery costs
Installation costs
Testing costs
Professional fees

In the case of self-constructed assets, direct costs will also include labour costs.

116

An initial estimate of future dismantling costs where there is an obligation to dismantle the
asset after use.

1/2 per
example to
max of 2

LSB_F7_Rev Kit:297mm x 210mm

28/10/09

13:37

Page 117

ANSWERS
The following costs may not be capitalised as part of property, plant and equipment:

A proportion of administration costs

Abnormal costs such as those arising from planning error or strike action

(ii) Subsequent expenditure on property, plant and equipment may only be capitalised where it meets
the definition of an asset. In other words the expenditure enhances the economic benefits
expected to flow from the asset.This may be the result of:

An extension of useful life

Reduced operating costs

Increased quality of output

IAS 16 expressly states that subsequent expenditure on day to day servicing and repairs and
maintenance does not meet this definition and so must be expensed to profit or loss.

An example of subsequent expenditure which does meet the definition of an asset and may be
capitalised is an extension to a property which enhances future economic benefits by providing
increased operational capacity.

IAS 16 also considers complex assets and major inspections:

(b)

Complex assets are those which are comprised of a number of different parts, each
depreciated separately. Where one of the parts reaches the end of its useful life and is
replaced, the replacement cost may be capitalised.

Certain assets, such as an aircraft, require major inspections periodically. IAS 16 allows the
cost of these to be capitalised and depreciated to the next inspection.

Revaluation surpluses and deficits

1
Max 3 for
part (a) (ii)

IAS 16 permits either the cost model or the revaluation model to be applied to non-current assets.

Under the revaluation model, assets whose fair value can be measured reliably are carried at their fair
value at the date of revaluation less any subsequent depreciation and impairment losses.

Where the revaluation model is applied:

It must be applied consistently to an entire class of property, plant and equipment;

Revaluations should be made sufficiently regularly such that the carrying amount of an asset does
not differ materially from fair value.

Determination of the fair value of assets is normally undertaken by a professionally qualified valuer.
Accounting for a revaluation surplus

1
1

A revaluation surplus arises where the fair value of an asset exceeds its carrying amount. Its treatment
depends on whether the surplus reverses a previous revaluation deficit:

Where a revaluation surplus does not reverse a previous deficit, it is unrealised and therefore is
not recognised in profit or loss. Rather, it is recognised as other comprehensive income in the
statement of comprehensive income and accumulated in a revaluation surplus within equity.

An increase which reverses a previous revaluation deficit recognised in profit or loss is itself
recognised in profit or loss.

Accounting for a revaluation deficit

A revaluation deficit arises where the fair value of an asset is less than its carrying amount.Again, the
treatment depends on whether the deficit reverses a previous revaluation surplus:

A deficit which does not reverse a previous revaluation surplus is recognised in profit or loss.

117

LSB_F7_Rev Kit:297mm x 210mm

28/10/09

13:37

Page 118

FINANCIAL REPORTING (INTERNATIONAL)

A deficit which reverses a previous revaluation surplus recognised as other comprehensive income
and accumulated in a revaluation surplus in equity is itself recognised in the same way.That is, the
deficit is presented as an expense within other comprehensive income and debited to the
revaluation surplus.

The fair value of revalued assets is depreciated over the assets remaining useful life.Where this
results in an increased depreciation charge compared to that based on historical cost, the extra
amount of depreciation is transferred annually from the revaluation reserve to retained earnings.

When a revalued asset is disposed of, profit recognised in the income statement is calculated as the
difference between sales proceeds and carrying value at the date of disposal. Any surplus in the
revaluation reserve relating to the asset is transferred to retained earnings by way of a reserves
transfer.

(c)

(i) Measurement of plant

$
240,000
(30,000)
222
210,000
2,750
12,500

List price
Trade discount (12.5% x 240,000)
Shipping and handling costs
Estimated pre-production testing
Site preparation costs:
Cabling (14,000 6,000 abnormal costs)
Concrete reinforcement
Own labour costs
Dismantling and restoration costs (15,000 + 3,000)

8,000
4,500
7,500
18,000
222
263,250

Tutorial note

1
1
1
1
Max 8 for
part (b)

1/2
1/2

1/2
1/2

Early settlement discounts are not taken into account when determining the cost of an asset only
trade discounts.
(ii)Revaluation of property lease

Statement of financial position of Broadoak at 30 September

Property, plant and equipment (W1)


Revaluation surplus (W2)

2008
$
231,000
61,000

Statement of comprehensive income of Broadoak for the year ended 30 September


2008
$
Depreciation (W1)
20,000
Revaluation deficit (35,000 10 (W2))

Other comprehensive income


Revaluation surplus (W2)
11,000

118

2009
$
175,000
50,000
2009
$
21,000
25,000
(10,000)

1
1

1
1
1

LSB_F7_Rev Kit:297mm x 210mm

28/10/09

13:37

Page 119

ANSWERS
(W1)

$
240,000
(20,000)
222
220,000
11,000
222
231,000
(21,000)
222
210,000
(35,000)
222
175,000

Cost (1.10.07)
Amortisation (240,000/12 years)
CV (30.9.08)
Revaluation surplus (

Valuation (30.9.08)
Amortisation (231,000/11 years)
CV (30.9.09)
Revaluation deficit (
Valuation (30.9.09)

(W2) Revaluation reserve


Brought forward
Surplus in 2008 (W1)

At 30 Sept 2008
Excess depreciation transfer (21,000 20,000)
Revaluation deficit

Old
$
50,000

222
50,000

222
50,000

New
$

11,000
222
11,000
(1,000)
(10,000)
222

Total
$
50,000
11,000
222
61,000
(1,000)
(10,000)
222
50,000

Remaining 2009 revaluation deficit of $25,000 is charged to profits.

19. Wilderness
(a)

Impairment losses

(i) An impairment loss is defined by IAS 36 as the amount by which the carrying amount of an asset
exceeds its recoverable amount.
The recoverable amount of an asset is the higher of its fair value less costs to sell and its value in
use, in other words its value to the business.
Fair value less costs to sell is the amount which could be obtained from the sale of an asset in an
arms length transaction between willing parties less the incidental costs of disposal.
Value in use is the present value of the future cash flows expected to be derived from an asset,
including those on its ultimate disposal. This is a subjective measure involving estimates of future
cash flows and appropriate discount rates.

An impairment test involves assessing the recoverable amount of an asset and comparing it to
carrying value.Where this can not be assessed for an individual asset, then the cash-generating unit
to which an asset belongs is tested for impairment.

An impairment test is required for all assets or cash-generating units when there is an indication
of impairment at the reporting date.

1
1
1
1

In addition IAS 36 requires certain assets to be tested for impairment annually.These are:

Goodwill acquired in a business combination

Intangible assets which are not yet available for use.

Intangible assets with an indefinite useful life

119

LSB_F7_Rev Kit:297mm x 210mm

28/10/09

13:37

Page 120

FINANCIAL REPORTING (INTERNATIONAL)


(ii)An impaired asset or cash-generating unit must be written down to its recoverable amount,
normally by crediting accumulated depreciation .The corresponding debit entry will depend on
whether the asset has been revalued upwards in the past.

Impairment losses on non-revalued assets are recognised in profit or loss

1. directly against any revaluation surplus to the extent that it relates to the asset which is
impaired, and then

Impairment losses on revalued assets are recognised:

2. in profit or loss.

Where a cash-generating unit is written down to its recoverable amount, assets are written down
in the following order:
1. goodwill

2. other assets on a pro-rata basis

No asset is written down below its recoverable amount.

(b)

Future depreciation charges are based on the impaired value of the asset spread over its remaining
useful life.

Wilderness impairments

(i) Statement of financial position of Wilderness as at 30 September 2009

$
112,500

Plant (W)

Income statement for Wilderness for the year ended 30 September 2009
Depreciation
1 Oct 08 31 March 09 (W)
1 April 09 30 Sept 09 (W)
Impairment
Working

Cost of plant
Depreciation to start of year

CV at start of year
Depreciation to accident (640,000 x 12.5% x 6/12m)

CV at 1 April 09
Impairment loss (

Recoverable amount higher of:


Value in use
150,000
Net realisable value
20,000
Depreciation for second half of year (150,000 /4)
CV at 30 Sept 09

Notes

120

The asset is depreciated on the old basis up until the date of the accident

40,000
37,500
50,000
$
640,000
(400,000)
222
240,000
(40,000)
222
200,000
(50,000)
222
150,000
(37,500)
222
112,500

On the date of the accident there is an obvious impairment, and recoverable amount is
assessed

LSB_F7_Rev Kit:297mm x 210mm

28/10/09

13:37

Page 121

ANSWERS

The trade in value is irrelevant since Wilderness do not intend to take advantage of the trade
in offer.Therefore the fair value less costs to sell is the lower amount of $20,000

Value in use is higher than fair value less costs to sell and therefore this is recoverable amount

The difference between the carrying value on the date of the accident and value in use is the
impairment loss, charged to profits
The impaired value is depreciated over the remaining useful life of two years.

(ii) Before considering an impairment of Mossel as a cash generating unit, impairment of the individual
assets should be considered:
Carrying value
$000
7,000
12,000
8,000
5,000
222

Brand
Land
Plant
Inventories

Impairment
$000
(7,000)

222

Brand

Impaired value
$000

12,000
8,000
5,000
222
25,000

The brand name Quencher is no longer used and so will not lead to economic benefits.Therefore it
does not meet the definition of an asset and must be written off in full.

The new brand name is internally generated and therefore may not be capitalised according to IAS 38.

Plant

The directors believe that it is necessary to spend money on upgrading the bottling plant, however
there is no indication that the plant is impaired on an individual basis at this time. Furthermore the
future expenditure is not provided for on the basis that there is no obligation.

1
1

Inventories

Inventories should be held at the lower of cost and net realisable value. The NRV of the bottles of
Quencher is $2.75 million ((2m x 150% )- 250,000).This is higher than cost of $2million and therefore
no adjustment is needed.

Subsequent to the assessment of individual assets, the carrying value of the CGU is $25million.
The value in use of the CGU is $20 million, indicating an impairment loss of $5million.
This loss is prorated between the land and plant as follows:

Land
Plant

CV
12,000
8,000

Impairment
12/20 x $5m = 3,000
8/20 x $5m = 2,000

Impaired value
9,000
6,000

Note that no impairment loss is allocated to the inventories as IAS 36 states that an impairment loss
shall not reduce the carrying value of an asset below recoverable amount.The recoverable amount of
the inventories is their NRV, and they are already held at cost which is lower than this.

1
1
1

121

LSB_F7_Rev Kit:297mm x 210mm

28/10/09

13:37

Page 122

FINANCIAL REPORTING (INTERNATIONAL)

20. Elite Leisure

Statement of financial position for Elite as at 30 September 2009

Cruise ship (W2)


Propulsion system
Cabins
Fabric
Income statement for Elite for the year ended 30 September 2009
Depreciation (W2)
Propulsion system
Cabins
Fabric
Write off of old propulsion system (W2)
Repainting of fabric

(W1) Carrying value of components at 30 Sept 08

Fabric
Cabins
Propulsion system

Cost
$m
300
150
100

CV at 30 Sept 08
Write off

Cost new system


Depreciation 5/50 x 140
Cabin upgrade

CV at 30 Sept 08
Upgrade
Depreciation 1/5 x 110

Fabric

Notes

CV at 30 Sept 08
Painting
Depreciation 300/25yrs

126
88
192
222
406

Accd
depreciation
$m
8/25 X 300 = 96
8/12 x 150 = 100
30/40 x 100 = 75
SFP
$m
25
(25)

140
(14)
126
22
50
60
22
110
(22)
22
88
204

(12)
22
192

122

1
1

1
1
1
1
1

CV
$m
204
50
25
IS
$m
25
14

22

20
12

The old propulsion system is written off and therefore the carrying value of $25m must be charged to
profits.
The new propulsion system is capitalised at cost and depreciated over 50,000 hours.

$m
14
22
12
25
20

(W2) Carrying value of components at 30 Sept 2009


Propulsion system

$m

4 marks
for
explanator
y notes.

LSB_F7_Rev Kit:297mm x 210mm

28/10/09

13:37

Page 123

ANSWERS
The Cabin upgrade is capitalised as it enhances the economic benefits of the cabins by extending their
useful life.
The upgrade plus the carrying value of the cabins prior to the upgrade is depreciated over the new
remaining useful life.
The repainting of the fabric does not enhance the economic benefits of the ship and is therefore
charged to profits.
The fabric continues to be depreciated over 25 years as before.

21. Linnet
(a)

Construction contracts

Accounting for construction contracts in accordance with IAS 11 is dependent upon whether the
outcome of a contract can be measured reliably.
A contract can be measured reliably when:

it is probable that the economic benefits associated with the contract will flow to the entity

the contract costs can be identified and measured reliably.

total contract revenue

stage of contract completion at reporting date.

In the case of a fixed price contract the following must also be measured reliably:

contract costs to completion

Where a contract can be measured reliably, revenue is recognised by reference to the stage of
completion.

Where a contract can not be measured reliably, revenue is recognised only to the extent that
contract costs incurred are recoverable.

This is an application of the concept of accruals.

(b)

This is an application of the concept of prudence.

Income statement for Linnet for the year ended 31 March 2009

Revenue (W3)
Cost of sales (W3)

Gross loss (W3)

Statement of financial position of Linnet at 31 March 2009

Current assets

Amounts recoverable on construction contracts (W4)

$m
70
(81)
222
(11)

1
2
1

$m
59

See W4

123

LSB_F7_Rev Kit:297mm x 210mm

28/10/09

13:37

Page 124

FINANCIAL REPORTING (INTERNATIONAL)


Workings

The outcome of this contract can be estimated reliably and therefore income statement amounts are
recognised by reference to the stage of completion:
(W1) Expected profit

$m
300
(195)
(45)
222
60

Contract price
Costs to date
Estimated costs to complete

Note that rectification costs are not taken into account at this stage (although they are
contract costs).This is because they are expensed in the period in which they are incurred
rather than spread across the contract.

(W2) Stage of completion


Sales value to date
2222222
Contract price

220*
22
300

73.3%
$m

* Sales value to date:


Work certified to 29 Feb 09 (100/90 x 180)
Work complete in March 09
(W3) Income statement
Revenue (work certified)
Cost of sales ( )
Profit (73.3% x 60 (W1))

Cumulative
$m
220
176
222
44

200
20
222
220
Taken previously
$m
(150)
(112)
222
(38)

Current year
$m
70
(64)
222
6

Rectification costs must be recognised in the year incurred and therefore the current year
figures calculated are amended as follows:
Revenue
Cost of sales (64 + 17)
Loss

(W4) Gross amount due to or from customers


Costs to date
Profit to date (W3)
Less: progress billings
Less: rectification costs
Due from customers

124

$m
70
(81)
222
(11)
$000
212
44
(180)
(17)
222
59

1
1
1
1

LSB_F7_Rev Kit:297mm x 210mm

28/10/09

13:37

Page 125

ANSWERS

22. Torrent

Income statement for Torrent for the year ended 31 March 2009
Revenue (W3)
Cost of sales (W3)
Gross profit (W3)

Statement of financial position of Torrent at 31 March 2009

$m
14.8
(14.5)
222
0.3

See w3
See w3
See w3

$m

Current assets
Amounts due from customers (W4)
Amounts due to customers (W4)
Receivables (14 12.6) + (2 1.8)

5.8
(1.5)
1.6

See w4
See w4
1

Workings

(W1) Estimated profit on contracts


Contract price
Costs to date (excluding
rectification costs)
Costs to completion
Expected profit/loss

Alfa 08
$m
20

Alfa 09
$m
20

(5)
(10)
222
5

(11.5)
(3.5)
222
5

(W2) Stage of completion

Alfa 08
Alfa 09
Sales basis:
work certified
222222
Contract price

Profit/loss to
be recognised

5.4 x 100/90
22222
20
30%

30% x 5 = 1.5

Beta
$m
6
(2)
(5.5)
222
(1.5)

Ceta
$m
12
(4)
(6)
222
2

Beta
Ceta
costs basis: costs to date
222222
total costs

12.6 x 100/90
22222
20
70%

70% x 5 = 3.5
(cumulative)

n/r as
loss making
Full loss
(1.5)

4
2
10
40%

40% x 2 =
0.8

125

LSB_F7_Rev Kit:297mm x 210mm

28/10/09

13:37

Page 126

FINANCIAL REPORTING (INTERNATIONAL)


(W3) Income statement figures

Alfa and Beta are assessed on the sales basis and therefore revenue is equal to work certified;
cost of sales is a balancing figure.
Ceta is assessed on the costs basis and therefore cost of sales is equal to costs incurred to
date; revenue is a balancing figure

Revenue
(Work
certified/ )
Cost of sales
( /costs incurred)

Profit / loss (W2)

Alfa
cumulative
$m

Alfa 08
$m

Alfa 09
$m

Beta
$m

Ceta
$m

Total
$m

14

4.8

14.8

(10.5)
22
3.5

(4.5)
22
1.5

(6)
22
2

Amendment for rectification costs in Alfa


Revenue
Cost of sales
Profit

Costs to date
Profit / loss to date
Rectification costs
Progress billings
(work certified)

Due from customers


Due to customers

(4)
22
0.8

Alfa 09
$m
8
(6)
(1)
222
1

As above
Rectification costs

(W4) Amounts due to / from customers

(3.5)
22
(1.5)

Alfa
$m
12.5
3.5
(1)

Beta
$m
2
(1.5)

Ceta
$m
4
0.8

(14)
222
1

(2)
222

222
4.8

1.5

(13.5)
22
1.3
Total
$m
14.8
(13.5)
(1)
222
0.3
Total
$m

1 1/2 + 1 +
1
1 + 1/2 +
1/2
1/2 + 1 +
1/2

1
1
1

5.8
1.5

23. Bowtock

Income statement of Bowtock for the year ended 30 September 2009

Depreciation (52,000/5 years)


Finance costs (800 + 1,872)
Statement of financial position for Bowtock as at 30 September 2009
PPE (52 (10.4 x 9/12) 10.4)
Current liabilities
Finance lease obligation (W)
Non-current liabilities
Finance lease obligation (W)
126

$
10,400
2,672

1
1

$
33,800

11,376

21,696

LSB_F7_Rev Kit:297mm x 210mm

28/10/09

13:37

Page 127

ANSWERS
Working

b/f at 1
Oct

42,400
33,072

y/e 30 Sept 08
y/e 30 Sept 09
y/e 30 Sept 10

Int x
3/12

800
624

c/f
52,000
43,200
33,696

Liability due > 1 year is $21,696


Liability due < 1 year is $11,376 (33,072 21,696)

Interest
Year 1 of lease
Year 2 of lease

Note

40,000 x 8%
31,200 x 8%

Repayt
on 1 Jan
(12,000)
(12,000)
(12,000)

Total
3,200
2,496

c/f at
1 Jan
40,000
31,200
21,696

Int x
9/12
2,400
1,872

c/f at
30 Sept
42,400
33,072

9m
2,400
1,872

3m
800
624

Interest is charged on the capital liability outstanding after each repayment is made. Due to the fact
that the lease repayments occur 3m into the financial year, each years interest falls 9m within one
financial year and 3 m within the next.

24. Triangle

Marks

(i) New processing plant

The initial measurement of the processing plant on 1 April 2008 should include both the
construction cost and the present value of future decontamination costs:
Construction cost
PV of decontamination

$000
15,000
5,000
222
20,000

At this stage a provision of $5m for the decontamination costs should also be made, as there is a
legal obligation to decontaminate the plant.The other recognition criteria for a provision are met
as the future decontamination costs are probable and can be reliably measured.

In the year ended 31 March 2009, the total cost of the plant should be depreciated by $2million
($20million/10 years). Therefore at the year end the plant will have a carrying value of $18million
($20million $2 million).

One year of discounting in the provision should also be unwound, resulting in a finance cost of
$400,000 ($5m x 8%). This amount will also increase the carrying value of the provision to
$5.4million.

The treatment adopted by the management of Triangle is therefore wrong in that the $5million
provision is not made in full and not treated as part of the cost of the asset.

127

LSB_F7_Rev Kit:297mm x 210mm

28/10/09

13:37

Page 128

FINANCIAL REPORTING (INTERNATIONAL)


Marking guide

Plant cost to include decontamination costs


Initial cost $20m
Provision for $5 to be made
Meets provision recognition criteria
Depreciation charge $2m
Year end carrying value of plant $18m
Discount to be unwound
Finance cost of $400,000
Provision CV of $5.4m

(ii)Fraud

1
1
1
1
1
1
1
1
1

Max 8

The fraud was discovered on 15 May 2009.This is after the 09 reporting date.

Assuming that the accounts have not been authorised for issue by this date, the fraud should be
treated as an event after the reporting period.

$210,000 of the fraud related to year ended 31 March 2009 and its discovery provides additional
evidence of conditions at the reporting date.Therefore this is an adjusting event and the loss
should be charged to the income statement in the year.

The remaining $30,000 of the loss has arisen after the reporting date and is therefore a nonadjusting event. The financial statements should not be amended for this amount, however it may
be disclosed if considered material.
Marking guide

Event after the reporting period


$210,000 is adjusting event as provides extra evidence
$210,000 charged to income statement in X5
$30,000 is non-adjusting event
Disclose if material

(iii) Insurance

1
1
1
1
1

Max 5

Triangle has a contingent asset, in other words, an uncertain item of income. IAS 37 states that
contingent assets are recognised only where their receipt is virtually certain.

This is not the case here, as the insurance company has dismissed the claim made by Triangle, and
as yet there is no legal opinion on the issue.
If the contingent asset is assessed as probable of receipt, it should be disclosed in Triangles
accounts. Otherwise it should be ignored.
Marking guide

Claim is a contingent asset


Recognise where virtually certain
Not virtually certain in this case
Disclose if probable
Ignore otherwise

128

1
1
1
1
1

Max 4

LSB_F7_Rev Kit:297mm x 210mm

28/10/09

13:37

Page 129

ANSWERS
(iv) Sale and repurchase

This transaction is an example of a sale and repurchase. Commercial substance prevails over legal
form in the recording of such a transaction.
If the substance of the transaction is a true sale, then it should be recorded as such.

A true sale has occurred where the risks and rewards of ownership of the goods are transferred
to the purchaser. Indicators that this is not the case here and this is not a true sale are:

Triangle retains the goods on its premises

Triangle may repurchase the goods at any time within 4 years at a less than market value price

Triangle pays the storage costs for the 4 years during which it has the option to repurchase (to
be reimbursed by Factorall if the option is not taken)

The sale was not at market value

Interest accrues on the cost of repurchase from the date of sale

Therefore this transaction should be accounted for as a financing arrangement as follows:

The proceeds of $5million should be recorded as a loan, accruing interest at 10% per annum.

The inventory should continue to be recorded as an asset in the accounts of Triangle. The costs of
storage should form part of the cost of this inventory in accordance with IAS 2.

Therefore the income statement for the year ended 31 March 2009 will include finance costs of
$500,000 ($5m x 10%).

The statement of financial position at that date will include inventory of $3.3 million ($3million +
$300,000 storage costs) and a loan of $5.5million ($5million + $500,000 interest).

Marking guide

Commercial substance over legal form


Sale recorded where transfer risks and rewards
Application of risks and rewards to scenario
Conclude this is financing arrangement
Loan in SFP $5.5m
Inventory in SFP $3.3m
Finance costs $500,000

1
1
1 per point max 4
1
1
1
1

25. Atkins

(i) Consignment inventory

Max 8

Marks

Atkins buys cars from a manufacturer to sell on to the public.The manufacturer transfers such
cars to Atkins showroom to facilitate such sales, however Atkins only pays the manufacturer on
the date that a car is sold to a customer (or after 6 months if sooner).

The issue here is when Atkins should record the purchase of the cars (and the manufacturer
should record the sale), and so whose inventory the cars are whilst they are in the showroom.
Arguably the purchase (sale) could be recorded when the cars are physically transferred or when
Atkins pays the manufacturer.

IAS 18 requires that a sale is recorded when the risks and rewards of ownership are transferred.
Therefore an assessment of which entity has these risks and rewards whilst the cars are in Atkins
showroom is required.

129

LSB_F7_Rev Kit:297mm x 210mm

28/10/09

13:37

Page 130

FINANCIAL REPORTING (INTERNATIONAL)


If Atkins bears the risks and rewards then it should record the purchase of the cars and an
associated liability to pay the manufacturer when the cars are delivered.

If the manufacturer bears the risks and rewards then Atkins should not record the purchase until
it has secured the onward sale (or after 6m if earlier).
Assessment of risks and rewards

The price paid by Atkins is fixed at the date of delivery meaning that Atkins bears the risk of a
subsequent fall in the retail price, but also benefits from any subsequent increase in the retail
price.

1 per point
to a max
of 4

Atkins pays a display charge to the manufacturer until the cars are paid for.This is, in effect,
interest on the payable outstanding in Atkins accounts. Therefore Atkins bears the risk of slow
moving goods (and so higher display charges.
Atkins right of return has never been exercised, indicating that this may not be a true benefit.
i.e.Atkins may suffer a penalty such as reduced supply if it does exercise this right.

The manufacturer can recall the cars at any time.This is a benefit retained by the manufacturer.

Conclusion

The above assessment indicates that Atkins bears the majority of risks and rewards of ownership,
and therefore should record the purchase of the cars when they are transferred to the showroom.
The display charge will accrue on a monthly basis.

1
2222
Max 6

(ii) Development land

Atkins has purchased 5 plots of development land, subsequently selling 3 to Landbank with an
option to repurchase them at any time within 3 years.

The issue here is whether the substance of this transaction is a true sale, or a loan secured on the
3 plots of land.

An assessment of the facts indicates that the commercial substance is a loan made by Landbank to
Atkins:

The sale is for 25% less than market value indicating that it is not a true sale

Atkins will pay a premium on the repurchase which can be considered interest.

Atkins can repurchase the land at any time over 3 years at the full market value as at October
X2 rather than at the market value on the date of repurchase
Landbank has a put option to require Atkins to repurchase the land after 3 years. Landbank is
likely to exercise this if the market value of the land has fallen below the option price of
$3.2million.

Land considered sold to Landbank


Income statement of Atkins for the year ended 30 September 2009
Profit on sale of land ($2.4 m (3/5 x $2.4m))

Statement of financial position of Atkins at 30 September 2009


Investment property (2/5 x $2m)

130

1 per point
to
maximum
of 2

$000
1,200
800

1
1

LSB_F7_Rev Kit:297mm x 210mm

28/10/09

13:37

Page 131

ANSWERS
Transaction considered financing arrangement
Income statement of Atkins for the year ended 30 September 2009
Finance costs ($2.4m x 10%)

Statement of financial position of Atkins at 30 September 2009


Investment property
Loan (2.4m x 1.1)

$000
240
2,000
2,640

26. CB
(a)

Earnings per share

Profits attributable to ordinary shareholders

$725,000

Weighted average number of ordinary shares in the period

Bonus fraction

Therefore EPS is
PE ratio
Price per share
(b)

Earnings per share

Bonus
fraction
x 7.5/7.3

No
3,000,000
3,750,000
Share price before issue
222222222
TERP
TERP
4 old shares @ $7.50 =
1 new share @ $6.50 =
2
5
$725,000
=
2222
3,694,349

Significance of PE ratios

1/2

1 1/2

Marks

Calculation of EPS and PE ratio

1 Feb 08 28 Feb 08
1 March 08 31 Jan 09

$30
$6.5
22
$36.5
19.6c

625
22
19.6c

Time
X 1/12
X 11/12

No shares
256,849
3,437,500
2222
3,694,349

so TERP $36.5/5 = $7.30

= 31.88

PE ratios provide indication of market confidence in a company. A calculated PE ratio is the multiple
of earnings per share that an investor is willing to pay for each share, so a PE ratio of 10 reveals that
an investor is willing to pay 10 times the current year EPS to buy a share. Therefore the investor is
confident that the company will make future profits and perform well.

1
1

1 mark per
point to
max 4

PE ratios are of course a product of the share price in the market, and therefore it should be noted
that any general market slump will affect this ratio.

CB has a PE ratio of 31.88 compared to an average 28.4 for the business sector and 42.5 of its main
competitor.This indicates that CB is viewed as one of the better performers within the sector,
however the market has more confidence in its competitor in terms of future performance.

131

LSB_F7_Rev Kit:297mm x 210mm

28/10/09

13:37

Page 132

FINANCIAL REPORTING (INTERNATIONAL)

27. QRS

Marks

Accounting treatment of financial instruments


Non-redeemable preferred shares

IAS 32 requires that non-redeemable preferred shares are classified as equity.The nominal value of the
shares issued should be recorded within share capital, and any premium within the share premium
reserve.
The dividend should be reported in the statement of changes in equity.

1
1

Convertible bonds

The proceeds of the issue should be split between debt and equity as follows:

IAS 32 considers convertible bonds to be a hybrid instrument. It should therefore be classified part as
debt and part as equity, to represent the conversion rights.
The amount of debt is calculated as the present value of the bonds assuming there was no option
to convert, and based on a market interest rate for a bond without conversion rights.
Equity is the residual.

The debt element is presented as a liability; the equity element is presented within reserves; the
interest on the bond is all presented as interest in the income statement.
Split accounting for bonds:
Interest year 1: 360,000
Interest year 2: 360,000
Interest year 3: 360,000
Interest year 4: 360,000
Repayment year 4: 6m

Debt element
Equity element therefore

DF
1/1.08
1/1.082
1/1.083
1/1.084
1/1.084

PV$
333.333
308,642
285,780
264,611
4,410,179
2222
5,602,545
397,455
2222
6,000,000

28. Harper
(a)

Discontinued operations

(i) Users of the financial statements are particularly interested in the future of an entity what its
profits may be, whether it is worthwhile investment and so on.

The financial statements themselves are, however, historic, and therefore users must try to
forecast future performance based on past events. The separate presentation of discontinued
operations helps with this process by, in effect, identifying that portion of the profit or loss of an
entity which will not continue in future years.
Where a discontinued operation is loss making, future overall profits should increase; where a
discontinued operation is profitable, future overall profits will decrease, and investors may
question the strategy of divesting of a profitable operation.

Thus, the disclosure of discontinued operations also helps investors to assess the stewardship of
the management and how well they are running the company.

132

1
1
1

2
1

Marks
1 mark per
point to a
max 3

LSB_F7_Rev Kit:297mm x 210mm

28/10/09

13:37

Page 133

ANSWERS
(ii) Operating profit in the absence of discontinued operations information
Operating profit (100 x 110%)

Operating profit based on the information provided

Operating profit
Continuing activities (70/90) x 110%
Acquisitions (50 x 12/6m x 110%)
Discontinued
(b) Earnings per share

Gamma
$m
110

Toga
$m
110

Gamma
$m

Toga
$m

77

22
77

99
110

22
209

(i) Earnings per share is calculated by dividing the profit attributable to the ordinary shareholders of
an entity by the weighted number of ordinary shares in issue throughout the period.
An upwards trend in profits will be reflected in an upwards trend in EPS, provided that there are
no new issues of shares during the period concerned.

If there is an issue of shares, the increased capital is likely to be utilised to increase profits and so
there will likely be an upwards profit trend.The increased number of shares may, however mean
that a conflicting downwards EPS trend is identified.

(ii)

From an individual shareholder perspective, EPS is a more important measure of performance, as


EPS provides an indication of how much dividend per share may be paid in the future.
Basic EPS =

Dilutive factors:
Convertible loan stock
Increase in profits (due to no interest):
Increase in shares on conversion:

$50,m
200,000,000

1
1
1

25c

8% x 200m x 75%
= 12m
$200m/$100 x 70
= 140m
$12m
= 8.5c
22
140m
8.5c is lower than basic EPS therefore this is a
dilutive factor.

1
1

133

LSB_F7_Rev Kit:297mm x 210mm

28/10/09

13:37

Page 134

FINANCIAL REPORTING (INTERNATIONAL)


Share options

No effect on profits

Increase in free shares:

Diluted EPS

funds raised through options:


Number of shares at MV:
Therefore free shares:

50m x $1.50
$75m / $2.50
50m 30m

= $75m
= 30m
= 20m

Basic EPS

$50,m
22222
200,000,000

= 25c

$50,m
22222
200mm + 20m

= 23c

$50m + $12m
22222
220m + 140m

= 17c

Introduce effect of share


options (most dilutive factor)
Introduce effect of convertible
loan stock
Therefore fully diluted EPS is 17c.

(iii) Basic earnings per share is calculated based on actual profits for the year and the number of
ordinary shares actually in issue throughout the year.

A number of items in the statement of financial position may, however, lead to a future increase in
the number of ordinary shares. An example of such an item is share options. The exercise of
share options will increase the number of ordinary shares in issue and result in a lower EPS. It is
therefore said to be a dilutive factor.

1 per point
to a max
of 4

In some cases an increase in the number of shares will also have an effect on profits. For example
the conversion of loan stock into ordinary shares will increase the number of shares in issue and
also increase profits through the reduction in interest payable. This may result in an overall
decrease in EPS or it may result in an increase, depending on the proportionate increases in both
shares and profits. If such a conversion results in decreased EPS, it is a dilutive factor; if it results
in increased EPS, it is said to be anti-dilutive.

Diluted EPS adjusts basic EPS for all dilutive factors. In other words, it takes into account all future
potential dilutive issues of ordinary shares based on evidence in the current statement of financial
position, and their impact on profits. It may therefore be considered a worst case scenario EPS.

(c)

Note, however, that diluted EPS does not take account of future profit forecasts or timescales for
share issues, nor does it consider anti-dilutive factors. It is therefore not a prediction of future EPS.

In many cases the tax charge reported in a companys income statement is not equivalent to the tax
rate multiplied by the profit before tax.There are a number of reasons for this:

Tax is charged on taxable profits rather than the accounting profit before tax. Taxable profits are
calculated by adjusting the accounting profit for certain expenses which are not tax allowable, for
example depreciation, and income which is not taxable, for example dividends from companies in
the same jurisdiction.

The year-end tax charge is an estimated amount, and any difference between this reported amount
and the amount finally paid must be adjusted through the following years tax charge as an under
or overprovision.
The tax charge in the income statement includes any movement in the provision for deferred tax.

The tax charge in the income statement is unlikely to be the same amount as tax paid in the
statement of cash flows for the following reasons:

134

For most companies, tax is paid a number of months after the year to which it relates.Therefore
the amount reported as the tax charge in the income statement relates to the current year,
whereas the amount reported as tax paid in the statement of cash flow for the same period is
likely to be the previous years tax charge (with adjustment for any over or underprovision).

LSB_F7_Rev Kit:297mm x 210mm

28/10/09

13:37

Page 135

ANSWERS

Deferred tax, which is included in the income statement tax charge, is not relevant to the
statement of cash flows, as this is not tax currently payable.

29. FW

Briefing paper

Marks

From: AN Accountant

This briefing paper analyses and interprets the effects of making a $500 million environmental
provision on FWs key ratios, taking into account the possible effects on the public perception of FW.
2008

2009 Before provision

Return on equity

24.7%

Return on net assets

17.7%

Gearing

82%

Operating profit margin

10.1%

Earnings per share

12.2c

1,670
222222
(4,954 + 5.656)/2
= 31.5%
2,080
222222
(9,016 + 10,066)/2
= 21.8%
4,410
22
5,656
= 78%
2,080
22
20,392
= 10.2%
1,002
22
6,000
= 16.7c

2009 After provision


(see workings below for
adjusted figures)
1,170
222222
(4,954 + 5,156)/2
= 23.1%
1,580
222222
(9,016 + 9,566)/2
= 17.0%
4,410
22
5,156
= 85.5%
1,580
22
20,392
= 7.7%
502
22
6,000
= 8.4c

Commentary on ratios

Without exception, before considering the environmental provision, FWs key ratios improved, in
some cases significantly between 2008 and 2009. The most improvement was seen in return on
equity and return on net assets.

The introduction of the $500m provision into the accounts and subsequent recalculation of the key
ratios, however results in a deterioration of all results compared to 2004. Whilst return on equity and
return on net assets are only slightly below the 2004 level, the operating profit margin and earnings
per share are significantly lower.

Whilst many members of the public are unfamiliar with return on equity / net assets, earnings per
share is a commonly used measure, and the fall in this from 12.2c to 8.4c is likely to have a negative
impact on the publics perception of FW. Similarly, profit margins are widely understood by most
investors, and the fall from 10.1% to 7.7% will have a similar impact.

Although the key ratios show deterioration, it should be remembered that FW, by creating the
environmental provision, is demonstrating its commitment to green issues.These are widely viewed
as of increasing importance by the public, and therefore the companys creation of a provision to
restore damaged habitats and compensate displaced persons will show FW in a positive light.

1 1/2

1 1/2

1 1/2

1 1/2

1 1/2

1 for
discussion
of ratios
without
provision

2 for
discussion
of ratios
after
provision
2 for
impact on
public
1 1/2 for
positive
impact of
green
actions

135

LSB_F7_Rev Kit:297mm x 210mm

28/10/09

13:37

Page 136

FINANCIAL REPORTING (INTERNATIONAL)


Workings: adjustments
Net assets at 28.2.09
Share capital and reserves at 28.2.09
Operating profit
Profit before tax
Profit for the period

10,066
5,656
2,080
1,670
1,002

Provision
(500)
(500)
(500)
(500)
(500)

Adjusted
9,566
5,156
1,580
1,170
502

30. LMN

Consignment inventory

Marks

The transaction described is an example of consignment inventory, typical of the motor industry. In
this type of transaction, goods are delivered from a manufacturer to a retailer and held on the
retailers premises until sold.

1 for
identifying
transaction

2 for
explanation
of
accounting

Legally, the manufacturer retains title to the goods until such time as they are sold to a third party.
The transaction is, however, accounted for according to the substance of the transaction:

If the risks and rewards of ownership are deemed to have passed to the retailer at the date the
manufacturer delivers the goods, then they are included as inventory in the retailers statement of
financial position for the time that they remain on the retailers premises.

If the risks and rewards of ownership are retained by the manufacturer when the goods are
delivered to the retailer, then the inventory remains in the manufacturers statement of financial
position until it is sold on to a third party.

The terms of LMNs contract with IJK must be examined in order to assess which company holds the
risks and rewards of ownership of the cars during the time that they are held on LMNs premises:

LMN is required to insure the vehicles whilst they remain on its premises.This is a risk of
ownership and indicates that the inventory is an asset of LMN at delivery.

The price at which vehicles are supplied is determined at the date of delivery.Therefore IJK is
unable to pass on any subsequent price changes, which indicates that the vehicles are inventory of
LMN at delivery.

LMN is entitled to use any of the vehicles supplied to it for demonstration purposes and road
testing.This is an example of a transferred reward and indicates that the vehicles are inventory of
LMN at delivery.

1 per point
for
assessment
of risks
and
rewards
Max 5

A penalty is payable by LMN if the number of kilometres driven in any vehicle exceeds a certain
level.The number of kilometres driven in test drives will clearly increase the longer that the
vehicle goes unsold and therefore this may be viewed as an example of slow movement risk borne
by LMN.This is a further indicator that the vehicles are inventory of LMN at delivery.

LMN has the right of return without penalty. It is unclear whether this right is exercised by LMN,
but assuming that it is, the risk of return is borne by IJK and this is an indicator that the vehicles
remain IJKs asset.

The majority of the contract terms therefore indicate that the risks and rewards of ownership of the
vehicles are transferred to LMN.

The vehicles should therefore be recorded as their asset from delivery, with LMN booking a purchase
and IJK booking a sale on the date of delivery.

136

2 for
conclusion

LSB_F7_Rev Kit:297mm x 210mm

28/10/09

13:37

Page 137

ANSWERS

31. EFG
(a)

Key ratios

Gross profit margin

429 x 100%
22
1,810
= 23.7%

Operating profit margin

Return on total capital employed

Gearing

(b)

Key ratios after adjustment


Gross profit margin

Marks
1/2

193 x 100%
22
1,810
= 10.7%

1/2

193 x 100%
22
(769 + 248)
= 19%

1/2

248 x 100%
22
769
= 32.5%

1/2

Revenue

No adjustment required

Gross profit

Per income statement


Add back extra depreciation on revaluation
($250,000/25 years)
Inventory adjustment:
Opening less closing inventory using
AVCO valuation (197,000 201,000)
(4,000)
Opening less closing inventory using
FIFO valuation (208,000 218,000)
(10,000)
222
Therefore add back difference:

Revised gross profit margin

Operating profit margin

445 x 100%
22
1,810
= 24.6%

Revenue

no adjustment required

Operating profit

Per income statement


Extra depreciation (see above)
Inventory adjustment (see above)
Add back bonus paid to skilled staff
Fixed salaries to replace bonus

Revised operating profit margin

255 x 100%
22
1,810
= 14.1%

$000

$000
429
10

6
222
445

$000
193
10
6
96
(50)
222
255

1/2

1/2

1
1/2

1/2
1/2
1/2
1/2
1/2
1/2

137

LSB_F7_Rev Kit:297mm x 210mm

28/10/09

13:37

Page 138

FINANCIAL REPORTING (INTERNATIONAL)


Return on total capital employed
Operating profit
Capital employed

Revised ROCE

Tutorial note

(see above)
Per statement of financial position (769 + 248)
Less: revaluation surplus
Adjustments to retained profit
Inventory b/f (208 197)
Inventory adjustment (for year)
Bonus
Fixed salaries
255 x 100%
22
880
= 29%

11
6
96
(50)
222
880

1/2
1/2

1/2
1/2
1/2
1/2

the preferred shares should be reclassified from equity to debt, however this does not affect
overall capital employed, and therefore the adjustment is not made here.

The revaluation surplus would not exist if J applied EFGs accounting policy.The $50,000 of the
surplus transferred to retained earnings since the revaluation needs no adjustment, as this
compensates for the lower retained earnings as a result of the revaluation policy.

The extra depreciation of $10,000 which forms an adjustment to operating profit is not adjusted
for in capital employed as it has been transferred from the revaluation surplus to retained earnings
by way of a reserves transfer (shown in the SOCE).

Retained earnings b/f must be adjusted for the difference in inventory valuation based on the
AVCO and FIFO measurement methods.

Gearing

Long-term liabilities

Per statement of financial position


Reclassification of preferred shares

Shareholders funds

Per statement of financial position


Reclassification of preferred shares
Less: revaluation surplus
Adjustments to retained profit
Inventory b/f (208 197)
Inventory adjustment (for year)
Bonus
Fixed salaries

Revised gearing

138

$000
255
1,017
(200)

298 x 100%
22
582
= 51.2%

$000
248
50
222
298
769
(50)
(200)
11
6
96
(50)
222
582

1/2
1/2
1/2
1/2
1/2

LSB_F7_Rev Kit:297mm x 210mm

28/10/09

13:37

Page 139

ANSWERS
Advice to directors

After adjustment to Js results to bring its accounting policies into line with those of EFG, the key
ratios are:
Gross profit margin
Operating profit margin
ROCE
Gearing

24.6%
14.1%
29%
51.2%

The operating profit margin and ROCE exceed the acquisition criteria set by EFG, however the gross
profit margin and gearing ratios fall short. The gross profit margin is only 0.4% lower than the
required level of 25%, and would be easily addressed, particularly with the effect of post-acquisition
synergies. Gearing, however, is more than double the acceptable level, and on this basis, the
acquisition should not be pursued.

32. Breadline
(a)

Subsidiary accounts

Marks

Subsidiary companies are required to produce individual entity accounts for statutory purposes in
most jurisdictions. These entity accounts are of limited use in the assessment of company
performance largely due to related party transactions with the parent company and / or other group
subsidiaries:

The subsidiary is likely to have engaged in transactions that it would not have done as a standalone company, possibly sales to group companies or loans from group companies.

Related party transactions are often not conducted at arms length. Examples may include:

Trading at non-commercial transfer prices which may artificially reduce the subsidiarys
company profit (in the case of a low transfer price for sales or a high transfer price for
purchases) or increase them (in the case of a high transfer price for sales or a low transfer
price for purchases)

Loans which are made interest free, or with interest rates lower than those commercially
available, so impacting profits.

The parent company may pass back operating expenses to the subsidiary, so lowering profits.

The parent company may provide management services free of charge to the subsidiary, so
artificially inflating its profits.

Certain functions are likely to be pooled at a group level and recharged to the subsidiary on a
basis decided by management of the parent.

1
1
1
1

(max 5)

139

LSB_F7_Rev Kit:297mm x 210mm

28/10/09

13:37

Page 140

FINANCIAL REPORTING (INTERNATIONAL)


(b)

REPORT
To:

Chief Executive

Date:

January 2009

From:
Subject:

An Accountant

Financial position of Breadline

This report analyses the overall financial position of Breadline. The company is a customer of
Judicious and a possible acquisition target for the Judicious Group.

The 2009 statement of financial position of Breadline reports net assets of $3.7million, and increase of
$1.2million since the previous year. The movements in net assets may be further analysed as follows:
Non-current assets

Non-current assets have increased from $2million to $4.12million. This is due to:

The disposal of freehold premises with a previous carrying value of $1.25million (based on
revalued amount) and acquisition of leasehold premises with a carrying value of $2.5million
Investment in plant resulting in the carrying value more than doubling from $750,000 to
$1.62million.

This increase in non-current assets has adversely affected non-current asset turnover which has fallen
from 3.25 to 2.06 times. It is possible that this will increase once again in the future as the full benefit
of the new plant is felt and translates into further increases in revenue.

As regards the premises, Breadline continues to trade from the same business address, suggesting that
the freehold and leasehold premises are one and the same.This indicates that Breadline may have
obtained financing in the form of a sale and leaseback transaction. Thus a one off profit of
approximately $1.25million ($2.5m - $1.25m) is included in cost of sales, and Breadline would have
benefited from cash proceeds of $2.5million.
It should be noted, however that finance costs relating to the leased property are not evident in the
statement of comprehensive income, nor is a lease liability reported in the SFP.This matter requires
further investigation, and details of the following should be sought, if indeed this is a leaseback
transaction:

Interest rate implicit in the lease

Length of the lease.

As regards the additions to plant, this may be indicative of an increased market or market share, and
so expansion of Breadlines operations. The existence of increased market share should be
investigated in order to justify the heavy investment in plant.
Working capital

Both inventory and receivables of Breadline have increased since 2008: Inventory has increased by
54% from $240,000 to $370,000 and receivables by 60% from $600,000 to $960,000.

Both of these increases are in part justified by the 30% increase in revenues (from $6.5million to
$8.5million) and the 50% increase in cost of sales as adjusted for the one-off profit ($4.81million to
$7.2million).

The increase in receivables in particular is disproportionate to the increase in revenue, as evidenced


by an increase in receivables days from 34 to 41. This may indicate a poor credit control function or
may be the result of offering increased credit terms to new customers in order to access new market
share. The terms offered to all customers should be investigated in order to assess Breadlines
position in this respect.

Inventory days remains relatively unchanged in 2009 at 19 days.The main concern here is whether this
period of stock holding is appropriate for a bakery business. Fresh produce is unlikely to last for so
140

1 for
format

1 per valid
extended
comment
to a
maximum
of 10
1 for each
matter for
concern /
investigatio
n to a max
of 5.

LSB_F7_Rev Kit:297mm x 210mm

28/10/09

13:37

Page 141

ANSWERS
long, and if old goods are being retailed, there will be a reputational impact which in turn will impact
sales and profits.The exact nature of inventories should be investigated in order to ascertain whether
this is indeed an issue.
The payables balance of Breadline has increased by 75% from $590,000 to $1.03million.Within this
amount is $100,000 owing to Judicious in X0 and $340,000 in 09. Some increase in payables is to be
expected given the increased activity of Breadline resulting in a 50% increase in cost of sales (see
above), however a 75% increase seems excessive.

Calculation of payables days reveals an overall increase from 44 days to 63 days. IN respect of the
Judicious balance, the increase is from 46 days to 103 days. The terms offered by our company are 30
days, and therefore this jump is a matter for some concern. Investigations should be undertaken as to
why this is the case, and indeed the credit period offered by other suppliers should be investigated.
There are clear indications of a cash crisis within Breadline, indicated in particular by an obvious
inability to pay suppliers and the reduction of the bank balance from a credit balance of $250,000 to
an overdraft of $220,000.

Liquidity problems are further evidenced by a current ratio reduced from 1.85:1 to 1.06:1, and quick
ratio reduced from 1.44:1 t0 0.78:1.
Financing

Breadline appears to have been financed during X1 from the following sources:

The sale and leaseback transaction which realised approximately $2.5million

A share issue which raised $600,000

An issue of loan notes which raised $500,000


An overdraft of $220,000

The money raised has been used to purchase plant and pay a $900,000 dividend.

As a result of the increase in loan notes and overdraft, gearing has increased from nil to 16%. Any
increase in gearing brings increased risk, largely due to the increase in fixed interest costs and
reduction in profit. At the current time profits are more than sufficient to cover interest, however the
interest charge in the income statement does appear to be artificially low due to:

The absence of finance lease interest

The non-commercial rate of interest on the loan note (6% below market rates).

It is assumed that the source of the low interest loan notes (and indeed the injection of capital) is
Breadlines parent company Wheatmaster. Wheatmaster is also the beneficiary of the large dividend,
which brings about the question of why monies are being passed around the Wheatmaster Group in
such a fashion. Certainly, if Judicious were to acquire Breadline, financing is likely to require
restructuring.
Conclusion

On the basis of the information contained within the financial statements of Breadline, its financial
position is relatively unhealthy, particularly in terms of liquidity. Furthermore there is clear
manipulation of the accounts, presumably in order to make them appear attractive to would be
purchasers of the company.The transference of cash around the Group through share issues/lending/
dividends masks the true position of Breadline and as such at this time an acquisition is not
recommended.

141

LSB_F7_Rev Kit:297mm x 210mm

28/10/09

13:37

Page 142

FINANCIAL REPORTING (INTERNATIONAL)


Appendix

Financial position ratios


Gearing
Current ratio
Quick ratio
Inventory days
cost of sales in X1 adjusted
to 7,200 (5,950 + (2,500 1,250))
Receivables days

Payables days (based on adjusted


cost of sales)
Payables days Judicious

Non-current asset turnover

2009
500 + 220/(500+220 +3,700)
16%

2008
Nil

850/590
1.44:1

960/1250
0.78:1

1090/590
1.85:1

1330/1250
1.06:1

370/7200 x 365
19 days

240/4,810 x 365
18 days

960/8,500 x 365
41 days

600/6,500 x 365
34 days

340/1200 x 365
103 days

100/800 x 365
46 days

1030/7200 x 365
52 days

8,500/4,120
2.06

590/4,810 x 365
45 days

6,500/2,000
3.25

33. Comparator
(a)

Problems of ratios

The use of ratios to assess company performance is widespread, and in particular they are useful in
identifying areas of the financial statements which require further investigation.

In order to achieve a full analysis of a companys position and performance, however, ratios should be
used in conjunction with the full financial statements and non-financial information about the
company.
When using ratios regard should also be given to the following factors which may limit their
usefulness:

142

1 mark per
relevant
ratio to a
maximum
of 6

Many ratios can be calculated in different ways; comparison is only valid as long as the method of
calculation is the same

Ratios mask the size of the companies being compared. Larger companies which can achieve
economies of scale and operate globally will have very different ratios from small localised
companies within the same industry.

Ratios are based on the financial statements, which in turn may be prepared under varying
accounting policies. For example, a company which applies the revaluation model to PPE may
report very different ratios from one which does not.

Estimates and judgements within the financial statements such as depreciation methods will
also affect related ratios and may limit comparability.
Ratios are only truly useful when comparing one company over time or two very similar
companies within the same industry.

1 per valid
problem to
a maximum
of 7

LSB_F7_Rev Kit:297mm x 210mm

28/10/09

13:37

Page 143

ANSWERS

Many ratios are based on year-end figures in the statement of financial position, for example
receivables days. These are therefore affected by seasonality and possible window dressing.

Certain ratios are easily manipulated, for example the current ratio can be siginificantly altered by
using cash to pay a liability immediately prior to the year end.

Additional problems of using ratios provided by interfirm comparison services are based upon the
fact that the companies on which the ratios are based are not identified:

(b)

The size of the companies used may not be appropriate for comparison with an individual
company
Year ends may vary, resulting in skewed results due to seasonality

The industry ranges may be broad and not correspond directly to the business of an individual
company.

Ratios for Comparator


ROCE

Interfirm
comparatives
22.1%

Revenue/capital employed

1.8 times

Gross profit margin

30%

Net profit before tax margin

12.5%

Current ratio

1.6:1

Quick ratio

0.9:1

Inventory holding period

46 days

Accounts receivable
collection period

45 days

Accounts payable payment


period

55 days

Debt to equity

40%

Dividend yield

6%

Dividend cover

3 times

Comparator
34.6%

1/2

2,425
2222
335 + 300

3.8%

1/2

22.9%

1/2

186
22
2,425

7.7%

1/2

1.19:1

1/2

0.64:1

1/2

54 days

1/2

186 + 34
2222
335 + 300

555
22
2,425

595
22
500

595 275
2222
500

275/1,870 x 365

320/2,425 x 365
350/1,870 x 365
300
22
335

(90,000/600,000 shares)
22222222
6
96/90

48 days

1/2

68 days

1/2

89.5%

1/2

2.5%

1/2

1.07

1/2

143

LSB_F7_Rev Kit:297mm x 210mm

28/10/09

13:37

Page 144

FINANCIAL REPORTING (INTERNATIONAL)


(c)

REPORT
By:

Subject:

An Accountant

Analysis of financial performance of Comparator

1 mark for
format

This report analyses the financial performance of Comparator based on a comparison with sector
averages.
Profitability

Comparators ROCE of 34.6% is significantly higher than the sector average of 22.1%. Its revenue is
3.8 times its capital employed compared to an industry average of 1.8 times.

1 mark per
point to
max 4 for
profitability

Both of these ratios indicate that Comparator uses the resources at its disposal more efficiently than
other companies in order to create revenue and profits.
It should be noted, however, that these ratios are likely to be skewed by:

the relatively low carrying value of Comparators non-current assets. These assets are clearly old
with a written down value of just $540,000 on a cost of $3.6million.

the exceptional write off of inventory to profits (affecting ROCE only)

the fact that some companies included within the sector averages will adopt the revaluation model
for non-current assets, so pushing ROCE down.

Comparator is clearly at the stage where it should be considering the replacement of its non-current
assets; at such a time as this takes place, its ROCE and revenue/capital employed will reduce
significantly, and may be in line with sector averages.
The gross profit margin of Comparator is 22.9% compared to a sector average of 30%. Analysis of
this figure is difficult without information on Comparators position within the industry, but the low
figure may be due to:

Comparator being a relatively small player which is unable to achieve the economies of scale that
other companies can.

Comparator concentrating on a less premium product than the companies in the sector average
calculations, and so intentionally achieving lower margins.

The net profit margin of 7.7% is significantly lower than the sector average of 12.5%. Part of the
reason for this is likely to be a knock-on effect from the low gross margins. A further explanation is
the one-off inventory write off.A recalculation of Comparators net profit margin gives 12.6% ((186 +
120)/2425). This recalculation suggests that the lower gross profit margin of Comparator is balanced
by lower operating costs than sector average in order to achieve a consistent net margin.
Working capital management

Both Comparators current and quick ratios are below sector average suggesting potential liquidity
and cash flow problems.

This is further evidenced by high working capital days compared to sector average:

inventory days of 54 compared to 46 sector average

receivables days of 48 compared to 45 average

payables days of 66 compared to 55 average

Whilst the high inventory days are not inappropriate for the product, it should be remembered that
this is calculated after the write off of obsolete items. It may suggest that further items require
writing off. Alternatively, Comparator may have increased year-end inventory due to an order
received or upcoming sale event. Either way, associated with increased inventory are increased holding
costs which will adversely affect profit.
144

1 per point
to max of
3 for
working
capital

LSB_F7_Rev Kit:297mm x 210mm

28/10/09

13:37

Page 145

ANSWERS
Receivables and payables days should be assessed compared to credit terms.Without this information,
analysis is of limited use. If receivables do exceed terms given, this is indicative of poor credit control
and possible bad debts; if payables exceed terms received, late payment is likely to result in stops to
supply and a breakdown of relationships with suppliers.
Solvency

Comparator has a very high gearing ratio of 90% compared to 40% sector average.This is also
evidenced by the interest cover of just 2.This places Comparator in a relatively risky position
whereby raising further finance is difficult.This will become more of an issue as Comparator requires
investment in non-current assets in the short term.

1 per point
to max of
2 for
solvency

Dividends

The dividend yield of 2.5% is significantly lower than the sector average of 6%.This reduces the
attractiveness of Comparator to investors.This, together with the gearing ratio makes the raising of
finance through a share issue an unlikely option.

Dividend cover of 1.07 is also low, indicating that Comparator pays out almost all of its profit for the
year as a dividend.This is an unwise policy for a company which is already suffering from cash flow
problems and with limited access to future funds.

1 per point
to max of
2 for
dividends

Conclusion

Comparator appears to be in a relatively weak financial position based on sector averages. Its
performance ratios are boosted only by the low carrying values of its non-current assets, and with
regard to its long and short term financing, it may be argued that the companys going concern status
is in doubt.

34. Bigwood
(a)

Statement of cash flows for Bigwood for the year ended 30 September 2009

Profit before tax


Finance costs
Depreciation (W1)
Loss on disposal of plant (1,200 + 50)
Increase in inventory
Increase in receivables
Increase in payables

$000

Cash generated from operations


Interest paid
Income taxes paid (W2)
Net cash from operating activities

Cash flows from investing activities


Purchase of PPE (W3)
Costs of disposal of PPE
Net cash used in investing activities

(10,500)
(50)

$000
700
300
3,800
1,250
(1,400)
(50)
950
222
5,550
(300)
(480)
222
4,770

Marks
1/2
1/2

1
1

1/2

1/2
1/2

1
1

222
(10,550)

1/2

145

LSB_F7_Rev Kit:297mm x 210mm

28/10/09

13:37

Page 146

FINANCIAL REPORTING (INTERNATIONAL)


Cash flows from financing activities
Issue of share capital ((5,000 + 1,000) 3,000)
Proceeds from long-term borrowings (3,000 1,000)
Dividends paid
Net cash used in financing activities

Net decrease in cash and cash equivalents


Cash and cash equivalents at beginning of period
Cash and cash equivalents at end of period

Workings

(W1) Depreciation
Balance b/f
Disposal (3,000 1,200)
Charge for year ( )
Balance c/f

(W2) Income tax paid


Payable b/f 450 IS tax charge
Paid ( )

Payable c/f

(W3) Purchase of PPE


Cost of PPE b/f
Disposals
Additions ( )

Cost of PPE c/f


(b)

REPORT
By:

Subject:

3,000
2,000
(600)

1
1

222
4,400
222
(1,380)
450
222
(930)

1/2
1/2

$000
3,000
(1,800)
3,800
222
5,000
$000
250
(480)
222
220
$000
9,500
(3,000)
10,500
222
17,000

An Accountant

Performance and position of Bigwood

1 for
format

This report analyses the performance and position of Bigwood.The analysis if based on income
statements for the years ended 30 September 2008 and 2009; statements of financial positions at
those dates, a statement of cash flows for the year ended 30 September 2009 and the ratios included
in the appendix to this report.
Performance

Bigwoods ROCE at 9.3% in 2009 is significantly lower than the 2008 equivalent figure of 33.9%. This
drop initially indicates that Bigwood is not using the assets at its disposal as efficiently as in the
previous year to create profits.The result is, however, skewed by the significant increase in noncurrent assets during the year from a carrying value of $6.5m to $12m.This increase represents the
acquisition of 5 new stores (primarily for the sale of clothing) and the refurbishment of others. It is
unclear as to when this investment took place during the year, but it is very possible that its effect on
profits has not yet been felt in full, as evidenced by a fall in clothing sales per square metre from $445
to $333. In future years ROCE is likely to rise again as the new stores become established and
generate increased revenues and profits.

A further reason for the fall in ROCE is low profits in 2009. The overall gross profit margin has
dropped from 20% to 16% and the net margin from 7.1% to 2%.
146

1 per valid
explained
point to a
max of 4
for performance

LSB_F7_Rev Kit:297mm x 210mm

28/10/09

13:37

Page 147

ANSWERS
The gross margin can be further analysed between clothing and food:

The gross margin relating to clothing has halved from 18.6% to 9.4%
The margin relating to food has increased from 25% to 32%.

The fall in clothing margin is surprising given the involvement of a famous designer in creating the
clothing items. Such involvement would usually result in a premium being charged. It may be that
increased competition on the high street has meant that this is not possible, however the main reason
for the fall in margins is the increased depreciation included within cost of sales relating to clothing as
a result of the new stores and increased floor space, 92% of which relates to clothing.
The increase in the margin relating to food may be explained by Bigwood achieving economies of
scale on its supplies, or sourcing alternative suppliers who charge lower prices.Alternatively, Bigwood
may have marketed its food as premium produce and thus been able to increase prices. Such
marketing costs may contribute to 2009s higher operating expenses.

In any case, the decision to allocate the majority of the new selling space to clothing seems
questionable given the higher margins in the food business.The reasons for this should be investigated.
The overall fall in net margin is partly the result of the drop in gross margin. Other reasons include:

Increased operating expenses (12% of revenues in 09 compared to 10% in 08). This is accounted
for by the $1.25m loss on refurbishment of stores. If this exceptional item is stripped out, then
operating expenses are $1.5m, which is just 7% of revenues
The increased interest expense related to $2million of new borrowings and the overdraft.

Interest cover has fallen from 25 times in 08 to 3.3 times in 09.This is due to both the increased
interest expense and the fall in profits. 3.3 times interest cover is acceptable, although further falls in
this ratio would be a matter for concern.
Position

As already discussed, the non-current asset base of Bigwood has increased considerably with the
acquisition of new stores and refurbishment of others.
This has been financed by way of:

A new $2m loan

$3m raised through the issue of shares.

1 per valid
explained
point to a
max of 4
for
position

As a result of these movements, Bigwoods gearing ratio has increased from 17% to 28%. On the face
of it, and in light of the interest cover, this level of debt appears acceptable, however it should be
assessed against the industry norm.
The inventory holding period of Bigwood has increased from 39 to 68 days in respect of clothing and
dropped from 17 to 15 days in respect of food.

The increase in clothing inventory days is indicative of obsolete stock; the very high inventory days
also suggest that Bigwood does not operate in high fashion (where new collections are launched
weekly in some cases) and therefore the new designer may have created some pieces which are too
cutting edge for its usual customers and so have not sold. A write down is likely to be required; this
will adversely affect profits.

The food inventory days has fallen, however remains high given the nature of the product.This level
should be assessed against food shelf life to assess whether it is appropriate.

Payables days is high at 59 in 09, and has increased from 50 in 08. This must be compared to credit
facilities offered in order to come to a valid conclusion as to whether it is appropriate. Based on
normal credit terms of 30 days, it is very high and could result in interest being charged, damaged
supplier relationships, and in the worst case scenario, stopped supplies. The increase is also indicative
of cash flow problems as further evidenced by the cash / overdraft balance.
147

LSB_F7_Rev Kit:297mm x 210mm

28/10/09

13:37

Page 148

FINANCIAL REPORTING (INTERNATIONAL)


The current ratio of Bigwood is very low at 0.71:1 (0.77:1 in 08).A low ratio is acceptable given the
nature of the business (retailing to the public means that the level of receivables is negligible), however
Bigwoods high payables and overdraft mean that the ratio is still lower than it should be.
Cash position

The business is clearly in a weak position as regards cash, with a healthy balance of $450,000 in 08
becoming an overdraft of almost $1m in 09.Analysis of the statement of cash flows identifies that this
is almost exclusively due to the heavy investment in new stores. The cash problem is likely to be
short term, as in due course these stores should generate cash and profits. In the meantime the
company would benefit from formalising the overdraft into a loan agreement in order to reduce bank
charges.
Assuming that cash generated by operations remains static in the coming year (i.e. disregarding an
expected growth), the $4.7million generated will be sufficient to repay the overdraft and the loan.
Share price

The share price of Bigwood has fallen significantly over the year from $6 to $3.This is in part due to
the share issue but may also suggest a lack of market confidence in the company, possibly associated
with the general economy or possibly associated with Bigwoods financial position. Reasons for this
should be further investigated.

1 per valid
explained
point to a
max of 2
for cash

1 for
explanation
of fall in
MV

Conclusion

Bigwood has recently undergone a massive expansion, increasing its selling space by 35%. As yet the
effects of this have not been felt in terms of profit and cash generation. In future years the position
should improve, although it is questionable whether the company is right to allocate the majority of
the increased floor space to the lower margin clothing product.
Appendix

Gross profit margin (overall)


Share of total revenue
Clothing
Food

Operating expenses as % of revenue


Sales per square foot
Clothing
Food

148

2009
3,750/23,000
16%

2008
3,900/19,600
20%

69%
31%

80%
20%

16m/48,000
$333
7m/6,000
$1,167

15.6m/35,000
$445
4m/5,000
$800

2,750/23,000
12%

1,900/19,600
10%

1 for
conclusion

1 mark per
relevant
ratio to a
max of 3

LSB_F7_Rev Kit:297mm x 210mm

28/10/09

13:37

Page 149

ANSWERS

35. Tabba
(a)

Statement of cash flows for Tabba for the year ended 30 September 2009

Net cash from operating activities

$000
50
(40)
260
2,200
(4,600)
(250)
(300)
222
(2,680)
(700)
(500)
1,100
222
(2,780)
(260)
(1,350)
222
(4,390)

Net cash used in investing activities

(2,900)
12,000
40
950
222
10,090

Profit before tax


Investment income
Finance costs
Depreciation (W1)
Profit on disposal (12 7.4)
Release of grant (W2)
Increase in insurance claim receivable
Increase in inventory
Increase in receivables
Increase in payables

Cash generated from operations


Interest paid
Income taxes paid (W3)

Cash flows from investing activities


Purchase of PPE (W4)
Proceeds of sale of PPE
Interest received
Grant proceeds
Cash flows from financing activities
Redemption of loan
Proceeds from long-term borrowings
Finance lease payments (W5)
Net cash used in financing activities

Net increase in cash and cash equivalents


Cash and cash equivalents at beginning of period
Cash and cash equivalents at end of period

(4,000)
800
(1,100)
222
(4,300)
222
1,400
(550)
850

Marks
1/2

1/2
1/2

1
1
1

1/2
1/2
1/2
1/2
1/2

See W4
1/2

1/2
1/2

1
1

See W5

1/2

1/2

149

LSB_F7_Rev Kit:297mm x 210mm

28/10/09

13:37

Page 150

FINANCIAL REPORTING (INTERNATIONAL)


Workings

(W1) Depreciation $000


Balance b/f
Disposal
Charge ( )
c/f

(W2) Government grant $000


Balance b/f (900 + 400)
Awarded in year
Amortised ( )

c/f (1,400 + 600)

(W3) Income taxes


Balance b/f (1,200 current + 500 deferred)
Income statement
Paid ( )
c/f (100 + 200)

(W4) PPE
Balance b/f
Disposal
Additions by way of lease
Additions ( )

c/f

(W5) Finance lease


Balance b/f (1,700 + 800)
New lease
Payment ( )

c/f (2,000 + 900)


(b)

$000
4,400
(1,200)
2,200
222
5,400
$000
1,300
950
(250)
222
2,000
$000
1,700
(50)
(1,350)
222
300
$000
20,200
(8,600)
1,500
2,900
222
16,000
$000
2,500
1,500
1,100
222
2,900

Change in financial position of Tabba

The net assets of Tabba remain similar at 30 September 2009 ($8.55m) as they were at the previous
year end ($8.45m). Further analysis of the companys financial statements reveal, however, significant
changes in the elements of the financial position of the company.
Non-current assets

The carrying value of Tabbas non current assets has fallen from $15.8million to $10.6million.
$2.2million of this is accounted for by the depreciation charge for the year, and the remainder of the
difference is due to the sale of the companys factory.This initially indicates that Tabbas capacity has
reduced and that it is may face operational problems. In this case, however, the sale is part of a sale
and operating lease back transaction. Such transactions are commonly used by companies wishing to
realise cash, however retain the use of their assets.

150

The reduction in non-current assets is therefore not a matter for concern in itself, although the need
for the sale and leaseback in order to realise cash may be indicative of liquidity or solvency problems.

1/2
1/2

1/2

1/2

1/2

1 per valid
explained
point to a
max of 8

LSB_F7_Rev Kit:297mm x 210mm

28/10/09

13:37

Page 151

ANSWERS
Current assets and working capital

The cash position has improved significantly compared to 08, when there was an overdraft of
$550,000.The increase in cash to a positive balance of $850,000 is largely explained by the sale of the
factory on the sale and leaseback transaction, although a grant of $950,000 and loan issue of $800,000
also contribute to the positive cash inflow.

What is particularly worrying is that the company is not generating a positive cash flow from
operations.This is a sustainable source of income, whereas financing transactions and grant receipts
are not ongoing.This lack of generation of cash by the companys operations indicates solvency
problems in the near future. Indeed it is clear that in the current year the sale and leaseback
transaction has been used to finance the cash outflows suffered by the companys operations and
those required to meet tax and interest obligations.
The negative cash flow from operations can be explained in two main ways:

The company makes very small profits in the first place, indeed if the profit on disposal of
$4.6million, the increase in insurance claim of $300,000 and grant amortisation of $250,000 were
stripped out,Tabbas profit of $50m becomes a loss of $5.1m
Management of working capital appears inefficient, with increases in both inventories and
receivables.

The increased inventories of Tabba at the 09 year end are explained by the write off at the year end of
inventories in the previous year. Indeed if the $1.5m insurance claim were added to the 08 inventories,
the 09 stocks are reduced on the previous year.This may be in line with reduced sales levels, however
income statements for the two years are required in order to conclude on this matter.
Trade receivables are 19% higher than the previous year at $3.1million.Again, without income
statement information, the reasons for this are unclear, although it may be that the credit control
function is not operating efficiently and a write off is required. Certainly, in order to improve their
cash position,Tabba should be ensuring that its debts are paid on time.

The payables of the company have increased significantly,Whilst this has a positive impact on the
statement of cash flows, it is a further indicator of liquidity problems, and Tabbas inability to meet its
obligations.
Liabilities

Tabba has two main financing arrangements:

6% loan notes

Finance leases (4 years remaining)

It is questionable in the light of the above analysis whether the obligations in respect of these items
can be met when they fall due.

The future cash status of Tabba is likely to be little improved; it is unlikely that a bank would finance a
company in this position and the only expected future cash inflows are those of the insurance claim
and tax repayment.These amounts are not substantial enough to absorb the operating cash losses and
therefore unless Tabba sells more non-current assets it is likely to become insolvent in the near future
and lose its going concern status.

151

LSB_F7_Rev Kit:297mm x 210mm

28/10/09

13:37

Page 152

FINANCIAL REPORTING (INTERNATIONAL)

36. Nedberg
(a)

Statement of cash flows for Nedberg for the year ended 30 September 2009

Net cash from operating activities

$m
870
30
320
50
130
(90)
20
222
1,330
(480)
(310)
145
222
685
(20)
(130)
222
535

Net cash used in investing activities

(250)
(500)
20
50
222
(680)

Net cash used in financing activities

450
200
(320)
222
330

Profit before tax


Finance costs
Depreciation
Loss on disposal
Amortisation of development expenditure (W1)
Government grant release (W2)
Goodwill impairment
Increase in inventory
Increase in receivables
Increase in payables

Cash generated from operations


Interest paid (W3)
Income taxes paid (W4)

Cash flows from investing activities


Purchase of PPE
Development expenditure
Proceeds of sale of PPE (W5)
Grant received
Cash flows from financing activities
Issue of share capital (W6)
Proceeds from long-term borrowings (300 100)
Dividends paid
Net increase in cash and cash equivalents
Cash and cash equivalents at beginning of period

Cash and cash equivalents at end of period

152

185
(115)
222
70

Marks
1/2
1/2
1/2
1/2

1 1/2
1 1/2
1/2
1/2
1/2
1/2

1 1/2
2

1/2
1/2

2 1/2
1/2

3
1

1/2

1/2
1/2

LSB_F7_Rev Kit:297mm x 210mm

28/10/09

13:37

Page 153

ANSWERS
Workings

(W1) Development expenditure


Balance b/f
Capitalised
Amortisation charge (
c/f

(W2) Government grant

$m
300
50
(90)
222
260

Balance b/f (900 + 400)


Awarded in year
Amortised ( )
c/f (1,400 + 600)

(W3) Loan interest


Accrual b/f
Income statement 30 Paid (
Accrual c/f

(W4) Tax

Balance b/f (160 current + 140 deferred)


Income statement
Paid ( )
c/f (130 + 310)

(W5) PPE

Balance b/f
Revaluation
Additions
Depreciation
Disposal CV
c/f

Proceeds of sale = 50 loss + 70 CV = $20m


(W6) Share issue
Revaluation reserve b/f
Revaluation
Transfer
Bonus issue ( )
Revaluation reserve c/f

$m
100
500
(130)
222
470

$m
5
(20)
222
15
$m
300
270
(130)
222
440
$000
1,830
200
250
(320)
(70)
222
1,890

$m

200
(10)
50
222
140

153

LSB_F7_Rev Kit:297mm x 210mm

28/10/09

13:37

Page 154

FINANCIAL REPORTING (INTERNATIONAL)

c/f
Bonus issue
Share issue (
b/f

(b)

Share capital Share premium


$m
$m
750
350
(50)
(200)
(250)
222
222
500
100

Total
$m
450

Commentary on financial position of Nedberg

Nedberg has an overall cash inflow of $330m.The main sources of the cash inflows are:

$685m cash generated from operations

$450m proceeds of a share issue

$200m proceeds of a loan issue.

Of these inflows, the cash generated by operations is sustainable and will continue in future years,
assuming that Nedberg continues to trade as present.
The financing inflows are one-off cash flows, and Nedberg can not raise finance in this was on an
ongoing basis.

Whilst the operating cash flows amply cover the ongoing cash outflows of interest, tax and a dividend,
the one-off cash inflows have been used to finance one-off expenditure, namely the purchase of new
plant and development expenditure.

Both of these items of expenditure are expected to result in future benefits to Nedberg, in the form
of revenues, profits and in turn cash inflows.They are therefore an efficient way to spend the proceeds
of the financing activities.

IN terms of cash from operations, an operational profit of $900m ($870m + $30m finance costs)
equates to a cash flow of $685m. Where operational cash flow is less than operating profit,
explanation should be sought,

In this case the lower amount is largely due to the management of working capital and in particular
the increases in receivables and inventory.Although these have a negative impact on cash flow, the
increases may have valid business reasons, for example due to increased activity or providing
extended credit to new customers or stocking up ahead of a big order. If these reasons are not in
existence, the increases may be evidence of poor working capital management, which should be
addressed.
Finally the issue of the dividend payment should be considered.A dividend of $320m is paid in the
same year that the company has raised $650m in finance. More efficient management of cash flows
would have seen the dividend withheld for a year and less external financing sought.

154

1 per valid
explained
point to a
maximum
of 5

LSB_F7_Rev Kit:297mm x 210mm

28/10/09

13:37

Page 155

ANSWERS

37. Minster
(a)

Marks

Statement of cash flows for Minster for the year ended 30 September 2009

Net cash from operating activities

$000
142
(20)
40
255
45
30
110
(25)
(205)
222
372
(28)
(54)
222
290

Net cash used in investing activities

(410)
(180)
(10)
5
222
(595)

Net cash used in financing activities

265
120
(100)
222
285

Profit before tax


Investment income
Finance costs
Depreciation
Amortisation (180 135)
Decrease in inventory
Decrease in receivables
Increase in amounts on construction contracts
Decrease in payables
Cash generated from operations
Interest paid (40 (8% x 150) unwinding)
Income taxes paid (W1)

Cash flows from investing activities


Purchase of PPE (W2)
Purchase of software
Purchase of investments (W3)
Investment income (20 15 gain on investments)
Cash flows from financing activities
Issue of share capital
Proceeds from long-term borrowings
Dividends paid 5c x 2m (W4)
Net increase in cash and cash equivalents
Cash and cash equivalents at beginning of period (35-40)
Cash and cash equivalents at end of period

(20)
(5)
(25)

1/2

1/2
1/2
1/2
1/2
1/2
1/2
1/2
1/2

1
1

1 1/2
1/2

1
1

1/2

1/2

155

LSB_F7_Rev Kit:297mm x 210mm

28/10/09

13:37

Page 156

FINANCIAL REPORTING (INTERNATIONAL)


Workings
(W1) Income taxes

$000
75
57
(54)
222
78

b/f (50 + 25)


Income statements
Cash paid ( )
c/f (60 + 18)

(W2) PPE
B/f
Depreciation
Revaluation
Provision
Cash additions (
c/f

$000
940
(255)
35
150
410
222
1,280

(W3) Investments
b/f
Gain
Additions (

c/f

(W4) Share issue


b/f
Bonus issue 1 for 4
Cash issue ( )
c/f

(b)

$000
125
15
10
222
150
Share capital
$000
300
75
125
222
500

Share premium
$000
85
(75)
140
222
150

Total
$000
265

No shares at 1 July 500/4 = 2m

Commentary on financial performance and position of Minster


Performance

In the absence of a comparative statement of comprehensive income for the year ended 30
September 2008 or industry comparatives, commentary on Minsters performance is difficult.

At a very basic level, however, the company is profitable making 21% gross profits and 6% net profits.
It also has a more than adequate interest cover of 4 (162/40). It should, however, be noted that the
dividend of $100,000 exceeds the profit for the year of $85,000, meaning that Minster is using its
reserves to pay a dividend.
Position

The statement of financial position reveals an increase in net assets from 2008 to 2009 of 21%.This
overall increase can be explained by the following factors:

156

PPE has increased $340,000 to $1,280.This is in part due to the $35,000 revaluation in the year,
but more importantly due to the acquisition of a coal mine and related plant costing $410,000.
This acquisition strengthens Minsters position by increasing revenue-making capacity.

1 mark
per valid
point to a
maximum
of 10

LSB_F7_Rev Kit:297mm x 210mm

28/10/09

13:37

Page 157

ANSWERS

The acquisition of software costing $180,000. Again this purchase provides access to future
revenues which will become evident in the income statement of future years.

The acquisition of new investments held at fair value through profit or loss, and unrealised gain on
existing investments.The unrealised gain indicates a good investment strategy on Minsters part.
Fair value through profit or loss investments are those investments held for trading; at any time,
Minster could crystallise the gain by selling the investments to realise cash.
A reduction in both inventories and trade receivables. In the absence of historical income
statement information, it is unclear whether this reduction is the result of a decrease in trading
activities.Alternative explanations are more efficient working management or changes in stockholding policy / credit terms offered.

A deterioration in the net bank position with a net overdraft of $5,000 in 08 becoming a net
overdraft of $25,000 in 09.The cash position is further explained by the statement of cash flows
(see below)

The creation of an environmental provision related to the new mine has limited impact on the net
assets of Minster due to its inclusion as both a provision and as part of the cost of the asset.
A decrease in trade payables of $205,000 indicating prompter payment of suppliers. If payment is
earlier than credit terms this may indicate poor use of what is in effect a free credit facility.

The overall increase in net assets is financed by:

A share issue raising $265,000, the success of which indicates market confidence in Minster

The issue of $120,000 9% loan notes which increases gearing from nil to 7%.Although this level of
gearing is low and therefore acceptable, it does mean an increase in mandatory interest payments
and so introduces risk to the business.

Cash position

Minster reports a net cash outflow in 09 of $20,000. $385,000 has been raised through the share and
loan issue, and $372,000 cash has been generated from operations.The net overall outflow is due to
the very high expenditure on the new mine and related plant and the software.As mentioned above
this use of cash will lead to future economic benefits and strengthens Minsters future prospects.
Cash generated by operations is healthy and well above the related profit figure. If Minster continues
to generate at least similar amounts of cash in the year ahead, it will be in a position to pay off the
loan and restore its bank balance to a positive position.

157

LSB_F7_Rev Kit:297mm x 210mm

28/10/09

13:37

Page 158

FINANCIAL REPORTING (INTERNATIONAL)

38. Rytetrend
(a)

Statement of cash flows for Rytetrend for the year ended 31 March 2009

Net cash from operating activities

$000
3,400
300
460
7,350
700
620
850
870
350
222
14,900
(460)
(910)
222
13,530

Net cash used in investing activities

(15,550)
222
(15,550)

Net cash used in financing activities

3,000
2,000
(4,000)
(430)
222
570
222

Profit before tax


Equipment installation (part of PPE cost)
Finance costs
Depreciation (W2)
Loss on disposal (W2)
Decrease in inventory (3,270 2,650)
Decrease in receivables (1,950 1,100)
Increase in payables (2,850 1,980)
Increase in warranty provision (500 150)
Cash generated from operations
Interest paid
Income taxes paid (630 + 1,000 720)

Cash flows from investing activities


Purchase of PPE (W1)
Cash flows from financing activities
Issue of share capital ((11,500 + 1,500) 10,000)
Proceeds from long-term borrowings
Redemption of long-term borrowings
Dividends paid
Net increase in cash and cash equivalents
Cash and cash equivalents at beginning of period
Cash and cash equivalents at end of period

158

1,450
400
(1,050)

Marks
1/2
1/2
1/2

1
1

1/2
1/2
1/2
1/2

1
1

See W1

1/2
1/2
1/2
1/2

1/2
1/2

LSB_F7_Rev Kit:297mm x 210mm

28/10/09

13:37

Page 159

ANSWERS
Workings

(W1) PPE

Cost b/f
Disposal
New equipment
Additions other than equipment (
Cost c/f

Therefore total cash additions:


New equipment net of trade in allce (8,000 500)
Installation costs of new equipment (not capitalised)
Other additions

(W2) Depreciation
B/f
Charge for the year ( )
Disposal (6,000 x 20% x 4 years)

c/f

Therefore loss on disposal:


Proceeds (trade in allowance)
CV at disposal (6,000 4,800)

(b)

REPORT
By:

Subject:

Loss

$000
27,500
(6,000)
8,000
7,750
222
37,250
7500
300
7,750
222
15,550

1/2

1/2
1/2

$000
10,200
7,350
(4,800)
222
12,750
500
1,200
222
(700)

AN Accountant

Performance and position of Rytetrend

This report analyses the performance and position of Rytetrend for the years ended 2008 and 2009. It
is based on the income statements and statements of financial position for these two years and a
statement of cash flows for 2009.

1 for intro

Profitability

Rytetrends profit for 2009 is $2.4million, an increase of $800,000, or 50% on the previous year.
Further analysis of the income statement reveals that this increase is largely due to a 35% jump in
sales revenue to $31.8 million.The investment in non-current assets, and particularly the mid-year
purchase of new display equipment is likely to have contributed to this increase.

The increased level of operations has, however, resulted in a corresponding increase in costs. Cost of
sales has increased by 40% to $22.5 million. This is proportionately greater than the increase in
revenue, as evidenced by a fall in gross margin from 32% to 29%. This disproportionate increase
suggests that Rytetrend may have reduced its prices in order to achieve the volume growth in sales.
The increase in costs will also be due to:

1 mark per
point to a
max of 4
for performance

the non-current asset investment which results in higher depreciation (it should be noted that a
further $60,000 depreciation relating to the capitalised installation costs will increase cost of sales
to $22.56m)

increased warranty costs associated with increased levels of sales.

159

LSB_F7_Rev Kit:297mm x 210mm

28/10/09

13:37

Page 160

FINANCIAL REPORTING (INTERNATIONAL)


Operating expenses have increased by 18% to $5.44million, however it should be noted that $300,000
of staff costs included within this amount should have been capitalised as part of the cost of the new
equipment.A revised figure for operating expenses is $5.14 million, meaning an increase of just 12%
since 2008.This increase seems reasonable given the increased volume of activitiy.
Interest costs for 2009 are in line with 2008, despite the restructuring of Rytetrends loan financing.
The beneficial effect of the reduction of debt by half, and replacement of a 10% loan with a 6% loan is
negated by interest costs relating to the $1.05million overdraft of $200,000. The companys financing
arrangements are discussed in more detail below.
Cash flows

It appears that the 50% increase in net profits is masking Rytetrends cash flow and liquidity issues.
Overall the company has seen a net cash outflow in 2009 of $1.45 million, causing the $1.05million
overdraft.

This cash deficit can be explained by the heavy investment in non-current assets of more than
$15.5million. The issue of shares and new debt are largely used to redeem old debt, meaning that just
$1million of the proceeds of these issues contribute towards this investment.

1 mark per
point to a
max of 4
for cash
flows

The remainder of the funding is provided by cash generated from operations and the overdraft.
Whilst cash generated by operations is healthy, contributing $13.5 million cash to the business (after
the payment of tax and interest), the company appears to have miscalculated the availability of funds
for investment in non-current assets.
Certainly, an overdraft is a very expensive way of financing a business, and Rytetrend should seek to
formalise the overdraft into a cheaper loan agreement, secured on its non-current assets.

Position

As discussed above, there has been heavy investment in non-current assets, which is beneficial for
Rytetrend in terms of improving and growing the business. It may be that the full effect of these
additions has not been felt yet, and future revenues will continue to grow as a result of the
investment.

1 mark per
point to a
max of 4
for
position

Inventory and receivables have both decreased since 2008 (despite the increase in sales volume),
whilst payables has increased.The fall in inventory and receivables are further indicators that
Rytetrend has a liquidity problem: it is calling in debts sooner and avoiding tying up funds in inventory.
This approach to working capital is likely to be unsustainable, with goodwill with customers being lost
and the possibility of stock outs.
Conclusion

Rytetrend is a profitable company with a relatively healthy statement of financial position, particularly
in respect of non-current assets. It must, however, address its financing structure through negotiation
of a new loan in order to ease its apparent liquidity problems. If this can be achieved in the short
term, the company has a good long-term prospect.

39. Update
(a)

Problems of the historical cost convention

Application of the historical cost convention means that items in the financial statements are carried
at their cost when purchased.The impact of this can be very notable in the statement of financial
position, less so in the income statement.

In the statement of financial position, all assets, both current and non-current are measured at their
historical cost.Whilst this is generally a relatively up-to-date measurement as regards current assets, it
can be very outdated as regards non-current assets.
160

Marks
1 per valid
point to a
max of 6

LSB_F7_Rev Kit:297mm x 210mm

28/10/09

13:37

Page 161

ANSWERS
For example, the historic cost of land purchased a number of years prior to the reporting date would
bear no resemblance to the cost of a more recently purchased piece of similar land. Both however
provide access to similar economic benefits. It is therefore misleading to users of the financial
statements to report the pieces of land at different historic costs, and users may be lead to believe
that the more recently purchased land, measured at the higher value will provide greater benefit, due
to being a larger size or in a better position, when this is not the case.

The position becomes more complicated when the asset is depreciated, as not only are non-current
asset measurements in the SFP understated compared to current values, but depreciation is lower
than it would be under current cost accounting, so inflating profits in the income statement.

Other figures in the income statement are arguably less distorted, as they generally reflect
transactions within the previous year, and over this period, prices are unlikely to change dramatically.

In any case, the distortion of both SFP and income statement figures impacts key ratios, particularly
those which compare SFP and IS figures, such as ROCE and asset turnover. Here, income statement
values are relatively current, whereas asset values may be understated, so resulting in better ratios.

(b)

Where comparison of two companies is undertaken, the problem is more apparent when one
companys assets were purchased more recently than anothers: the company will older assets will
report better ratios than that with newer assets.Again, this is misleading as there may be no
difference between the underlying performance of the companies.
Different measurement bases.
Historical cost basis

Depreciation charge (250,000 x 20%)


Cost
Depreciation (50,000 x 3 years)

Carrying value at 31 March 2009

Current purchasing power basis


Current purchasing power value of asset 216/180 x $250,000
Depreciation charge :
CV based on 3 yrs depreciation ($300,000 x 80% x 80% x 80%)
CV based on 2 years depreciation ($300,000 x 80% x 80%)
Therefore, depreciation charge

CV of asset at 31 March 2009

Current cost basis


Current cost value of asset adjusted for productivity $320,000 x 420/480
Depreciation charge :
CV based on 3 yrs depreciation ($280,000 x 80% x 80% x 80%)
CV based on 2 years depreciation ($280,000 x 80% x 80%)
Therefore, depreciation charge

CV of asset at 31 March 2009

$
50,000
250,000
(150,000)
222
100,000
$
300,000

153,600
192,000
222
38,400
153,600

1/2

1/2

1/2

1
1

280,000

1/2

143,360
179,200
222
35,840

143,360

161

LSB_F7_Rev Kit:297mm x 210mm

28/10/09

13:37

Page 162

FINANCIAL REPORTING (INTERNATIONAL)

40. Appraisal
(a)

(b)

Ratios for Reactive

ROCE

2008
28.1%

Net asset turnover

4 times

Gross profit margin

17%

Net profit before tax margin

6.3%

Current ratio

1.6:1

Closing inventory holding period

46 days

Trade receivables collection period

45 days

Trade payables payment period

55 days

Dividend yield

3.75%

Dividend cover

2 times

Analysis of position and performance of Reactive

200 + 20
2222
1,160 - 480

4,000
2222
1,160 480

2009
32.4%

550
2222
4,000

13.8%

5%

610
2222
480

1.3:1

26 days

44 days

45 days

6%

1.67 times

200
2222
4,000

250
x 365
2222
3,450

360
x 365
2222
4,000 x 75%
430
x 365
2222
3,450
90m/400m
2222
3.75
150
2222
90

Performance

During the year, Reactive disposed of significant levels of non-current assets as part of a strategy to
purchase rather than manufacture goods.This disposal has reduced the companys asset base by
approximately $80million.

As a result, ROCE has increased from 28.1% to 32.3% and net asset turnover has increased from 4
times to 5.9 times. A recalculation of these ratios with adjustment for the disposed of plant gives
ROCE of 23.7% (220 40 / (680 + 80)) and asset turnover of 5.3 times (4,000/(680 + 80)).

The recalculated ROCE reveals that there is an underlying poor performance in 2009, although the
recalculated asset turnover does suggest an increase in turnover compared to 2008.

The increased turnover may be the direct result of the advertising campaign and rebate policy, whilst
the

The fall in the recalculated ROCE is further evidenced by the gross profit and net profit margin,
which have both decreased, the latter in spite of the $40million exceptional gain on disposal of the
plant.This reduction may be the result of :

162

The costs of the rebates offered

5.9 times

It should be noted that without full financial statements for the year ended 31 march 2008, analysis is
limited and based primarily on ratios.

Marks

1 per point
to max 10

LSB_F7_Rev Kit:297mm x 210mm

28/10/09

13:37

Page 163

ANSWERS

The advertising campaign costs

The changed cost base as a result of buying in goods rather than producing them in house.

Liquidity and working capital

The current ratio has fallen slighty, and may now be considered to be too low. The possible liquidity
issue is further highlighted by the existence of a $10million overdraft at 31 March 2009, despite the
cash inflow in the year of $120m from the sale of the plant.

IN terms of working capital, receivables days remains relatively constant, however there is a fall in
both inventory and payables days.The inventory holding period has fallen from 46 days in 2008 to 26
days in 2009, and the payables period from 55 days in 2008 to 45 days in 2009. Both may be the result
of a changed working capital policy, related to the new strategy of buying in goods.
Dividends

Dividend cover has fallen (based on the same level of dividend as 2008), giving a clear indication of
lower absolute profits than 2008. Furthermore the increase in dividend yield can only be explained by
a fall in the share price of the company, indicating a lack of market confidence in Reactive.
Conclusion

(c)

In conclusion, the sale of the plant and new strategy to buy in goods appear to have skewed the ratios
to, in many cases, appear to have improved. The underlying performance of the company, however,
appears to have weakened, as does its position, particularly in respect of cash and liquidity.
Analysis of a not-for-profit organisation

Not-for profit organisations include schools, hospitals and charities. As the name suggests, their main
objective is not the creation of wealth for shareholders, but instead to fulfil their reason for existence.
Therefore many of the ratios applicable to commercial organisations, such as ROCE or profit margins
are not applicable to these not-for-profit entities.

1 mark per
point to a
max of 5

It should be noted, however, that some ratios, particularly those relating to efficiency of operations
such as receivables days are still relevant, as are liquidity ratios.

As an alternative to assessing these entities based on profitability, a generally accepted method is to


assess them based on the 3 Es: economy, efficiency and effectiveness.
This type of assessment concentrates not only on financial key performance indicators, but also on
no-financial factors, for example:

Exam pass rates for schools

Number of patients treated for hospitals

Number of activities funded for charities.

163

LSB_F7_Rev Kit:297mm x 210mm

28/10/09

13:37

Page 164

FINANCIAL REPORTING (INTERNATIONAL)

164

LSB_F7_Rev Kit:297mm x 210mm

28/10/09

13:37

Page 165

F7
Feedback
and
Review Form

LSB_F7_Rev Kit:297mm x 210mm

28/10/09

13:37

Page 166

FINANCIAL REPORTING (INTERNATIONAL)

166

LSB_F7_Rev Kit:297mm x 210mm

28/10/09

13:37

Page 167

FEEDBACK AND REVIEW FORM

F7

Please take the time to complete this feedback and review form about the study materials that you have used for your
ACCA exams. We really appreciate your comments.
YOUR DETAILS
Name

Address

How did you use this material?

Home study (only using books)

Classroom course

Home study (books and InterActive videos)

What made you buy this material?

Saw information on LSBF website

Saw information on InterActive website

Recommendation from friend/colleague

Saw advertisement

Recommended by lecturer at college

Used LSBF/InterActive materials before


Other (please state)

STUDY MANUALS
Please make an assessment about the quality of the videos were in the following areas:
Clarity of tutor explanations

Engaging and interesting tutor


Exam focus, hints and tips

Examples and exercises


Overall Opinion

Would you use our materials again?

Yes

Very useful

Useful

Not useful

No

INTERACTIVE VIDEOS (IF USED)


Please make an assessment about the quality of the videos were in the following areas:
Very useful

Useful

Not useful

Exam focus, hints and tips

Overall opinion

Clarity of tutor explanations

Engaging and interesting tutor


Examples and exercises

Please return this form to: Paul Merison, Publications Manager, London School of Business and Finance,
8-9 Holborn, London EC1N 2LL.
167

LSB_F7_Rev Kit:297mm x 210mm

28/10/09

13:37

Page 168

FINANCIAL REPORTING (INTERNATIONAL)


Please use this space to make any additional comments that you have about either the InterActive videos,
study manuals or any other aspect of our service:

Thank you for taking your time to complete this form. Good luck in your forthcoming exams.
If you would like to make any other general comments about this manual, please forward them to
feedback@studyinteractive.org or complete our electronic feedback on
www.lsbf.org.uk/pbfeedback
168

Das könnte Ihnen auch gefallen